NPTE Practice Questions

Réussis tes devoirs et examens dès maintenant avec Quizwiz!

Which of the following compression garment pressures is contraindicated in patients with lymphedema? A. 12 mmHg B. 25 mmHg C. 30 mmHg D. 50 mmHg

D. 50 mmHg Any pressure higher than 45 mmHg causes lymphatic collapse!

If the test shown in the photograph has a positive result, which of the following tests would MOST likely also have a positive result? A. Allen test B. Froment test C. Piano keys test D. Carpal compression test

D. Carpal compression test The test shown in the photograph is the Phalen test, for which a positive result is indicative of carpal tunnel syndrome. A positive result of the carpal compression test is also indicative of carpal tunnel syndrome. Allen test: blood supply to the hands Froment test: ulnar nerve palsy Piano keys test: dysfunction of the distal radioulnar joint

A patient underwent surgical repair of a SLAP lesion 10 days ago. Which of the following interventions is LEAST appropriate at this time? A. Active isometric contraction of the biceps B. Passive humeral rotation with shoulder in scapular plane C. Pendulum exercises within the tolerated range D. Active isometric contraction of the triceps

A. Active isometric contraction of the biceps The long head of the biceps attaches to the superior labrum so you would not want to be disrupting the repair.

A 6-month-old child was referred to physical therapy for right torticollis. The MOST effective method to stretch the muscle is by positioning the head and neck into A. Flexion, left side-bending, and left rotation B. Extension, right side-bending, and left rotation C. Flexion, right side-bending, and left rotation D. Extension, left side-bending, and right rotation

D. Extension, left side-bending, and right rotation In torticollis, sternocleidomastoid muscle is most commonly affected and it becomes short. The main action of SCM, is same side bending and opposite side rotation. So, to stretch right SCM, left side bending, right rotation and extension will be the ideal position.

A physical therapist examines the viscosity and color of a sputum sample after completing postural drainage activities. The sputum is yellowish-greenish color and is very thick. The PT can best describe the sputum as: A. Fetid B. Frothy C. Mucoid D. Purulent

D. Purulent Fetid- foul smelling Frothy- seen with pulmonary edema due to heart failure, usually pink in color (think of the froth they put on a PINK STRAWBERRY milkshake) Mucoid- thick, resembles mucous Purulent- typically yellowish-green and indicates an infection

Which of the following factors is NOT considered to be a primary risk factor for atherosclerosis? A. High blood pressure B. Increased lipids C. Cigarette smoking D. Sedentary lifestyle

D. Sedentary lifestyle

A 60 y/o male pt c/o a shock-like and stabbing sensation running through his jaw line which has become frequent and excruciating lately. He says that the episodes occur usually when he goes out in cold weather or when shaving or brushing his teeth. The therapist suspect a possible cranial nerve inflammation. What is the suspected condition and the cranial nerve affected? A. Bell's palsy; CN 7 B. Trigeminal Neuralgia; CN 7 C. Bell's palsy; CN 5 D. Trigeminal neuralgia; CN 5

D. Trigeminal neuralgia; CN 5

A patient presents to a clinic with subacromial impingement syndrome. The PT chooses to use ultrasound for deep heating of the rotator cuff muscles. Which set of US parameters will be MOST effective at increasing the temperature of tissue 4 centimeters deep? A. 1.0 MHz, 100% duty cycle with 5 cm sound head B. 3.0 MHz, 100% duty cycle with 2 cm sound head C. 1.0 MHz, 50% duty cycle with 2 cm sound head D. 3.0 MHz, 50% duty cycle with 5 cm sound head

A. 1.0 MHz, 100% duty cycle with 5 cm sound head 1 MHz is used for deep structures (>3 cm), 3 MHz is used for more superficial structures. So, you can rule out B and D. If you want to heat, you want to no time off. So you would need a 100% duty cycle. A typical sound head is 5 cm. 2 cm sound heads are used on small areas like the fingers and toes. 3 MHz= 3 pulses/second 1 MHz = 1 pulse/second

The medical record indicates a patient has been diagnosed with chronic respiratory alkalosis. Which laboratory finding is the MOST consistent with this condition? A. Elevated arterial blood pH, low PaCO2 B. Low arterial blood pH, elevated PaCO2 C. Elevated arterial blood pH, elevated PaCO2 D. Low arterial blood pH, low PaCO2

A. Elevated arterial blood pH, low PaCO2 Elevated pH, low PaCO2 = respiratory alkalosis Elevated pH, elevated PaCO2 = partially compensated metabolic alkalosis Low pH, low PaCO2 = partially compensated metabolic acidosis Low pH, elevated PaCO2 = respiratory acidosis

A patient post modified radical mastectomy is referred for treatment of associated soft tissue restrictions and pain. During the examination, the physical therapist becomes concerned since the surgical site is extremely warm to touch, tender, and discolored, as shown in the image. Given the patient's recent history, which of the following conditions is the MOST likely cause of this complication? A. Dermatitis B. Cellulitis C. Mastitis D. Erysipelas

B. Cellulitis Cellulitis tends to develop in areas where the skin's protective barrier (e.g., surgical site, wound) and lymphatic flow (e.g., lymph node removal, excessive soft tissue, edema) have been disrupted. Cellulitis is a common post-op complication of surgical breast cancer treatment.

Patient is 8 weeks post-op of an Achilles tendon repair. Patient has recently started discontinuing the CAM boot and is unable to bear weight with complaints of pain with weight bearing. The PT should FIRST emphasize: A. Stretching of posterior compartment muscles in standing B. Heel lift C. Continue with CAM boot D. Notify the doctor

B. Heel lift You don't want to posterior muscles to be stretched yet so if you use a heel lift you can reduce the strain on the Achilles tendon and maintain some PF. The heel should be about 1-1.5 cm.

A patient who has a boggy end-feel with passive knee extension MOST likely has which of the following conditions? A. Arthritis B. Hemarthrosis C. Meniscal displacement D. Patellofemoral syndrome

B. Hemarthrosis A boggy end-feel is produced by fluid or blood in the joint.

A physical therapist is teaching a spinal cord injury patient to negotiate a 4 inch curb with a wheelchair. The MOST appropriate instruction that the therapist gives to the patient is: A. Hook the arms around push handle and descend backward B. Ascend backward with large wheels first C. Ascend in a wheelie position by lifting the front casters D. Place the front casters down first during descent

C. Ascend in a wheelie position by lifting the front casters

While in the acute care setting, PT reads an ECG strip of a 52-year-old patient. ECG strip displays a widened QRS segment, flattened P wave, and a peaked T wave. Which of the following would be indicated by your findings? A. Hypokalemia B. Hypercalemia C. Hyperkalemia D. Hypocalemia

C. Hyperkalemia Hyperkalemia is too much potassium in the blood

Two PTs perform a test on a patient using the Oswestry Disability Index. The patient visit is Monday (M) and Wednesday (W). One PT reports scores of 29 and 30 on M & W; other PT scored 36 and 37 (M & W). This is indicative of a problem in: A. Concurrent validity B. Intra-rater reliability C. Inter-rater reliability D. Construct validity

C. Inter-rater reliability

Which of the following methods is MOST appropriate for handling a 1-year-old child who has cerebral palsy and who exhibits strong extensor tone in the trunk and extremities? A. Carrying the child in sitting position B. Carrying the child over one's shoulder C. Keeping contact with the back of the child's head D. Picking the child up under the upper extremities

A. Carrying the child in sitting position The sitting position promotes visual attending, use of the upper extremities, and social interaction (Martin, pp. 98-100). A flexed posture is preferred so the shoulders are forward. A child who exhibits extensor posturing should be carried in a symmetric position that does not allow axial hyperextension and keeps the hips and knees flexed (Palisano).

A PT notices that a patient is experiencing early toe-off during terminal stance in gait. Which of the following identifies a likely cause AND an appropriate intervention to address that cause? A. Hip flexion contracture, prolonged stretch B. Hip adductor weakness, progressive strengthening C. Gastrocnemius weakness, ultrasound D. Great toe flexion weakness, progressive strengthening

A. Hip flexion contracture, prolonged stretch You can rule out B because you need to stay in your plane. Ultrasound is a poor intervention for gastrocnemius weakness so you can rule out C. The hip flexors are being stretched which they don't like so there is an early toe-off to relieve stretch on the hip flexors. Note: gastrocnemius tightness will affect midstance but hip flexion tightness will affect terminal stance

A patient suffered from a left hemisphere CVA a few weeks ago. The PT instructed the patient to pick up an object and the patient did not pick it up. The PT stepped away within view from the patient and noticed him picking up the object. Which of the following MOST LIKELY describes the scenario? A. Ideomotor apraxia B. Ideational apraxia C. Aphasia D. Ataxia

A. Ideomotor apraxia Ideomotor apraxia is the inability to carry out, on command, learned motor acts despite adequate motor and sensory abilities

A 69-year-old patient has symptoms of lightheadedness, along with numbness and tingling of arms and legs. His blood gas report is: pH- 7.48, PaCO2- 30 mm Hg, and HCO3- 26. Which of the following is the most INAPPROPRIATE treatment? A. Intravenous injection of sodium bicarbonate B. CO2 administration C. Pain control D. Using rebreathing mask/paper bag

A. Intravenous injection of sodium bicarbonate Based off of the values the patient demonstrates a high pH (7.35-7.45 is normal) which indicates he is in a state of alkalosis. His PaCO2 is also abnormal (35-45 mm Hg is normal) which means it is respiratory alkalosis. Since the pH and PaCO2 are both abnormal but the HCO3 is normal it tells you that the body is not trying to balance out PaCO2 and HCO3 to correct for the pH so this is uncompensated respiratory alkalosis.

A patient who has a lesion in the area indicated in the illustration is MOST likely to have an abnormal finding with which of the following examination procedures? A. Muscle testing B. Coordination testing C. Proprioceptive testing D. Tactile sensory testing

A. Muscle testing Muscle testing involves testing the strength of muscles. The area indicated in the illustration is the primary motor cortex, which initiates motor movement.

A 68-year-old patient suffering from Rheumatoid arthritis comes for a physical therapy evaluation. You observe a Boutonniere deformity. Which of the following characteristics the Boutonniere deformity? A. Ruptured central band B. Flexed DIP C. Hyperextended PIP D. Hyperflexed DIP

A. Ruptured central band Boutonniere Deformity: Rupture of the central band (central slip) of the extensor hood results in the lateral bands of the extensor apparatus (extensor hood) slipping in a volar direction to the PIP joint, causing PIP flexion and DIP extension.

A patient complains of right hip pain. The PT finds that the right ASIS is higher than the left ASIS, and right PSIS is lower than the left PSIS. While performing the long sitting special test, the right limb is shorter in supine and appears to get longer in sitting. Based on the examination, what is the MOST appropriate treatment? A. Stretch the right hip extensors B. Stretch the right hip flexors C. Strengthen the right hip flexors D. Strengthen the right hip extensors

A. Stretch the right hip extensors There is a posterior rotation of the right innominate. Always choose stretching over strengthening!!!

The PT is trying to improve closure of the right T4-5 facet joint. The mobilizing hand of the PT for PA mobilization techniques should be placed at: A. Transverse processes of right T5 B. Spinous processes of T5 C. Transverse processes of right T4 D. Transverse processes of left T5

A. Transverse processes of right T5 To improve closure you mobilize the bottom one so you can rule out C. The issue is on the right side so you can rule out D. That leaves you with A and B. If the issue is on the right side, mobilizing at the spinous process will not help you which rules out B.

A physical therapist reviews the results of a patient's pulmonary function test prior to examining a patient. The therapist notes that the patient's total lung capacity is significantly increased when compared to established norms. Which of the following medical conditions would MOST likely produce this type of result? A. Chronic bronchitis B. Emphysema C. Spinal cord injury D. Pulmonary fibrosis

B. Emphysema As a result of the pathologic changes to the lung tissue in emphysema, the lungs become hyperinflated. Due to the loss of elastic recoil, obstruction to airflow is seen as an increase in total lung capacity, residual volume, and functional residual capacity.

A PT is teaching a patient different strategies for breathing, one of which includes using tongue and pharyngeal muscles to force air into the lungs in an series of "gulps". Patients with which of the following diagnoses will benefit the MOST with this technique? A. Patients with T1 level SCI and TMJ dysfunction B. Patients with C4 level SCI C. Patients with level C7 level SCI D. Patients with T1 level injury and heartburn

B. Patients with C4 level SCI Injury at or above C3-C5 causes injury to the phrenic nerve which innervates the diaphragm. These patients will struggle with breathing. They will typically be on a ventilator.

According to role and slide mechanism, during right forearm pronation at the proximal radioulnar joint, the radial head will: A. Roll and slide posteriorly B. Roll anteriorly and slide posteriorly C. Roll and slide anteriorly D. Roll posteriorly and slide anteriorly

B. Roll anteriorly and slide posteriorly At the proximal radioulnar joint, convex rim of the radial head articulates with the concave radial notch on the ulna. So, according to concave-convex rule, roll and slide will occur in opposite direction. During pronation at the proximal radio-ulnar joint, radial head roll anteriorly and slides posteriorly.

A PT is treating a male patient in an acute care setting. Which of the following laboratory values should a physical therapist be concerned about in a patient taking anticoagulants? A. White blood cell count of 8,000 cells per microliter B. Red blood cell count of 5.5 million cells per microliter C. International normalized ratio of 5.5 D. Erythrocyte sedimentation rate of 15 mm/hr

C. International normalized ratio of 5.5 International normalized ratio is used to assess the adequacy and effectiveness of anticoagulant therapies, such as warfarin (Coumadin). A normal INR is 0.9-1.1 ("call '911' cause they're bleeding"). If they are on warfarin it may be 2-3.5 which would be fine. If a higher number is noted, then the patient is at risk for excessive bleeding. This can be dangerous and should be noted in individuals undergoing anticoagulation therapy.

A 30-year-old female presents with agonizing, nauseating, exhausting, tiring and unbearable pain. What pain pattern is MOST consistent with these symptoms? A. Vascular B. Neurogenic C. Musculoskeletal D. Emotional

D. Emotional The adjectives used to describe the patient's pain are subjective in nature, and suggest a variety of origins. Agonizing, nauseating, exhausting, tiring and unbearable are all the words a patient might use to describe pain in emotional terms.

A physical therapy student is studying about falls and objective measures to assess balance and mobility. Which of the following scores represents a higher risk of falls in the elderly? A. Score of 18 on Performance-Oriented Mobility Assessment B. Score of 18 seconds on Timed Up and Go Test C. Score of 49 on Berg Balance scale D. Score of 25 on Functional Gait Assessment

A. Score of 18 on Performance-Oriented Mobility Assessment 25-28: low risk 19-24: medium risk <19: high risk

A fireman was brought to the ER with burns on his anterior chest and trunk, anterior right arm, and bilateral anterior legs. Which of the following accurately describes the area of distribution of the burn? A. 40.5% B. 20.5% C. 54% D. 36%

A. 40.5% According to the rule of nines: anterior chest and trunk = 18%, anterior right arm = 4.5%, anterior leg = 9%. So... 18 + 4.5 + 9 + 9 = 40.5%

A 22-year-old patient is recovering from a complete spinal cord injury at the level of L4. Functional expectations for this patient include which of the following? A. Ambulation using bilateral AFOs and canes B. Ambulation using bilateral KAFOs and a reciprocating walker C. Ambulation using bilateral KAFOs, crutches and a swing-through gait D. Wheelchair locomotion using an active duty light weight chair

A. Ambulation using bilateral AFOs and canes A complete spinal cord injury means there is no motor or sensory below the level of injury. Since the level of injury is L4 that means you will not have toe extension or PF but everything else will be fine. Therefore, you only need support at the foot and ankle.

A PT reviewing ECG strip of a patient in a rehabilitation hospital. Which of the following is the MOST appropriate interpretation of this ECG strip? A. Atrial fibrillation B. Atrial flutter C. First degree heart block D. Third degree heart block

A. Atrial fibrillation This image is clearly an issue with the atria so you can rule out C and D. The atria are contracting super fast and unorthodox so it is not an atrial flutter. Atrial flutter would be slower with about 4-5 beats of the atria before the ventricle. R intervals are also equally spaced with an atrial flutter but NOT with atrial fibrillation. First degree block: P-R interval is >0.2 seconds (1 large box) Third degree block: atrial rate and ventricular rate are independent of each other

A patient four weeks post ACL reconstruction questions the PT as to why they are still partial weight bearing. Which of the following impairments would be the MOST likely rationale for the patient's weight bearing status? A. The patient lacks full active knee extension B. The patient has good (4/5) quad strength C. The patient has fair (3/5) hamstrings strength D. The patient has diminished superficial cutaneous sensation

A. The patient lacks full active knee extension A patient s/p ACL reconstruction surgery may continue to use an assistive device for weight bearing if they do not possess full active knee extension. Ambulation on a flexed knee can result in excessive irritation of the patellofemoral joint.

A PT is examining a 46-year-old patient who underwent a radical mastectomy. The patient has developed lymphedema on the right UE. Which of the following statements is the CORRECT intervention for this patient? A. The trunk and axilla should be decongested first followed by the arm and hand B. Manual lymphatic drainage should involve proximal to distal stroking C. The hand should be decongested first followed by trunk and axilla D. Manual lymphatic drainage is contraindicated in post mastectomy

A. The trunk and axilla should be decongested first followed by the arm and hand You decongest the PROXIMAL portion first to make room for fluid from the distal area. Then you stroke DISTAL TO PROXIMAL towards the lymph nodes.

A physical therapist reviews the results of pulmonary function testing on a patient who has emphysema. Assuming the patient's testing was classified as unremarkable, which of the following lung volumes would MOST likely approximate 10% of the patient's total lung capacity? A. Tidal volume B. Inspiratory reserve volume C. Residual volume D. Functional residual capacity

A. Tidal volume The average tidal volume in a healthy adult is 500 mL. Total lung capacity is usually around 5,000 mL. Thus, normal tidal volume is approximately 10% of total lung capacity.

A physical therapist administers a submaximal exercise test to a patient in a cardiac rehabilitation program. The protocol requires the patient to ride a cycle ergometer for a predetermined amount of time using progressive workloads. In order to predict the patient's maximum oxygen uptake, it is necessary to determine the relationship between which of the following parameters? A. Heart rate and rate of perceived exertion B. Heart rate and workload C. Blood pressure and rate of perceived exertion D. Blood pressure and workload

B. Heart rate and workload The physical therapist can use the heart rate response to one or more submaximal workloads to predict maximum oxygen uptake.

A physical therapist is evaluating a patient who has a history of CAD. Currently, at rest, the patient presents with a BP of 135/75 and HR of 85 bpm. Also, noted was a RR of 19 and SpO2 of 95%. After 15 minutes of light exercise the physical therapist noted that the BP was 145/74, HR 86 bpm, RR 23, and SpO2 92%. Which medication would MOST influence these post exercise values? A. Nitrostat B. Propanolol C. Lasix D. Lidocaine

B. Propanolol Propanolol is a beta-blocker that slows the heart. The heart stayed relatively the same because of the beta blocker. Nitrostat- used for angina Lasix- antidiuretic used to treat edema (dehydration) Lidocaine- local anesthetic

Which of the following signs would be MOST indicative of a patient who is experiencing chronic lower limb ischemia? A. There is an increase in the hair growth in the lower extremities B. The skin of the lower extremities has become transparent and appears dehydrated C. The nail beds of the toes have become thin and supple in texture and strength D. There is an increase in skin temperature in the lower extremity

B. The skin of the lower extremities has become transparent and appears dehydrated With chronic ischemia due to arterial insufficiency, a cardinal sign is that the skin in the lower extremities becomes thin, scaly or shiny, and transparent due to inadequate blood flow.

A physical therapist reviews the medical record of a patient with a suspected head injury. During testing using the Glasgow Coma Scale, the patient exhibited spontaneous eye opening, was able to follow selected motor commands, and was considered to be "oriented" based on verbal responses. What score should the therapist assign to this patient? A. 6 B. 12 C. 15 D. 18

C. 15 The Glasgow Coma Scale is a scale ranging from 3-15 with a higher score representing greater level of consciousness. You can automatically rule out D because it is not a possible score. For this patient: Spontaneous eye opening = 4 Oriented to verbal responses = 5 Obeys motor commands: 6 6 + 5 + 4 = 15 < 8: severe head injury 9-12: moderate head injury 13-15: mild head injury

A physical therapist performs autolytic debridement in an attempt to remove nonviable tissue from a stage 4 pressure injury. Autolytic debridement removes necrotic tissue from the wound by using which of the following methods? A. A sharp instrument B. An externally applied force C. The body's own mechanisms D. A commercially prepared enzyme

C. The body's own mechanisms Autolytic debridement refers to using the body's own mechanisms to remove nonviable tissue. Common methods of autolytic debridement include transparent films, hydrocolloids, hydrogels, and alginates.

A pediatric PT is evaluating a 2 month old child. The PT pulls the child from a supine to seated position using the child's forearms. The child extends the trunk in response to this stimuli. Which of the following statements is the MOST accurate? A. The child has a normal traction reflex B. The child has a normal moro reflex C. The child has an abnormal moro reflex D. The child has an abnormal traction reflex

D. The child has an abnormal traction reflex This question describes the traction reflex. The traction reflex is tested by pulling a child from supine to sit. A normal response is to fully flex and grasp. An extensor response would be considered abnormal.

During a cardiac rehabilitation session, PT notices after 5 minutes that the ECG strip is displaying a PR interval of 0.28. The QRS segment is slightly widened. When PT asks the patient, he says he is fine and can keep on going. Which of the following would be indicative of your findings? A. 1st AV block B. 2nd AV block C. 3rd AV block D. 2nd AV block, Mobitz I

A. 1st AV block A 1st degree AV block has a PR interval of more than 0.2 but still shows no abnormality in the QRS segment

While assessing a patient with a R CVA, the therapist asks the patient to fold a piece of paper into half and notices that the patient does not use the L hand at all. On asking the patient what is wrong with the L hand, she says that she slept funny so the hand is not moving for now and that it should be moving by the end of the day. Which of the following is the MOST LIKELY medical diagnosis for this presentation? A. Somatoagnosia B. Right-left discrimination disorder C. Ideational apraxia D. Anosognosia

D. Anosognosia Anosognosia is is the denial or lack of awareness of the presence or severity of one's paralysis.

A low back pain patient's Oswestry Disability Questionnaire score was 18 points. Four weeks later, the score was 50 points. Which of the following is the BEST option for the physical therapist? A. Continue PT until the pt returns to a score of 60 B. Discharge the pt as there is an improvement C. Document the improvement and start HEP D. Ask the pt to have a physician consult

D. Ask the pt to have a physician consult The higher the score, the more disability they have. So this patient has worsened over the last 4 weeks.

A physical therapist treats a patient who has end-stage renal disease for general deconditioning. Which of the following signs and symptoms would this patient MOST likely demonstrate? A. Increased urine output and polycythemia B. Increased urine output and anemia C. Decreased urine output and polycythemia D. Decreased urine output and anemia

D. Decreased urine output and anemia With renal failure, the kidneys have a decreased ability to adequately filter fluids, therefore, urine output is significantly decreased or absent. Due to the kidneys' role in the production of erythropoietin (hormone produced by the kidneys that plays a role in the production of RBCs), patients with ESRD often display signs and symptoms of anemia.

A patient fails to attain established physical therapy goals within the number of visits initially set by the physical therapist. The patient has made substantial progress in therapy, however has plateaued over the last 4 sessions. The most appropriate action for the physical therapist is? A. Request additional visits from the referring physician B. Document a progress note that updates the patient's current status C. Talk to the patient regarding their compliance D. Discharge the patient from physical therapy with a home exercise program

D. Discharge the patient form physical therapy with a home exercise program With the patient's status plateauing, the therapist can no longer document that the patient's treatment is medically necessary. For this reason, it is best to discharge the patient with a home exercise program.

A patient's severe knee sprain resulted from medial rotation of the femur on the tibia with simultaneous application of a valgus force while the foot was placed on the ground. Which structures are MOST likely to be involved? A. Medial collateral ligament, medial meniscus, anterior cruciate ligament B. Medial collateral ligament, lateral collateral ligament, posterior cruciate ligament C. Lateral collateral ligament, medial meniscus, anterior cruciate ligament D. Medial collateral ligament, lateral meniscus, posterior cruciate ligament

A. Medical collateral ligament, medial meniscus, anterior cruciate ligament This is considered the "unhappy triad"

A 12-year-old girl has a structural right thoracic idiopathic scoliosis. The clinical features you would expect to find include a high: A. Right shoulder, a prominent right scapula, and a left hip that protrudes B. Left shoulder, a prominent left scapula, and a right hip that protrudes C. Right shoulder, a prominent left scapula, and a right hip that protrudes D. Left shoulder, a prominent right scapula, and a left hip that protrudes

A. Right shoulder, a prominent right scapula, and a left hip that protrudes A right scoliosis means the convexity is on the RIGHT. You would see same side shoulder elevation, same side scapula prominence, and opposite side hip more prominent.

A 44 y/o female pt arrived to a clinic after sustaining a fall involving a hand/wrist injury. Upon examination, the PT observed the radius dislocated in a volar direction. Which of the following conditions would MOST likely match this description? A. Smith's Fx B. Colle's Fx C. Scaphoid Fx D. Dinner fork deformity

A. Smith's Fx Colle's Fx- the radius goes in the DORSAL direction Dinner fork deformity- this is another name for Colle's Fx

A 50-year-old male patient presents to an outpatient clinic with complaints of medial knee pain. The picture (left) represents the position of the foot during normal quiet standing. What is the MOST appropriate diagnosis of this condition? A. Uncompensated forefoot valgus B. Compensated forefoot valgus C. Uncompensated forefoot varus D. Compensated forefoot varus

A. Uncompensated forefoot valgus You can immediately tell the foot is going into pronation which is forefoot valgus so you can rule out C and D. Valgus at the knee leads to valgus at the forefoot! If the HEEL IS NEUTRAL it is not helping to compensate for the forefoot valgus and it is UNCOMPENSATED!

With inadequate dorsiflexion at the ankle, it is assumed that the client will not utilize an ankle rocker appropriately. What is one of the MOST common compensations for inadequate ankle rocker? A. Premature heel strike at initial contact B. Excessive knee flexion in swing C. Excessive trunk rotation at midstance D. Backward trunk lean at midstance

B. Excessive knee flexion in swing Excessive hip and knee flexion during swing is often caused by lack of ankle dorsiflexion of the swing limb. It happens to clear the toes of the swing limb.

Which of the following clinical manifestations would MOST likely be associated with right ventricular failure? A. Pulmonary edema B. Jugular venous distention C. Paroxysmal nocturnal dyspnea D. Muscular weakness and fatigue

B. Jugular venous distention In patients who have right ventricular failure, the right side of the heart is unable to adequately pump fluid through the pulmonic valve. This fluid backs up into the jugular vein through the superior vena cava.

A 55-year-old male patient diagnosed with Cushing's disease is referred to physical therapy. Which of the following signs and symptoms is NOT consistent with this disease? A. Distension of the abdomen B. Susceptibility to bruising C. Adrenal hypoplasia D. Cardiac hypertrophy

C. Adrenal hypoplasia Adrenal HYPERplasia is characteristic of Cushing's disease "Cushion's disease"- everything is big and soft

A patient has a superficial partial-thickness wound resulting from an abrasion. The wound bed is red and moist and with minimal exudate. Which of the following wound dressings is MOST appropriate to use? A. Wet to dry gauze B. Foams C. Calcium alginate D. Transparent films

D. Transparent films Foams are used for moderate exudate. Calcium alginate is used for heavy exudate. Transparent films are used for very mild exudate. It is also red and moist meaning it is healing well.

A PT treats a patient diagnosed with Parkinson's disease. When working on controlled mobility, which of the following would best describe the physical therapist's objective? A. Facilitate postural muscle control B. Promote weight shifting and rotation trunk control C. Emphasize reciprocal extremity movement D. Facilitate tone and rigidity

B. Promote weight shifting and rotation trunk control

A physical therapist is testing the muscle strength of a patient with complaints of shoulder pain. The patient is positioned prone with head turned to one side, arms at sides with test arm internally rotated. The patient is instructed to raise his arm off the table. If the patient is having difficulty completing the test, which of the following muscles is MOST likely responsible for this impairment? A. Infraspinatus B. Rhomboids major C. Trapezius D. Latissimus dorsi

D. Latissimus dorsi Latissimus dorsi is a shoulder extensor, adductor and internal rotator. To test the muscle, the patient is prone with head turned to one side, arms at sides; test arm is internally rotated and the patient is instructed to raise arm off table.

A patient is demonstrating an upper extremity flexion synergy following a CVA. Which of the following is MOST likely associated with this type of synergy? A. Scapular retraction B. Elbow extension C. Wrist extension D. Forearm pronation

A. Scapular retraction Upper extremity flexion synergy components are scapular retraction/elevation, shoulder abduction and external rotation, elbow flexion, forearm supination, wrist and finger flexion. B & D are components of extension synergy.

A physical therapist is evaluating the sensitivity of a balance test in a group of older adults. Which of the following results indicates a true positive finding? A. Patients with a history of falls test negatively B. Patients with a history of falls test positively C. Patients with no history of falls test negatively D. Patients with no history of falls test positively

B. Patients with a history of falls test positively A true positive finding requires POSITIVE results so you can rule out A and C. Decide if it is a true or a false story and then match the last word of the question with the last word of the answer. A- false negative (type II) B- true positive C- true negative D- false positive (type I)

A physical therapist while assessing a patient noticed that the patient winced in pain when the arm was in the position as shown in the picture below. Which of the following is the MOST LIKELY cause of this presentation? A. Ulnar nerve entrapment B. Anterior subluxation of the shoulder C. Impingement at the shoulder D. AC joint instability

C. Impingement at the shoulder The therapist is performing the Hawkins-Kennedy Test by forcibly medially rotating the humerus with the arm flexed to 90 degrees. It is considered to be positive if the test causes pain and is indicative of impingement at the shoulder joint.

During exams week, the physical therapist observes the posture of a 29-year-old female patient. The PT notices that she has a forward head posture. Which of the following characterizes forward head posture? A. Extension of lower cervical spine B. Flexion of upper cervical spine C. Increase in the resistance moment arm of the head D. Decrease in the resistance moment of the head

C. Increase in the resistance moment arm of the head With a forward head posture, the patient has his lower cervical spine flexed along with upper cervical extension. This causes the head to translate anteriorly and the load of the weight of the head (load arm) on the long cervical extensors increases resulting in an increase in the external moment arm (resistance arm). (The external moment arm is the perpendicular distance between the axis of rotation and the external force.)

A patient with a rotator cuff tear is in rehab for conservative management. The therapist will use biofeedback on the supraspinatus and would like to strengthen the muscles. What would be the MOST appropriate protocol? A. Move electrodes far apart and decrease sensitivity B. Move electrodes close and decrease sensitivity C. Move electrodes far apart and increase sensitivity D. Move electrodes close and increase sensitivity

C. Move electrodes far apart and increase sensitivity When you are trying to strengthen a muscle you want to move the electrodes farther apart so it covers more area and you want increased sensitivity so that any contraction that the weak muscle is doing is caught on the computer screen for the patient to see.

A patient recovering from a L3 ASIA D spinal cord injury reports difficulty going down ramps with unsteady, wobbly knees. Which of the following would be the MOST appropriate intervention for this problem? A. Ultrasound therapy to reduce hamstring contracture B. Icing to decrease knee extensor spasticity C. Progressive resistance training for the quadriceps D. Stretching the plantar flexors using a posterior resting splint

C. Progressive resistance training for the quadriceps ASIA D: more than 1/2 of key muscles have a grade of 3 or more L3 myotome: knee extension The quadriceps help eccentrically control the knee as they are going down the ramp. They have a motor component in the quads so you can strengthen them.

Members of a community health task evaluate a proposal for a new adolescent screening program. Several members of the task force raise questions as to the validity of the screening instrument. Which measure of validity examines the instrument's ability to identify individuals with a disease by comparing true positives? A. Adaptability B. Selectivity C. Sensitivity D. Specificity

C. Sensitivity Sensitivity is a measure of the validity of a screening test, based on the probability that the screening test will be positive in someone with the disease or target condition (i.e., true positive)

A PT is using a new special screening test to help determine if a tennis player has a rotator cuff tear among other possibilities. The test has a VERY high sensitivity of 0.93 (or 93%). If the results of the test are negative the PT should: A. Try another special test for rotator cuff tear B. Consider the test results inconclusive C. Rule in rotator cuff tear D. Rule out rotator cuff tear

D. Rule out rotator cuff tear SnOut: if it is negative you rule them out

A PT is performing an assessment on a patient. The patient presented with hips and knees hyperextended, with the pelvis in neutral but shift anteriorly, elongated neck flexors, and favored leg appearing longer in standing. The patient MOST likely has which type of deformity? A. Round back form of kyphosis B. Flat back form of kyphosis C. Gibbus deformity D. Swayback

D. Swayback

A patient who is four months post surgery to repair a torn biceps tendon still lacks 40 degrees of elbow extension. Because conservative efforts have failed, the physician orders serial casting to improve the patient's mobility. After one round of casting, what would be the MOST likely expected increase in range of motion? A. 5 degrees B. 15 degrees C. 25 degrees D. 35 degrees

A. 5 degrees Serial casting is a casting technique that is used to improve ROM at a joint that has developed a contracture. The procedure consists of placing the joint in a submaximal position and then applying a cast. After wearing the cast for several days, it is removed. With each round of casting, the joint should make modest ROM gains (5-7 degrees). Serial casting can last for a few weeks or several months depending on the extent of the contracture and the relative success of the intervention.

A physical therapist prepares to assess the blink reflex in a patient with suspected neurological involvement. Which cranial nerve components are assessed with this reflex? A. Afferent cranial nerve V; efferent cranial nerve VII B. Afferent cranial nerve VII; efferent cranial nerve V C. Afferent cranial nerve IX; efferent cranial nerve X D. Afferent cranial nerve X; efferent cranial nerve IX

A. Afferent cranial nerve V; efferent cranial nerve VII The afferent cranial nerve V (trigeminal) is associated with face sensation. The efferent cranial nerve VII (facial) is responsible for closing the eyes.

A 54-year-old patient reports to an outpatient clinic with history of insidious onset of pain and paresthesias in the lateral forearm with cervical extension. Which of the following is the MOST likely diagnosis? A. C6 radiculopathy B. C8 radiculopathy C. Ulnar nerve entrapment D. Radial nerve entrapment

A. C6 radiculopathy C6- lateral forearm C8- medial forearm Nerve entrapments are not affected by cervical extension!

When performing range of motion exercises with a patient who sustained a head injury, a physical therapist determines that the patient lacks full elbow extension and classifies the end-feel as hard. The presence of which of the following findings is the MOST likely cause? A. Heterotopic ossification B. Spasticity of the biceps C. Anterior capsular tightness D. Triceps weakness

A. Heterotopic ossification Heterotopic ossification is abnormal bone growth in tissue. Individuals with head injuries have a high chance of developing heterotopic ossification. Bone will always give you a hard end-feel. B- this would be a firm end feel C- this would also be a firm end feel D- end feel is a passive assessment so it does not matter if the patient is weak

Which of the following findings is MOST important to assess in a patient who is 2 days post radical mastectomy (radical mastectomy is removal of one or two breasts, chest muscles and lymph nodes to treat breast cancer)? A. Limited ROM in the involved upper extremity B. Atrophy in the involved upper extremity C. Chest wall adhesions D. Contractures of the elbow flexors

A. Limited ROM in the involved upper extremity After mastectomy, it is common that the patients experience temporary and sometimes long-term loss of shoulder mobility. It can be due to incisional pain, muscle guarding or any other factors. Chest wall adhesions can also develop after surgery, but it cannot be seen after two days of surgery.

A 20-year-old patient in ROTC complains of anterior hip pain. During treatment, the patient starts flirting with the male PT. The PT acts professionally but the patient persists and makes the PT uncomfortable. What would be the MOST appropriate response? A. Refuse to treat the patient and refer to another male PT B. Ignore the flirting and continue to treat C. Notify patient's ROTC supervisor D. Joke along to keep open the possibility of dating in the future

A. Refuse to treat the patient and refer to another male PT PT has full rights to refuse treatment. Referring to another male PT does not mean she will flirt with him too. Notifying the ROTC supervisor is a HIPAA violation.

A physical therapist treats a patient status post stroke. Which of the following actions would be the MOST likely to facilitate elbow extension in a patient who has hemiplegia? A. Turn the head to the affected side B. Turn the head to the unaffected side C. Extend the lower extremities D. Flex the lower extremities

A. Turn the head to the affected side Patients status post CVA are likely to exhibit abnormal tonic reflexes. The asymmetrical tonic neck reflex produces extension of the affected UE when the patient's head is turned toward the affected side. The UE on the skull side will flex.

A physical therapist is examining a 41-year-old male patient in an outpatient clinic. Which of the following sounds is the physical therapist auscultating? A. Vesicular sounds B. Mitral valve C. Pulmonary valve D. Tracheal sounds

A. Vesicular sounds These are located over most of the lungs with a soft intensity and low pitch. Inspiratory sounds are longer than expiratory at this location.

A 45-year-old patient has been walking 3 days/week for 20 minutes for the past 3 weeks. When progressing the exercise program, which of the following modifications will MOST likely accomplish the weight-loss goal? A. Walk 4 days/week at current walking speed and increase duration to 45 minutes B. Increase the walking speed and keep the duration at 20 minutes C. Walk 6 days/week and decrease the duration to 10 minutes D. Decrease the walking speed and increase the duration to 25 minutes

A. Walk 4 days/week at current walking speed and increase duration to 45 minutes For weight loss: 3-5 days/week, 40-60% of HRmax, 45-60 minutes of moderate exercise

A physical therapist treats a patient who has rheumatoid arthritis. During the patient interview, the patient indicates that they are diabetic. Which type of pharmacological agent would be the LEAST likely to be used to treat rheumatoid arthritis given the stated comorbidity? A. Nonopioid analgesic agents B. Corticosteroid agents C. Biologic response modifiers D. Disease-modifying antirheumatic agents

B. Corticosteroid agents Corticosteroids (e.g., prednisone, prednisolone) are the most powerful class of anti-inflammatory agents available; however, side effects of short and long-term use can be serious to life-threatening. Patients with diabetes mellitus may be unable to use corticosteroids since a side effect of use is the elevation of blood sugar.

A patient diagnosed with multiple sclerosis uses extensor tone to assist them to successfully complete a sit to stand transfer. Which pharmacological agent would MOST limit the patient's ability to complete the transfer? A. Calcium carbonate B. Dantrolene sodium C. Levodopa D. Secobarbital

B. Dantrolene sodium Dantrolene sodium is used to treat spasticity by interfering with the release of calcium. Reducing the patient's spasticity may significantly impact their ability to complete the transfer since it will likely reduce the patient's extensor tone which is presently being relied upon to complete the transfer.

A PT is reviewing the lab values of a 58-year-old male patient. Which of the following about glucose monitoring is LEAST appropriate? A. Hypoglycemia is documented for fasting blood glucose value of 40 mg/dL B. Diabetes mellitus is when fasting blood glucose levels are between 80 and 110 mg/dL measured on two separate days C. Glycosylated hemoglobin normal reference range is 4-6% D. A1C level above 10% requires immediate insulin therapy

B. Diabetes mellitus is when fasting blood glucose levels are between 80 and 110 mg/dL measured on two separate days Normal A1C: <5.7% Pre-diabetic A1C: 5.7-6.4% Diabetic A1C: 6.5% or greater

Which of the following positions of the humerus is BEST for application of an ultrasound treatment to the supraspinatus tendon insertion? A. Flexion and lateral (external) rotation B. Extension and medial (internal) rotation C. Abduction and lateral (external) rotation D. Flexion and medial (internal) rotation

B. Extension and medial (internal) rotation Extension and medial (internal) rotation of the shoulder puts the supraspinatus tendon in the most accessible position.

A 58-year-old patient has chronic obstructive pulmonary disease. Which of these pulmonary test results will NOT be increased when compared with those of a 58-year-old healthy individual? A. Total lung capacity B. FEV1/FVC ratio C. Residual volume D. Functional residual capacity

B. FEV1/FVC ratio Total lung capacity increases because residual volume increases. When residual volume increases, functional residual capacity also increases.

Which of the following strategies is optimal for BEST positioning for a patient with a CVA when lying on the hemiplegic side? A. Scapula retracted B. Forearm supinated C. Hip flexed and knee extended D. Fingers flexed and thumb adducted

B. Forearm supinated Effective positioning of the hemiparetic extremities encourages proper joint alignment while positioning the limbs out of the abnormal postures typically assumed. When lying on the more affected side, forearm should be should be kept in supination.

A PT is treating a pediatric patient with cerebral palsy. The patient is seen in standing with a toe-in posture. Which of the following postural strategies most accurately correlates with the observed foot position? A. Internal tibial torsion, increased femoral retroversion B. Internal tibial torsion, increased femoral anteversion C. External tibial torsion, increased femoral retroversion D. External tibial torsion, increased femoral anteversion

B. Internal tibial torsion, increased femoral anteversion Toe-in is related to femoral anteversion so you can automatically rule out A and C. When you have a toe-in posture your foot wants to pull your tibia in which leads to an internal tibial torsion.

Scheuermann disease, commonly seen in young gymnasts, is also known as: A. Adolescent scoliosis B. Juvenile kyphosis C. Jumper's disease D. Bamboo spine lordosis

B. Juvenile kyphosis Scheuermann disease is also known as juvenile kyphosis. In this condition, inflammation of the bone and cartilage occurs around the ring epiphysis of the vertebral body. The condition often leads to an anterior wedging of the vertebra.

Examination of a patient's right lower extremity reveals weakness in great toe extension and decreased sensation along the lateral leg and dorsum of the foot. Which of the following nerve roots is MOST likely contributing to these findings? A. L4 B. L5 C. S1 D. S2

B. L5 L4: dorsiflexion L5: great toe extension S1: plantarflexion S2: knee flexion

A 6-year-old male child is referred to PT. The mother is concerned that the child is using his hands to climb up his legs in order to stand up. Which of the following conditions should the PT suspect? A. Cerebral palsy B. Muscular dystrophy C. Spina bifida D. Down syndrome

B. Muscular dystrophy This is an example of a Gower's sign which is common in Duchenne muscular dystrophy. It indicates weakness of the proximal muscles. DMD is a genetic (recessive) condition in MALES which is also hinted at in the question. It is characterized by progressive muscular weakness and the life expectancy is in their teens.

Upon physical therapy examination of the upper extremity, the physical therapist diagnoses an intact medial collateral ligament of the elbow. The PRIMARY function of the medial collateral ligament at the elbow is that it: A. Compresses the medial side B. Resists valgus torque C. Resists varus torque D. Compresses the lateral side

B. Resists valgus torque The fan-shaped medial collateral ligament is the most important ligament in the elbow for providing stability against valgus stress, particularly in the range of 20-130 degrees of flexion and extension.

A 63-year-old patient suffered a CVA 3 months ago. During ascending stairs, the patient is able to keep the involved foot on the step above but is unable to transfer the weight to the next stair level due to weak quadriceps. The BEST intervention to solve the problem is: A. Bridging, holding B. Standing, partial wall squats C. Supine, knee flexion with hip extension D. Standing, side steps

B. Standing, partial wall squats Stick to your plane! D does not stick to the plane. You also want an exercise that in a similar position to your goal movement so you would want to be in a standing position.

A physical therapist works with a patient who has been on bed rest for one month. Which of the following muscle groups should the therapist anticipate being the MOST affected by the prolonged immobilization? A. Extensor muscles of the upper extremities B. Flexor muscles of the upper extremities C. Extensor muscles of the lower extremities D. Flexor muscles of the lower extremities

C. Extensor muscles of the lower extremities The lower extremities tend to be more affected by immobilization than the upper extremities. Extensor muscles are impacted more than the flexor muscles.

A 7-year-old complains of a vague ache in the groin that radiates to the medial thigh and inner aspect of the knee. On examination, the PT notices decreased abduction and internal rotation. What is the MOST appropriate intervention for this child? A. Improving the strength of abductor muscles using closed chain exercises B. Improving flexibility of abductor and external rotators C. Improving the containment of the femoral head in acetabulum using the brace D. Improving the flexibility of adductor and internal rotators

C. Improving the containment of the femoral head in acetabulum using the brace The condition described could be either Legg Calves Perthes or slipped capital femoral epiphyses based off of decreased mobility. However, the age is more representative of Legg Calves Perthes. Legg Calves Perthes is a flattening of the femoral head. SCFE: Mr. Fab (limitations in medial rotation, flexion, and abduction) Legg Calves Perthes: limitations in medial rotation, EXTENSION, and abduction

A 60-year-old male is undergoing cardiac rehabilitation post MI in the hospital. On day three, the patient has progressed to walking. Which of the following is the MOST appropriate way to document this activity? A. Patient ambulated for 20 minutes with RPE 10/20 B. Patient ambulated 300 ft with RPE 4/20 C. Patient ambulated 500 ft in 6 minutes with RPE 10/20 D. Patient ambulated in the ICU hallway with RPE 10/20

C. Patient ambulated 500 ft in 6 minutes with RPE 10/20 The RPE scale is 6-20 so you can automatically rule out B. Proper documentation will include distance, time, and RPE.

While evaluating a 30-year-old male with shoulder pain, the therapist applies pressure at the end range of shoulder abduction and external rotation. The patient feels a sudden PARALYZING pain and WEAKNESS in his shoulder. This finding is MOST LIKELY an indication of: A. Thoracic outlet syndrome B. Myotome involvement C. Anterior instability D. Cervical spondylosis

C. Anterior instability The therapist is performing anterior apprehension test or crank test. This test is primarily designed to check for traumatic instability problems causing gross or anatomical instability of the shoulder; positive anterior apprehension test indicates anterior instability.

A patient with several motor and sensory abnormalities exhibits signs of autonomic nervous system dysfunction. Which of the following physiological responses is a parasympathetic response, rather than an indicator of increased sympathetic involvement? A. Anxiety, distractibility B. Mottled, cold, shiny skin C. Constriction of the pupils D. Rapid, shallow breathing

C. Constriction of the pupils Constriction of the pupils is characteristic of a parasympathetic response. The parasympathetic division will also decrease HR, stimulate digestion, constrict the lungs, and stimulate other internal organs.

The presence of ecchymosis is noted in which of the following conditions? A. Decreased RBC B. Increased platelet count C. Decreased platelet count D. Increased hemoglobin count

C. Decreased platelet count Low platelet count can lead to bruising, spontaneous bleeding under the skin leading to ecchymosis. Increase hemoglobin causes polycythemia. Decrease RBC causes anemia causing increased fatigue and reduced exercise tolerance.

A PT consult is ordered for a 50-year-old gentleman in the late stages of ALS. In the patient's chart is an electromyography report and nerve conduction velocity test. All of the following findings are consistent with his diagnosis EXCEPT: A. Decreased amplitude of motor unit action potential B. Decreased duration of motor unit action potential C. Decreased sensory evoked potential D. Decreased polyphasic action potential

C. Decreased sensory evoked potential Sensory evoked potential will not be seen in ALS as ALS is a motor neuron disease affecting the motor nerves and not sensory nerves. A, B and D are findings consistent with ALS.

A physical therapist reads in a patient's medical chart that the patient has been prescribed albuterol. Which of the following conditions would MOST likely require the use of this medication? A. Breast cancer B. Angina pectoris C. Exercise-induced asthma D. Spinal cord injury

C. Exercise-induced asthma Albuterol is a medication that is commonly prescribed for conditions that result from bronchoconstriction, such as asthma or COPD.

Active insufficiency of the right iliopsoas will be caused by? A. Hip extension and right lateral flexion of the trunk B. Hip extension and left lateral flexion of the trunk C. Hip flexion and right lateral flexion of the trunk D. Hip flexion and left lateral flexion of the trunk

C. Hip flexion and right lateral flexion of the trunk Active insufficiency is when the muscle has shortened as much as it can. Since the iliopsoas is a hip flexor, you know that it shortens during hip flexion. Due to this you can rule out A and B. The iliopsoas also laterally flexes to the same side so that rules out D.

A 50-year-old patient complains of restriction of shoulder movements during daily overhead activities. The patient has a history of myasthenia gravis. The PT notes hyperactive reflexes. The patient has been taking anti-inflammatory drugs for the last four weeks but has had no resolution of symptoms. Which condition is MOST associated with the above findings? A. Hypothyroidism B. Biceps tendinitis C. Hyperthyroidism D. Supraspinatus tendinitis

C. Hyperthyroidism If anti-inflammatory drugs have not helped that tells me it is NOT a tendinitis issue so you can rule out B and D. You would also not see any effect on reflexes with musculoskeletal issues like tendinitis. HYPERthyroidism is correlated with HYPERactive reflexes.

A posterior approach was used for a case of revised total hip arthroplasty. Which of the following should NOT be included while educating the patient in order to prevent posterior dislocation? A. Transfer to the sound side from chair to bed B. When descending stairs, lead with the operated leg. C. Keep the knees higher than the hips when sitting D. Avoid standing activities that involve rotating the body towards the operated extremity

C. Keep the knees higher than the hips when sitting You do not want to flex the hips to greater than 90 degrees following a posterior approach THA!!! FADIR is avoided with the posterior approach!!! (flexion, adduction, IR)

A 25-year-old patient underwent a medial meniscectomy. The POC includes exercising the quadriceps femoris against accommodating resistance. Which of the following exercises is MOST appropriate? A. Passive knee extension range of motion by the PT B. Concentric knee extension with a 10 lbs ankle weight C. Knee extension on an isokinetic exercise device D. Eccentric knee extension with a 5 lbs ankle weight

C. Knee extension on an isokinetic exercise device The only way to control the resistance through a certain speed is through isokinetic exercise. If isokinetic exercise is not provided the next most important would be CONCENTRIC exercise.

A physical therapist is treating a patient who complains of a vague ache in the lower extremities. As shown in the picture below, what is the MOST likely structural malalignment present? A. Lateral tibial torsion with lateral rotation of femur B. Medial tibial torsion with lateral rotation of femur C. Lateral tibial torsion with medial rotation of femur D. Medial tibial torsion with medial rotation of femur

C. Lateral tibial torsion with medial rotation of femur This is an image of a child W-sitting. For this the feet are going outward which means there is a LATERAL torsion of the tibia.

A patient is getting treated for pelvic pain and is accompanied by his wife to the PT session. While treating, the patient expressed dissatisfaction in his marital life and says he has regular suicidal thoughts. What would be the BEST PT action? A. Ask the patient to seek an appointment with a mental health practitioner B. Disagree with the patient and say positive things about living life C. Refer the patient immediately to a mental health practitioner and stay there until help arrives D. Inform the patient's wife that this problem is outside the scope of physical therapy practice

C. Refer the patient immediately to a mental health practitioner and stay there until help arrives Patients often do not seek an appointment with a mental health practitioner when you suggest it. To ensure the safety of your patient you need to wait until the mental health practitioner arrives. If you are busy with other patients have another LICENSED professional watch them until the practitioner comes.

A PT is consulted to evaluate a patient in an acute care hospital. The patient has symptoms of muscle aches, cramps, soreness, and weakness in a proximal to distal pattern in bilateral lower extremities. The patient mentions that his urine has been tea colored recently. The patient's lab report from the previous day indicates a creatine kinase level more than 10 times the upper limit of normal. Which of the following is MOST consistent with these clinical findings? A. Hyponatremia B. Hypokalemia C. Rhabdomyolysis D. Myasthenia gravis

C. Rhabdomyolysis Rhabdomyolysis is a potentially fatal condition in which myoglobin and other muscle tissue contents are released into the bloodstream as a result of muscle tissue disintegration. This could occur with acute trauma, severe burns, overexertion, from alcohol abuse or alcohol poisoning or with statins. It leads to muscle aches, cramps, weakness and soreness. Dark color of urine is due to liver failure.

A patient presents with limited mouth opening of 25 mm due to pain. There is no complaint of clicking sound or mouth deviation when he opens his mouth. The patient is unable to completely close his mouth with teeth clenched together. What is the MOST likely diagnosis? A. Hypomobility B. Disc displacement with reduction C. Synovitis D. Capsulitis

C. Synovitis There is no clicking sound so there is no reduction of a displaced disc so you can rule out B. If it was simply hypomobility you would not have pain so you can rule out A. Synovitis has NO DEVIATION but capsulitis has deviation!!!

A 54-year-old patient is admitted to the hospital for lung congestion. The patient's daughter, who works in the same hospital as a respiratory therapist, visits him in the hospital. She wants to look at her father's medical record. The PT should: A. Tell her to ask the consultant physician for permission B. Tell her she cannot see the chart because she could misinterpret the information C. Tell her that she must have the permission of her father before she can look at the chart D. Giver her the chart and let her read it as she may have some insights

C. Tell her that she must have the permission of her father before she can look at the chart

A patient arrives to therapy and the physical therapist notices that the patient has edema. When testing for the edema, the PT notices that there is a deep indentation that returns to normal in 25 seconds. What stage of pitting edema does the patient have? A. 4+ B. 2+ C. 3- D. 3+

D. 3+ 0: no edema 1+: slight pitting (2 mm), rebounds immediately 2+: somewhat deeper pit (4 mm), rebounds in <15 seconds 3+: deep pit (6 mm), rebounds in 15-30 seconds 4+: very deep pit (8 mm), >30 seconds to rebound

A PT examines a patient who complains of foot pain while running. The examination shows that the patient has excessive foot pronation. Which of the following would be the MOST appropriate orthotic insert? A. A lateral forefoot post under the fifth metatarsal head B. A lateral rearfoot post under the calcaneus placing it in an everted position C. A middle post just proximal to the third metatarsal head D. A medial post just proximal to the first metatarsal head

D. A medial post just proximal to the first metatarsal head

A physical therapist is performing joint mobilizations on a patient in the outpatient clinic. The PT notes the electrocardiogram in the photograph. The physical therapist's INITIAL response should be to: A. Continue with joint mobilizations at same intensity B. Continue with joint mobilizations at a lower intensity C. Stop the treatment and monitor ECG for 10 minutes D. Activate the emergency system or call 911

D. Activate the emergency system or call 911 This ECG is demonstrating ventricular fibrillation. A ventricular fibrillation is an emergency and you should call 911 immediately.

During your nerve examination of extraocular muscle function, you note that your patient has vertical nystagmus during smooth pursuits. The MOST likely diagnosis for this condition is: A. Lesion of optic nerve B. Posterior canal BPPV C. Acoustic neuroma D. Central nervous system lesion

D. Central nervous system lesion In central nervous system pathology vertical and pendular nystagmus is observed. In peripheral vestibular system pathology horizontal nystagmus is observed.

A patient has right flank pain after sustaining a blow to the back during a sporting event. The spine is pain free upon palpation. The paraspinal muscles are free of muscle spasms and tenderness. Percussion of the right costovertebral angle reproduces the pain and causes the pain to radiate to the right groin area. Which of the following structures is MOST likely involved? A. Bladder B. Appendix C. Spleen D. Kidney

D. Kidney The kidneys are located in the region of the costovertebral angle. Pain upon percussion of this region is common in kidney involvement. Pain associated with the kidneys usually refers to the ipsilateral flank and groin

Which of the following tests or measurements is BEST to assess risk for skin ulceration in a patient who has diabetes? A. Rate pressure product B. Pulse pressure C. Capillary refill D. Light touch

D. Light touch Because peripheral neuropathy is a common secondary complication of diabetes, it is imperative for a physical therapist to assess a patient's ability to detect light touch and presence of protective sensation to determine risk of skin ulceration

A patient who reports a history of liver damage exhibits signs of hyperbilirubinemia. Which of the following clinical manifestations is MOST likely present? A. Pallor B. Clear urine C. Darkened stools D. Light-colored stools

D. Light-colored stools The liver removes bilirubin from the blood, conjugates it, and excretes this complex into the intestine. It is this conjugated bilirubin that darkens fecal material. Liver disease that limits bilirubin transport into the liver increases serum bilirubin (hyperbilirubinemia) and limits the amount of conjugated bilirubin entering the gut. Stools become light in color. This sign helps to distinguish jaundice caused by liver disease from jaundice caused by extrahepatic causes.

A patient came to a clinic with the diagnosis of end stage renal disease. The PT is most concerned about prescribing a program for general conditioning. During treatment, the PT notices that the patient is developing skin pallor, fatigue, and dyspnea. The PT monitors their vitals, and will expect which value to be MOST affected based on the patient's diagnosis? A. Increase SBP B. Increase DBP C. Decrease SBP D. Normal or lower than normal DBP

D. Normal or lower than normal DBP Patients with end stage renal diseases have decreased Diastolic BP. Increase in BP is a cause for renal failure.

A 58-year-old patient comes to the PT clinic for a gait and posture evaluation. The patient has a right sided transtibial amputation and is unable to fully flex the right knee in stance phase. What is the MOST likely cause for this gait deviation? A. Anterior displacement of the socket relative to the foot B. The knee bolt is rotated externally C. The prosthesis is too light D. Posterior displacement of the socket relative to the foot

D. Posterior displacement of the socket relative to the foot Stick to your plane! You can automatically rule out B. If the socket is too far posterior the leg will want to fall back into knee extension so there will be a decrease in knee flexion.

A physical therapist attempts to identify an appropriately sized wheelchair for a patient. The therapist determines that the patient's hip width in the sitting position and the measurement from the back of the buttocks to the popliteal space are each 16 inches. Given these measurements, which of the following wheelchair specifications would BEST fit this patient? A. Seat width 16 inches, seat depth 14 inches B. Seat width 18 inches, seat depth 18 inches C. Seat width 16 inches, seat depth 18 inches D. Seat width 18 inches, seat depth 14 inches

D. Seat width 18 inches, seat depth 14 inches Seat width = hip width + 2 inches (You want extra room on both sides to be comfortable!) Seat depth = posterior buttock to popliteal space - 2 (You don't want the seat rubbing behind the knees!)

Which of the following substitution patterns should be prevented when measuring active forearm supination? A. Shoulder medial (internal) rotation and shoulder abduction B. Shoulder medial (internal) rotation and shoulder adduction past 0 degrees C. Shoulder lateral (external) rotation and shoulder abduction D. Shoulder lateral (external) rotation and shoulder adduction past 0 degrees

D. Shoulder lateral (external) rotation and shoulder adduction past 0 degrees

A physical therapist instructs a patient to squeeze a piece of paper between the index and middle fingers while the therapist attempts to pull it away. This type of testing would be the MOST appropriate to assess which of the following myotomes? A. C6 B. C7 C. C8 D. T1

D. T1 C6- elbow flexion and wrist extension C7- elbow extension and wrist flexion C8- thumb extension T1- finger abduction and adduction

A patient who is scheduled for a right total hip arthroplasty (THA) is referred to PT. A posterolateral surgical approach will be used. Following surgery what is the MOST appropriate bed positioning intervention for the PT to recommend? A. Change of position from supine to prone every 2 hours B. Use of a hospital bed to elevate the lower extremities to 90 degrees C. Side-lying with the lower extremities adducted D. Use of an abductor pillow between the lower extremities

D. Use of an abductor pillow between the lower extremities Posterolateral precautions: avoid IR past neutral, avoid adduction past neutral, avoid hip flexion past 90 degrees

As part of a cognitive assessment, a physical therapist asks a patient to count from one to twenty-five by increments of three. What component of cognitive function does this task MOST accurately assess? A. Attention B. Constructional ability C. Abstract ability D. Orientation

A. Attention Attention can be assessed by asking a patient to count from 1-25 by increments of 3. The task should be relatively easy for most individuals; however, it requires the person to exert a sustained, consistent effort. Attention deficits are common with many neurological disorders including brain injury, stroke, and dementia.

A PT is running an education program for pregnant females about the changes seen during pregnancy. Which of the following is MOST appropriate regarding physiological anatomical changes related to pregnancy? A. Blood pressure decreases in first half of the pregnancy and then increases; heart rate decreases about 15-20 bpm and returns to normal after delivery B. Cardiac output increases with pregnancy and blood volume levels stay the same or decrease throughout pregnancy C. Normal fasting glucose levels are usually lower but metabolic rate increases D. Center of gravity shift inferiorly and posteriorly causing compensation in the form of cervical and lumbar lordosis

A. Blood pressure decreases in first half of the pregnancy and then increases; heart rate decreases about 15-20 bpm and returns to normal after delivery The COG shifts ANTERIORLY and SUPERIORLY so you can rule out D. Cardiac output increases but blood volume levels also INCREASE so you can rule out B.

A 39-year-old female presents to physical therapy with new complaints of increased fatigue, cold hands and feet. Upon assessment, dependent edema was present bilaterally. Which of the following tests or measures are the MOST appropriate for a PT examination? A. Body temperature and lung auscultation for rales B. Heart rate and blood pressure C. MMSE and lung auscultation for rales D. Turgor and CBC

A. Body temperature and lung auscultation for rales The patient is presenting the signs of CHF, so we will assess the body temperature and lung auscultation for rales. Rales are crackling/bubbling sounds suggesting fluid in the lung. Heart rate and blood pressure measurements are not specific to CHF. MMSE is done for neurological conditions. Turgor is for dehydration.

A PT assesses a patient's voice sounds as part of a respiratory examination. The therapist positions the stethoscope over the thorax and asks the patient to say "99". Which type of voice sound is assessed using this technique? A. Bronchophony B. Egophony C. Pectoriloquy D. Rhonchi

A. Bronchophony Egophony- have the patient say a long "E" and it turns into a nasally "A" Pectoriloquy- whisper "1, 2, 3" and it is loud to the PT Rhonchi- low-pitched, rattling sounds that resemble snoring in patients with COPD, bronhiectasis, pneumonia, chronic bronchitis, and CF

A patient has a pressure ulcer as shown in the picture. The wound has necrotic tissue and heavy exudate is present. Which of the following is the BEST wound care option? A. Calcium alginate B. Hydrocolloid dressing C. Hydrogel dressings D. Transparent films

A. Calcium alginate If there is necrotic tissue it is safe to assume the wound is infected. Hydrogels and calcium alginate are the only dressings used with infected wounds so you can automatically rule out B and D. Hydrogel dressings are used for minimal exudate so that would not be appropriate. Calcium alginate can be used for an infected wound with heavy exudate.

Which one of the following sympathetic cardiovascular changes occurs in a patient with long-standing type 1 diabetes mellitus? A. HR becomes fixed B. Higher resting HR C. Lower resting HR D. Exercise increases HR

A. HR becomes fixed Long term complication of Type 1 diabetes mellitus is cardiac denervation syndrome which results in a fixed HR that is unresponsive to exercise, stress or sleep.

A patient who gave birth 2 weeks ago with a vaginal delivery is referred to physical therapy for a strengthening program. Which of the following muscles is LEAST likely to require strengthening intervention? A. Hip flexors B. Trunk flexors C. Hip extensors D. Pelvic floor muscles

A. Hip flexors Trunk flexors, hip extensors and pelvic floor muscles are likely to become weak after vaginal delivery. However, hip flexors have a tendency to become short due to anterior pelvis tilting. Therefore, hip flexors need stretching as an intervention not strengthening.

Which of the following joint mobilization techniques would MOST effectively increase elbow joint flexion? A. Humeroulnar distraction B. Humeroradial posterior glide C. Radioulnar anterior glide D. Radioulnar posterior glide

A. Humeroulnar distraction The purpose of humeroulnar distraction is to increase flexion (or extension) of the elbow joint.

A patient complains of R hip pain. The PT observed hip malalignment of coxa valga along with this related posture. Which of the following would be LEAST likely to occur with this posture? A. Medial rotation of leg B. Lateral rotation of leg C. Long ipsilateral leg D. Posterior pelvic tilt

A. Medial rotation of leg Coxa valga would mean you have genu varus at the knee. Genu varus is characterized by LATERAL rotation.

A physical therapist completes an examination on a patient diagnosed with Parkinson's disease. Results of the examination include Good (4/5) strength in the lower extremities, 10 degrees flexion contracture at the hips, and exaggerated forward standing posture. The patient has difficulty initiating movement and requires manual assistance for gait on level surfaces. Which of the following activities is the MOST appropriate to incorporate into a home program for this patient? A. Prone lying B. Progressive relaxation exercises C. Lower extremity resistive exercises with ankle weights D. Postural awareness exercises in standing

A. Prone lying Prone lying is a commonly employed positional technique designed to stretch the hip flexors in patients with Parkinson's disease. Prone lying is a static positioning activity designed to stretch the hip flexors. If the patient was unable to tolerate prone lying, the physical therapist could place one or more pillows under the patient's hip and gradually remove pillows over time as the patient improves their flexibility. Other options are appropriate, but they will not provide the same degree of benefit.

You are evaluating a 30 year old patient with right elbow pain radiating into the forearm. The patient works as a bagger at a grocery store and reports his pain began 6 months ago after carrying a heavy box at work. The patient reports his pain is located on the medial side of the elbow and is worsened with prolonged gripping and carrying grocery bags. Which of the following diagnoses is MOST likely to be accurate for this patienterm-106t? A. Tendinopathy of the wrist flexors B. Biceps tendinopathy at distal insertion C. Biceps tendinopathy at proximal insertion D. Lateral epicondylalgia

A. Tendinopathy of the wrist The pain is on the medial elbow which is the location for the flexor tendon. (The lateral elbow is where the extensors are!) Flexor tendinopathy is known as "golfer's elbow" and is caused by repetitive wrist flexion. You can rule out D because that impacts the extensor tendon the lateral side of the elbow. Biceps tendinopathy makes no sense because the pain goes into the forearm.

A 45-year-old male with a BMI of 38 kg/m2 is enrolled in a 6 week fitness training program. Which is the MOST appropriate measure to assess change in fitness from pre and post fitness training? A. The time it takes for the heart rate to return to baseline B. Resting respiration rate at pre-training C. Rating on a Wong baker scale D. Increase in blood pressure during exercise

A. The time it takes for the heart rate to return to baseline A BMI of 38 classifies them as obese so you can assume they are out of shape. An unfit heart takes longer than a fit heart to return to baseline so measuring the time it takes to return to baseline is an appropriate measurement for heart health. The Wong baker scale is a pain scale for children made out of faces

Patient experiences range of motion limitation with left side bending and left rotation at the C5-6 junction. What is the MOST appropriate intervention? A. Apply PA glide at left C5 B. Apply PA glide at left C6 C. Apply PA glide at right C5 D. Apply PA glide at right C65

B. Apply PA glide at left C6 If they are struggling with left side bending and left rotation they are having difficulty CLOSING the gap on the left side. "Bottoms up" means you would do a PA glide at C6 to close the gap. OPENING THE GAP ON THE RIGHT SIDE (C) IS ALSO AN OPTON BUT SAME SIDE IS ALWAYS PREFERRED

50-year-old male patient with peripheral artery disease was referred for physical therapy. During the examination, PT examines patient's peripheral pulse as shown in the picture. Which artery is being palpated in the picture? A. Popliteal artery B. Dorsalis pedis artery C. Femoral artery D. Posterior tibial artery

B. Dorsalis pedis artery

Following multiple rib fractures on one side and an ipsilateral pneumothorax, which of the following pulmonary function tests is MOST effective to measure the patient's improvement in ventilation? A. Alveolar ventilation B. Inspiratory capacity C. Minute ventilation D. Total lung capacity

B. Inspiratory capacity The patient has a restrictive extrapulmonary condition that will most likely impair lung expansion and the amount of air being mobilized in each ventilation cycle. Inspiratory capacity refers to the largest volume of air that can be inspired in one breath from the resting expiratory level, and it can easily be measured with an incentive spirometer.

The patient suffered an injury on the right side and is having difficulty in pronation of the right arm. Which nerve is MOST likely injured? A. Ulnar nerve B. Median nerve C. Radial nerve D. Suprascapular nerve

B. Median nerve The pronator muscles of forearm i.e. pronator quadratus, pronator teres are supplied by median nerve. So, injury to median nerve will result in weak pronation.

In a patient who has weak oblique, rectus abdominis, and transversus abdominis muscles, which of the following interventions is MOST likely to improve the mechanical efficiency of the diaphragm? A. Use of a rigid trunk support B. Use of an abdominal binder C. Assuming an erect sitting position D. Assuming a forward leaning sitting position

B. Use of an abdominal binder Use of an abdominal corset in patients who have weak abdominal muscles can compensate for laxity and can improve respiratory function (Nichols-Larsen). SITTING WILL WORSEN THE DIAPHRAGM'S FUNCTION! Sitting shifts the abdominal contents inferiorly and anteriorly making it harder for the diaphragm.

A PT is educating one of their patients regarding the effects of pregnancy and the implication for positioning and posture. Which of the following statements is LEAST appropriate and must not be recommended by the therapist? A. Patient with pregnancy induced hypertension must not do moderate intensity exercises B. Breath-holding exercising and Valsalva's maneuver should be avoided C. A small wedge must be used under the left hip while sleeping in supine in the second trimester D. Borg's scale score of 12-14 during exercise is acceptable for uncomplicated pregnancy

C. A small wedge must be used under the left hip while sleeping in supine in the second trimester There are no limitations on exercise for a HEALTHY pregnancy but if there is pregnancy induced hypertension you need to be cautious so you can rule out A and D. Valsalva maneuver and breath-holding exercises increases the intraabdominal pressure so you would avoid that and can rule out B. If you put a wedge under your left hip in supine that is pushing you into right side-lying. ALWAYS LAY IN LEFT SIDELYING!!!

A patient's electrocardiogram shows a new ST-segment elevation from baseline and a sinus rhythm of 68 bpm. What is the MOST likely diagnosis? A. Bradycardia B. Low blood pressure C. Acute myocardial infarction D. Congestive heart failure

C. Acute myocardial infarction Bradycardia is <60 bpm

A PT is treating a patient who incurred a SCI secondary to a flexion injury status post a MVA. The patient has loss of motor function and pain and temperature below the level of the lesion; however, the patient has preserved proprioception and light touch. The PT should suspect the patient has which of the following spinal cord syndromes? A. Central cord syndrome B. Posterior cord syndrome C. Anterior cord syndrome D. Brown sequard syndrome

C. Anterior cord syndrome Central cord: bilateral motor loss of the UE's, loss of pain and temperature (hyperextension injuries) Posterior cord: bilateral loss of sensation Anterior cord: bilateral loss of motor function, loss of pain and temperature (hyperflexion injuries) Brown sequard: ipsilateral motor and sensory loss, contralateral pain and temperature loss

A 28 year-old female, 8 months pregnant, is working out in an outpatient clinic. The plan of care includes strengthening the core muscles by using the therapeutic ball. Which of following is the BEST intervention to train her lower abdominal muscles while sitting on the ball? A. Displace the ball anteriorly B. Displace the ball posteriorly C. Sit on the ball and flex both knees D. Training on ball is not recommended

A. Displace the ball anteriorly One of the main functions of lower abdominal muscles is posterior pelvic tilt. So, by using the therapeutic ball you are promoting posterior pelvic tilt. When the patient sits on the ball and moves in anteriorly, it causes posterior pelvic tilt. So, contraction of the lower abdominal results in posterior tilting of the pelvis and will help in strengthening these muscles.

A PT is training a 65-year-old to walk with a quadripod. He has severe arthritis of the R hip and was operated on two months ago. What is the MOST appropriate way the PT should instruct the patient to use the assistive device? A. Hold the cane in your L hand and move L hand and R leg together B. Hold cane in your R hand and move R hand and L leg together C. Hold cane in your L hand and move L hand and L leg together D. Hold cane in your R hand and move R hand and R leg together

A. Hold the cane in your L hand and move L hand and R leg together If the right side is affected you want the patient to hold the cane in the LEFT hand so you can rule out B and D. Moving the left leg and left hand at the same time is not a natural way to ambulate.

A physical therapist designs an exercise program for a patient who is three weeks post cardiac transplantation. Which of the following treatment modifications would be the MOST essential to incorporate into the patient's exercise program based on the transplantation? A. Increased warm-up and cool down period B. Increased duration of training sessions C. Increased target heart rate range during exercise D. Increased monitoring of exercise intensity through heart rate

A. Increased warm-up and cool down period A patient with a cardiac transplant would require an increased warm-up and cool down period. For several months after the transplant, the transplanted heart fails to respond normally to sympathetic nervous stimulation. Specifically, the heart rate response to exercise and recovery is delayed, thus requiring the increased warm-up and cool down periods. Heart rate is not a good measure of exercise intensity with these patients due to an abnormal response to exercise!!! Use RPE!!!

Documenting the care provided to the patient is essential and must be completed in a timely manner. Which of the following is NOT appropriate with respect to documentation? A. When a charting error is made, use white-out material to correct the text B. Mistakes must be crossed out with a single line through the error, and then both initialed and dated by the therapist C. When a charting error is made, it must be clearly indicated that a change was made without deleting the original record D. Medically approved symbols or abbreviations can be used for documentation

A. When a charting error is made, use white-out material to correct the text Documentation should be consistent with the Guidelines for PT Documentations, APTA. According to the Guidelines, white-out material must never be used to correct text in a medical record. All other statements are true.

A physical therapist prepares to treat a patient using continuous ultrasound. What general rule BEST determines the length of treatment time when using ultrasound? A. Two minutes for an area that is two times the size of the transducer face B. Five minutes for an area that is two times the size of the transducer face C. Five minutes is the maximum treatment time regardless of the treatment area D. Ten minutes is the maximum treatment time regardless of the treatment area

B. Five minutes for an area that is two times the size of the transducer face An accepted recommendation is that ultrasound can be administered to an area 2-3 times the size of the effective radiating area of the transducer face in a five minute period. This recommendation equates to roughly twice the size of the transducer face.

A physical therapist utilizes a special test in which the therapist passively performs the required motion without the need for active participation by the patient. Which test would meet this criterion? A. Adson maneuver B. Hawkins-Kennedy impingement test C. Roos test D. Speed's test

B. Hawkins-Kennedy impingement test The Hawkins-Kennedy impingement test is performed with the patient positioned in sitting or standing. The therapist flexes the patient's shoulder to 90 degrees and then medially rotates the arm. A positive test occurs when there is an onset or exacerbation of pain and may be indicative of shoulder impingement involving the supraspinatus tendon.

A 28-year-old software engineer reporting difficulty at work is referred to an outpatient physical therapy clinic. The patient complains of pain and numbness and tingling sensations in her left forearm, elbow, and hand. The PT suspects a median nerve injury but Phalen's test is negative. To confirm the median nerve involvement, the PT should MOST likely expect pain and weakness during which of the following motions? A. Left forearm pronation, finger flexion, and thumb adduction B. Left forearm pronation, finger flexion, and thumb opposition C. Left forearm supination, finger adduction, and thumb opposition D. Left forearm pronation, finger extension, and thumb extension

B. Left forearm pronation, finger flexion, and thumb opposition Median nerve injury causes difficulty with forearm pronation due to impaired pronator teres muscle, difficulty with index finger flexion (due to impaired FDS and FDP) and thumb opposition (ape hand deformity) due to impaired thenar muscles which are all supplied by the median nerve.

A 22-year-old patient who sustained multiple injuries, including a fractured tibia and a traction injury to the brachial plexus, is ready to begin gait training. The patient is partial weight bearing and has good upper extremity strength. Which of the following assistive devices is the MOST appropriate for the physical therapist to select for this patient? A. Axillary crutches B. Lofstrand crutches C. Walker with platform attachment D. Cane

B. Lofstrand crutches You do not want to risk putting pressure on the brachial plexus so you immediately rule out A. You can also rule out D because a cane does not allow for partial weightbearing. So you are left with B and C. While a walker with platform attachment is a reasonable option for the patient, it may offer TOO much support for such a young patient with good strength.

A patient post traumatic brain injury is presently at the confused-appropriate level of cognitive functioning. The patient has progressed well in therapy, however, has been bothered by diplopia. Which of the following treatment strategies would be the MOST appropriate to address diplopia? A. Provide non-verbal instructions within the patient's direct line of sight B. Place a patch over one of the patient's eyes C. Ask the patient to turn their head to one side when experiencing diplopia D. Instruct the patient to carefully focus on a single object

B. Place a patch over one of the patient's eyes A patient with diplopia is often instructed to wear a patch alternately over one of their eyes. Specific strengthening exercises of the extraocular muscles can serve to improve the patient's vision.

During palpation, a physical therapist determines that the spine of a patient's scapula is level with the spinous process of T2. Which postural deformity is MOST likely to be associated with this clinical finding? A. Forward head B. Shoulder elevation C. Rounded shoulders D. Scapular winging

B. Shoulder elevation The spine of the scapula typically aligns with T3. Elevation of the scapula commonly results in the spine of the scapula appearing to be level with a spinous process above its normal position at T3.

A patient who had a total knee arthroplasty demonstrates 0° to 85° of active knee range of motion. The patient will likely have the MOST difficulty performing which of the following activities in a normal fashion? A. Sitting down on a chair seat that is 18 inches (46 cm) high without using the hands B. Tying shoelaces while sitting and bringing the foot up from the floor C. Reciprocally ascending stairs that have a 5-inch (12.7 cm) rise D. Reciprocally descending stairs that have a 5-inch (12.7 cm) rise

B. Tying shoelaces while sitting and bringing the foot up from the floor The average value for knee flexion needed to tie shoelaces while sitting and bringing the foot up from the floor is 106° ± 9° (p. 371). Of the activities listed, this would be the most difficult for the patient.

The therapist is assessing a cyclist who complains of knee pain. The therapist is assessing the bicycle of the athlete. Which of the following factors can cause increased stress on the ITB? A. Low seat height B. Seat set too forward C. Cleats are excessively internally rotated D. Cleats are excessively externally rotated

C. Cleats are excessively internally rotated If it is internally rotated you have to put more stress on the outside resulting in ITB irritation.

A physical therapist prepares to use phonophoresis as a component of a patient's plan of care, but is concerned about the potential of the ultrasound to exacerbate the patient's current inflammation. Which of the following methods would be the MOST effective to address the therapist's concerns? A. Utilize ultrasound with a frequency of 1 MHz B. Limit treatment time to five minutes C. Incorporate a pulsed 20% duty cycle D. Select an ultrasound intensity less than 1.5 W/cm^2

C. Incorporate a pulsed 20% duty cycle When ultrasound is used in an pulsed mode with a 20% or lower duty cycle, the heat produced during the on time of the cycle is dispersed during the off time and as a result there is no measurable net increase in temperature. Ultrasound using a 20% or lower duty cycle would typically be used for nonthermal effects.

A 50-year-old male patient visited cardiac rehabilitation center referred by cardiologist with diagnosis of right ventricular failure. Which of the following is INCORRECT regarding right ventricular failure? A. S3 heart sound B. Increased pulmonary artery pressure with peripheral edema C. Increased pulmonary artery pressure and pulmonary edema with dyspnea on exertion D. Venous hypertension and stasis

C. Increased pulmonary artery pressure and pulmonary edema with dypsnea on exertion This describes left heart failure! Left-sided heart failure: fluid may back up in your lungs, causing SOB Right-sided heart failure: fluid may back up into your abdomen, legs, and feet causing swelling

A PT is assessing a patient's lymph nodes 6 months post-chemotherapy treatment. When assessing the lymph nodes, which presentation LEAST likely requires referral to physician? A. Hard and immobile lymph nodes less than 1 cm in diameter B. Rubbery and firm lymph nodes more than 1 cm in diameter C. Soft and non tender lymph nodes less than 1 cm in diameter D. Palpable and tender lymph nodes more than 1 cm in diameter

C. Soft and non tender lymph nodes less than 1 cm in diameter

A 74-year-old contractor has a prominent forward lean when he is in stance on the right lower extremity. From this observation, the PT would hypothesize that the MOST LIKELY cause is: A. Weak hip extensors on the left B. Weak hip flexors on the left C. Weak knee extensors on the right D. Weak knee flexors on the right

C. Weak knee extensors on the right You can rule out A and B because these impairments only apply when the individual is in stance phase which would be on the RIGHT side. With a forward lean you would have tightness in the hip extensors and weakness in the hip flexors. It is also associated with excessive PF which would result in hyperextension at the knees. If the knee extensors, such as the quads, are weak on the right side you would get knee hyperextension and a resultant forward lean. Causes of excessive knee extension: quad weakness, forward lean, hip flexor weakness, and extension contractures

A group of physical therapists designs a research study in which they record the shoulder ROM before and after treatment in three different age groups: adolescents, teenagers, and young adults. The therapists want to determine if there is a difference in the treatment effect based on the age of the patient. What statistical test would MOST likely be used to compare the differences between the three groups? A. T-test B. Z-test C. Chi-square test D. Analysis of variance (ANOVA) test

D. Analysis of variance (ANOVA) test An analysis of variance (ANOVA) test is a statistical test that is used when three or more variables are being compared. In the described scenario, the researchers are comparing the means of three different groups, therefore, an ANOVA test would be the most appropriate statistical test.

A physical therapist was observing a 65-year-old male patient participating in cardiac rehabilitation. The patient suddenly feels discomfort and dizziness, ECG strip is showing 3-4 ectopic beats in groups. What should be the therapist's first responsibility? A. Call 911 B. Start CPR C. Continue exercises D. Assist the patient to be in a comfortable position

D. Assist the patient to be in a comfortable position First responsibility is to confirm the patient's safety. THEN you call 911 or start CPR.

A patient with spinal cord injury has trouble wearing earrings and tying her hair. On assessment she was able to perform sit to stand transfers and maneuver the stairs relatively safely. She is a school teacher and her goals include being able to write on the whiteboard. Which of the following types of SCI is the patient MOST likely expected to have? A. Cauda equina syndrome B. Anterior cord syndrome C. Posterior cord syndrome D. Central cord syndrome

D. Central cord syndrome The patient demonstrates problems with the motor function of the UE's but NOT the LE's which is characteristic of a central cord syndrome. Anterior cord: bilat. motor loss, pain and temperature loss (hyperflexion injuries) Posterior cord: bilateral sensory loss Central cord: bilateral motor loss of the UE's, pain and temperature loss (hyperextension injuries such as MVA)

A six-month-old patient with developmental hip dysplasia is fitted with a Pavlik harness to help promote proper alignment of the hip joints. Which of the following motions would be the MOST restricted with this harness? A. Flexion and abduction B. Flexion and adduction C. Extension and abduction D. Extension and adduction

D. Extension and adduction Because the Pavlik harness positions the infant's hips in flexion and abduction it would restrict the opposing motions (extension and adduction). Studies have found that the positions of extension and adduction promote hip dislocation. The Pavlik harness attempts to minimize these motions and thus reduce the incidence of hip dislocation.

A 6-month-old child was referred to physical therapy for right torticollis. The MOST effective method to stretch the muscle is by positioning the head and neck into: A. Flexion, left side-bending, and left rotation B. Extension, right side-bending, and left rotation C. Flexion, right side-bending, and left rotation D. Extension, left side-bending, and right rotation

D. Extension, left side-bending, and right rotation With right torticollis you have tightness in the right SCM. The SCM does neck flexion, contralateral rotation, and ipsilateral neck flexion. So with right torticollis you will see flexion, right side-bending, and left rotation. Therefore, in order to stretch the right SCM you will do the opposite motions.

A patient with an ASIA B L1 spinal cord injury is asking their physical therapist what to expect with their bladder control. Which of the statements most accurately classifies the bladder dysfunction as well as treatment needed for this patient? A. Spastic urinary dysfunction, suprapubic tapping B. Flaccid urinary dysfunction, suprapubic tapping C. Spastic urinary dysfunction, intermittent bladder catherization D. Flaccid urinary dysfunction, intermittent bladder catherization

D. Flaccid urinary dysfunction, intermittent bladder catherization Anything below T12 will be flaccid urinary dysfunction and will require intermittent catherization every 3-6 hours! Above T12: spastic urinary dysfunction ASIA B: sensory intact but no motor

During the initial examination, a physical therapist determines a patient has 3/5 strength of the right subscapularis. Which of the following activities would be MOST difficult for the patient to perform? A. Performing a push-up plus with bilateral upper extremities B. Elevation of the right scapula C. Extension of the glenohumeral joint D. Flexion of the humerus overhead

D. Flexion of the humerus overhead Subscapularis is an important muscle of rotator cuff, stabilizing the humeral head in the glenoid cavity; thus preventing superior translation of the humerus. It is an important muscle in overhead sports. Tear of Subscapularis can weaken its hold over the Long head of the biceps causing difficulty in overhead activity.

A PT is treating a patient in an outpatient setting for a knee pain when he complains of sharp abdominal pain. The PT palpates by pushing in the left lower quadrant, but the pain is elicited on the right side. Which of the following is the MOST appropriate intervention? A. Apply a moist hot pack to the back and ask him to consult his physician B. Encourage the patient to ignore the pain and stretch C. Give the patient some pain killers and water D. Have him lie down, remain as still as possible, and notify the physician immediately

D. Have him lie down, remain as still as possible, and notify the physician immediately This is an example of Rovsing's sign for point migration. A positive test indicates possible appendicitis.

A 55-year-old male reports of pain and tingling over a localized area on the posterior trunk with fever and chills over the past few days. There are red papules over a small area in a dermatomal distribution. What is the MOST likely diagnosis and medication? A. Herpes simplex; antiviral drugs B. Herpes zoster; antibacterial drugs C. Herpes simplex; antibacterial drugs D. Herpes zoster; antiviral drugs

D. Herpes zoster; antiviral drugs Herpes simplex are cold sores on the lips so you can rule A and C out. If they have a fever and chills you know it is viral.

Which muscles are most important when looking to increase running velocity? A. Plantar flexors B. Hip flexors C. Knee flexors D. Hip extensors

D. Hip extensors The ability to extend the hips is the most important factor in determining sprinting speed. The research has also shown that stride length may be more important than stride frequency when it comes to running speed.With the right femur held relatively fixed, contraction of the external rotators would rotate the pelvis and the attached trunk to the left. This action of planting the limb and cutting to the opposite side is a natural way to abruptly change direction while running. The gluteus maximus appears uniquely designed to perform this action as, with the right limb securely planted, a strong contraction of the gluteus maximus would, in theory, generate a very effective extension and external rotation torque about the right hip, helping to provide the necessary thrust to the combined- cutting and propulsion action.

A patient with type 1 diabetes is planning to begin an exercise program. Which of the following actions is MOST appropriate for the patient to perform? A. Inject insulin just prior to starting an exercise program B. Avoid food consumption just prior to an exercise session C. Complete an exercise session within 1 hour of receiving an insulin injection D. Increase food intake prior to an exercise session

D. Increase food intake prior to an exercise session Exercise typically increases insulin sensitivity and enhances the effect of insulin. Therefore, glucose intake should be increased to counter the effects of exercise. Insulin intake could be decreased to counter the effects of exercise. During prolonged activity, a snack is recommended for every 30 minutes of activity.

A 63-year-old female patient has undergone surgery and has been on bed rest for four days. Therapist visits the patient to prepare her for ambulation. While getting the patient up from bed to standing, which of the following BP responses will have the therapist take an action of returning the patient back to bed? A. Decrease in SBP of 10 mmHg and increase in DBP of 3 mmHg B. Increase in SBP of 17 mmHg and increase in DBP of 5 mmHg C. Increase in SBP of 15 mmHg and decrease in DBP of 7 mmHg D. Increase in SBP of 18 mmHg and decrease in DBP of 11 mmHg

D. Increase in SBP of 18 mmHg and decrease in DBP of 11 mmHg > 20 mmHg change for SBP and > 10 mmHg of DBP means you need to stop!

A chronic smoker is diagnosed with COPD and undergoes ABG analysis. The physical therapist is MOST likely to see which of the following changes in the ABG report? A. Decreased PaCO2, increased PaO2, and decreased pH B. Decreased PaCO2, decreased PaO2, and increased pH C. Increased PaCO2, increased PaO2, and increased pH D. Increased PaCO2, decreased PaO2, and decreased pH

D. Increased PaCO2, decreased PaO2, and decreased pH CO2 and O2 are inversely related so you can automatically rule out B and C. People with COPD are struggle to expire all of their air. When you expire you are releasing CO2. Since they are not expiring their CO2 it is beginning to add up. CO2 and O2 are inversely related so O2 would have to go down. The more the CO2 in the body, the more acid the body is thus a lower pH. the CO2 in the body, the more acid the body is thus a lower pH.

The presence of which of the following devices would MOST likely limit mobility activities during physical therapy? A. Ventilator B. Tracheostomy tube C. Temporary pacemaker D. Intracranial pressure monitor

D. Intracranial pressure monitor An intracranial pressure monitor is the correct option because it takes very little to disrupt the values and interfere with the readings, such as changing the bed height. Therefore, mobility activities are significantly limited for patients who have an intracranial pressure monitor.

A physical therapist is administering iontophoresis for the management of a sclerotic scar in a patient with adhesive capsulitis. To administer this treatment, which ion and polarity should be used? A. Zinc, negative pole B. Acetate, positive pole C. Copper, positive pole D. Iodine, negative pole

D. Iodine, negative pole Use I SAD to determine which have a negative pole. Using this, you know that zinc is not negative and acetate is negative. This allows you to rule out A and B. Copper is positive BUT it is used for fungal infections. Iodine is negative and is used for sclerotic scars.

Which of the following descriptions BEST depicts the Cheyne-Stokes respiratory pattern? A. Regular respiration pattern characterized by a rate of less than 10 breaths/minute B. Regular respiration pattern characterized by a rate of more than 24 breaths/minute C. Irregular respiration pattern characterized by highly variable respiratory depth and intermittent periods of apnea D. Irregular respiration pattern characterized by a period of apnea followed by gradually increasing depth of respirations

D. Irregular respiration pattern characterized by a period of apnea followed by gradually increasing depth of respirations This is a typical Cheyne-Strokes respiratory pattern, which is an irregular respiration pattern characterized by a period of apnea followed by gradually increasing depth and frequency of respirations (pp. 57-58). This breathing pattern is often observed with depression of the cerebral hemisphere (e.g., coma), in basal ganglia disease, and occasionally with congestive heart failure.

A physical therapist treats a patient who had a reverse total shoulder arthroplasty. What is the MOST likely reason the patient had this surgical procedure as opposed to a total shoulder arthroplasty? A. Fracture of the humerus B. Glenohumeral osteoarthritis C. Significant deltoid weakness D. Irreparable supraspinatus tear

D. Irreparable supraspinatus tear In cases where a rotator cuff tear cannot be repaired, a reverse total shoulder replacement is indicated. The inverted alignment of the joint surface components allows for greater congruency and stability of the joint surface, decreased translation of the humeral component, and an increased deltoid moment arm. These features allow the patient to elevate their arm in the presence of an insufficient rotator cuff.

After undergoing a reverse total shoulder arthroplasty, a patient is MOST likely to dislocate the shoulder in which of the following positions? A. Lateral (external) rotation and abduction with flexion B. Medial (internal) rotation and abduction with flexion C. Lateral (external) rotation and adduction with extension D. Medial (internal) rotation and adduction with extension

D. Medial (internal) rotation and adduction with extension Patients are most likely to dislocate a reverse total shoulder arthroplasty by performing medial (internal) rotation and adduction in conjunction with extension. This position allows the prosthesis to escape anteriorly and inferiorly.

A patient presents to the clinic with a chief complaint of pain on the sole of the foot. The patient's symptoms were gradual in onset with no history of trauma. Weight-bearing on the affected extremity and hyperextending the great toe aggravate the symptoms. Tenderness is present in the web spaces of the toes with pressure. The patient has full pain-free AROM and 5/5 strength with no increase in symptoms during testing. Which of the following is the MOST LIKELY diagnosis for this condition? A. Plantar fasciitis B. Retrocalcaneal bursitis C. Midfoot sprain D. Morton's neuroma

D. Morton's neuroma Morton's Neuroma is gradual in onset with no known cause. The symptoms include pain on the sole of the foot which increases with WB with overpressure into toe extension during passive ROM. AROM is full and pain free with intact strength. The foot may be pronated with low arches with pain present in the web spaces of the toes. Morton's test is usually positive i.e. squeezing the metatarsal head reproduces the pain.

A 50-year-old male patient is diagnosed with a second-degree heart block type II. What would a physical therapist expect to find on the ECG strip? A. An increase in PR interval lengths with no dropped beats B. No relationship between P waves and QRS complexes C. A gradual increase in PR interval length in all the beats preceding a dropped beat D. Normal PR intervals in all the beats preceding a dropped beat

D. Normal PR intervals in all the beats preceding a dropped beat C describes a second-degree heart block type I so you can rule that out. B describes a third-degree heart block so you can rule that out. A describes a first-degree heart block so you can also rule that out.

Chest percussion may be an appropriate intervention for a patient who has which of the following findings? A. Diastolic pulmonary arterial pressure of 3 mm Hg B. Intracranial pressure of 30 mm Hg C. Platelet count of 30,000/mm^3 D. Partial pressure of arterial oxygen (PaO2) of 70 mm Hg

D. Partial pressure of arterial oxygen (PaO2) of 70 mm Hg Normal partial pressure of arterial oxygen is greater than 80 mm Hg (Hillegass, p. 354). As this value drops, the patient may become tachypneic and tachycardic (Paz, p. 64). Airway clearance techniques, such as percussion, can help to optimize ventilation and perfusion matching, increase gas exchange, and increase alveolar ventilation by mobilizing secretions.

A therapist is examining a patient with history of recurrent patellar subluxations and reports of knee giving way when stepping down a curb. Which of the following is MOST likely correct in this scenario? A. Patient will present with decreased Q angle B. Patient will present with coxa valga C. Patient will present with tightness of hip abductors D. Patient will present with reduced medial longitudinal arch of foot

D. Patient will present with reduced medial longitudinal arch of foot Genu valgum= INCREASE in Q angle

A patient sustained a sprained thumb in a volleyball game five weeks ago and continues to have decreased ROM with carpometacarpal abduction. What direction of glide should the physical therapist use to mobilize the first metacarpal on the trapezium in order to increase carpometacarpal abduction? A. Medial B. Lateral C. Anterior D. Posterior

D. Posterior Abduction and adduction occur in the sagittal plane. The metacarpal surface is convex and the trapezium surface is concave. Since the movement is convex on concave, the roll would be anterior and slide would be posterior.

A 13-year-old girl has been diagnosed with structural idiopathic scoliosis. The physical therapist notices a left thoracolumbar structural scoliosis. Which postural deviation would be expected in this patient? A. The body of the thoracic vertebrae rotated to the right B. High right shoulder C. Spinous processes of thoracic spine rotated to the left D. Posteriorly protruding left scapulae

D. Posteriorly protruding left scapulae The body of the vertebrae goes toward the CONVEX side (left side)

A patient complains of lack of grip strength. PT notices that the patient has a stronger grip with wrist in extension, but the grip strength decreases as the patient starts flexing the wrist. What is the MOST appropriate intervention to improve the ROM? A. Stretch the flexor digitorum and strengthen extensor digitorum B. Strengthen extensor digitorum and put on a flexion splint C. Stretch flexor digitorum and extensor digitorum D. Stretch extensor digitorum and strengthen flexor digitorum

D. Stretch extensor digitorum and strengthen flexor digitorum They are already strong in extension so you don't need to strengthen that. This person has active insufficiency of the flexors and passive insufficiency of the extensors. You always stretch passive and strengthen active.

A physical therapist assesses a patient's upper extremity deep tendon reflexes as part of a screening examination. Which of the following locations is the MOST appropriate to elicit the brachioradialis reflex? A. Radial tuberosity B. Antecubital fossa C. Biceps tendon D. Styloid process of the radius

D. Styloid process of the radius The brachioradialis reflex is tested by tapping the brachioradialis tendon at the distal end of the radius with the flat edge of the reflex hammer

A PT is conducting a 6MWT with a 50-year-old male patient with a quad cane and hypertension to determine his exercise capacity. Which of the following is the least important in giving out the test? A. The patient could walk as far as he can in a normal walking pace B. Standardized encouragement is provided periodically by the PT C. The patient could use his quad cane during the test D. The time stops when the patient stops and rests

D. The time stops when the patient stops and rests A, B, and C are all part of the protocol. The time never stops!

A physical therapist treats a patient with a suspected rupture of the patellar tendon. Which of the following objective findings would be the MOST indicative of this condition? A. Marked tenderness along the anterior surface of the knee joint B. Inability to actively extend the knee against gravity C. Limited ability to complete range of motion due to hemarthrosis D. Resistive isometrics are strong and painful for knee extension

B. Inability to actively extend the knee against gravity A functioning patellar tendon is essential to successfully extend the knee. When the quads contract, the force is transmitted through the patellar tendon using the patella as a fulcrum. Any lack of continuity in this tendon would significantly influence a patient's ability to actively extend the knee against gravity.

A physical therapist reads in the medical record that radiographs confirmed the presence of a reverse Hill-Sachs fracture. What injury would MOST likely be associated with this type of fracture? A. Anterior glenohumeral dislocation B. Posterior glenohumeral dislocation C. Rotator cuff tear D. Biceps tendon rupture

B. Posterior glenohumeral dislocation A REVERSE Hill-Sachs lesion is associated with a POSTERIOR glenohumeral dislocation. An anterior glenohumeral dislocation would be associated with a Hill-Sachs lesion.

The PT performs a ULTT by placing the shoulder in 110 degrees of abduction, elbow in extension, forearm in supination, wrist and fingers in extension. The patient does not report any symptoms. Which of the following modifications are MOST appropriate to test the musculocutaneous nerve? A. Decrease shoulder abduction to 10 degrees and laterally rotate it B. Decrease shoulder abduction to 10 degrees and medially rotate it C. Pronate the forearm and flex the wrist and fingers D. Pronate the forearm with 90 degrees of abduction and elbow flexed

A. Decrease shoulder abduction to 10 degrees and laterally rotate it

A PT is performing phonophoresis at 1.5 W/cm^2, 1.0 MHz, and 20% duty cycle on the distal hamstrings. During treatment, the patient reports feeling pain over the distal hamstrings. The PT's FIRST appropriate course of action is to: A. Decrease the intensity to 1.2 W/cm^2 and continue moving the head at the same speed B. Keep the same intensity at 1.5 W/cm^2 and change the setting to 50% duty cycle C. Increase the intensity to 2.0 W/cm^2 and add more coupling medium D. Turn off the ultrasound and immediately call the physician

A. Decrease the intensity to 1.2 W/cm^2 and continue moving the head at same speed

A physical therapist treats a patient rehabilitating from spinal fusion in the lumbar spine. The surgical procedure required a bone autograft to stabilize the lumbar segment. What post-op finding would be MOST likely based on the utilization of the bone graft? A. Hip pain B. Spinal hypermobility C. Hyporeflexia D. Myotomal weakness

A. Hip pain Bone grafts are most commonly taken from the anterior or posterior portions of the iliac crest. As a result, harvesting of the bone graft is often associated with post-op hip pain, bleeding, and increased swelling.

A patient with an acute burn is referred to physical therapy less than 24 hours after being admitted to the hospital. The patient's burns range from superficial partial-thickness to deep partial-thickness and encompass approximately 35 percent of the patient's total body surface area. Which of the following findings would be the MOST predictable based on the patient's injury? A. Increased oxygen consumption B. Hypernatremia C. Increased intravascular fluid D. Decreased core temperature

A. Increased oxygen consumption An acute burn produces hypermetabolism that results in increased oxygen consumption, increased minute ventilation, and increased core temperature.

A 55-year-old male is undergoing cardiac rehabilitation. The PT is educating the patient on symptoms of exertional intolerance. Which of the following symptoms are the signs for exertional intolerance? A. Persistent dyspnea and angina B. Weight loss and leg stiffness C. Numbness in legs and dizziness D. Insomnia and confusion

A. Persistent dyspnea and angina

An 18-year-old female volleyball player complains of right forearm and hand pain during overhead activities. Upon examination, the physical therapist finds the following range of motion values: Right elbow flexion 10-100, forearm supination 0-75, forearm pronation 0-45, wrist extension 0-75 and wrist radial deviation 0-20 degrees. What is the MOST appropriate intervention to help increase range of motion on the right side? A. Posterior glide of the radius at the proximal radio-ulnar joint B. Dorsal glide of ulna at the distal radio-ulnar joint C. Dorsal glide of radius at the distal radio-ulnar joint D. Medial glide of the radius at the radio-carpal joint

A. Posterior glide of the radius at the proximal radio-ulnar joint As per the given range of motions, the forearm pronation is the limited movement. To improve pronation at proximal radio-ulnar joint, a posterior/dorsal glide of radius is given. To improve pronation at distal radio-ulnar joint, a volar glide of radius is given. Typical pronation ROM is 80-90 degrees!

Which nerve is commonly injured in the Arcade of Frohse? A. Posterior interosseous nerve B. Anterior interosseous nerve C. Lateral cutaneous nerve of forearm D. Ulnar nerve

A. Posterior interosseous nerve The major branch of the radial nerve in the forearm is the posterior interosseous nerve, which may compress as it passes between the two heads of the supinator in the arcade or canal of Frohse.

A patient has a fracture of mid-shaft of humerus after falling on an outstretched hand. Which of the following nerves is most likely to be affected with this type of injury? A. Radial nerve B. Axillary nerve C. Median nerve D. Posterior interosseous nerve

A. Radial nerve The radial nerve can be injured if there is a fracture of the shaft of the humerus. The nerve may be damaged as it winds around behind the humerus in the radial groove and injury may occur at the time of the fracture, or the nerve may get caught in the callus of fracture healing. Axillary nerve can be injured with fracture of surgical neck of humerus. Most common causes of injury to median nerve are impingement in hypertrophied pronator teres and compression in carpal tunnel.

A physical therapist works on weight shifting activities with a patient who is sitting over the edge of a mat table with the feet positioned on the floor. The therapist facilitates an anterior weight shift through the patient's pelvis. What pattern of activity would be required for the patient to maintain an upright posture? A. Spinal extension due to concentric contraction of the spinal extensors B. Spinal flexion due to concentric contraction of the spinal extensors C. Spinal extension due to eccentric contraction of the spinal extensors D. Spinal flexion due to eccentric contraction of the spinal flexors

A. Spinal extension due to concentric contraction of the spinal extensors When a patient initiates an anterior weight shift through the pelvis, spinal extension must occur in order to maintain an upright posture in sitting. Concentric contraction of the spinal extensors is required to produce spinal extension.

When reaching to grasp a glass of water, a patient overreaches and knocks the glass over. Upon further examination, the patient also displays difficulty with finger-to-nose touching. The patient's condition is MOST likely caused by a lesion of which of the following neuroanatomical structures? A. Spinocerebellum B. Cerebrocerebellum C. Vestibulocerebellum D. Cerebellar peduncle

A. Spinocerebellum Spinocerebellar lesions result in a limb ataxia, such as dysmetria. Dysmetria is described as the inability to accurately move an intended distance, which would result in a patient overreaching for a target such as a cup. Cerebrocerebellum- planning the timing and coordination of voluntary movements Vestibulocerebellum- eye movements and postural control

A 39-year-old comes to an outpatient clinic complaining of hip problems. The PT notices a drop of the left hip during right midstance. The MOST appropriate treatment for this impairment would be: A. Stand on right leg and abduct left leg B. Stand on left leg and abduct right leg C. Stand on right leg and flex the right leg D. Stand on left leg and flex the right leg

A. Stand on right leg and abduct left leg Stick to your plane so you can automatically rule out C and D. Both A and B are plausible but you always choose closed-chain over open-chain so A is the correct answer.

Which of the following interventions would be MOST appropriate for a child who has Sever disease? A. Stretch the gastrocnemius and soleus, use a heel wedge B. Stretch the plantar fascia, use an arch support C. Stretch the quadriceps, use a patellar tendon band D. Stretch the tibialis posterior, use a medial heel wedge

A. Stretch the gastrocnemius and soleus, use a heel wedge Sever disease is a calcaneal apophysitis and will benefit from stretching to improve flexibility of the gastrocnemius and soleus and use of a heel wedge to decrease the stress and traction of the Achilles insertion.

Which of the following provocative maneuvers is MOST likely to elicit symptoms in the area highlighted in the photograph? A. Tapping the anterior wrist moving from proximal to distal B. Applying direct pressure to the base of the thumb (1st digit) C. Palpating along the pronator teres D. Compressing the cervical spine with ipsilateral rotation and extension

A. Tapping the anterior wrist moving from proximal to distal The photograph depicts the median nerve distribution, which is commonly disrupted in carpal tunnel syndrome. Tapping the anterior wrist describes the Tinel sign at the carpal tunnel.

A 74-year-old retired contractor walks to the treatment room with a prominent backward lean, when he is in stance on the left lower extremity. From this observation, you hypothesize that the MOST LIKELY cause is: A. Weak hip extensors on the left during stance phase B. Weak hip flexors on the left during stance phase C. Weak hip extensors on the right during swing phase D. Weak hip flexors on the right during swing phase

A. Weak hip extensors on the left during stance phase Deficits caused by weakness are typically on the same side as the weakness so you can rule out C and D (the lean would also be in stance phase). Weakness in the hip extensors would be the cause of a backward lean, whereas tightness in the hip flexors would be the other cause.

A 45-year-old female patient is recovering from a left fibula fracture and is restricted to partial (25%) weight-bearing on the left side. What is the BEST assistive device? A. Cane on the left side B. 2 axillary crutches C. Cane on the right side D. Forearm crutch on left

B. 2 axillary crutches If the fracture is on the left side you would want your cane on the RIGHT side so you can rule out A. You cannot maintain partial weight bearing with a cane or forearm crutch so you can rule out C and D, as well.

Which of the following options BEST represents a typical respiratory rate for a child who is 1 year old? A. 15 breaths/minute B. 30 breaths/minute C. 45 breaths/minute D. 60 breaths/minute

B. 30 breaths/minute The normal respiratory rate for a child who is 1 year old is between 25 and 35 breaths/minute.

A physical therapist is working with a patient who has tuberculosis. Which of the following options BEST describes the appropriate precautions and type of personal protective equipment that a physical therapist should wear? A. Contact precautions, N-95 respirator B. Airborne precautions, N-95 respirator C. Contact precautions, gloves and gown D. Airborne precautions, gloves and gown

B. Airborne precautions, N-95 respirator Airborne precautions: measles, varicella, tuberculosis

A physical therapist examines a patient with coordination deficits who exhibits excessive involuntary and extraneous movements including hemiballismus. This clinical presentation is MOST consistent with damage or a lesion in what area of the brain? A. Cerebellum B. Basal ganglia C. Frontal lobe D. Medulla oblongata

B. Basal ganglia Other basal ganglia deficits include choreoathetosis, hyperkinesis, rigidity, and bradykinesia. Damage to the cerebellum produces ataxia which is the difficulty initiating movement, as well as errors in rate, rhythm, and timing of motor responses (others include dysarthria, dysdiadochokinesia, and nystagmus).

Coxa vara at the hip joint represents a condition when the angle: A. Between the axis of the shaft of the femur and the long axis of the femoral neck is more than 150 degrees B. Between the axis of the shaft of the femur and the long axis of the femoral neck is less than 125 degrees C. Between femoral neck and medial-lateral axes through the femoral condyles is more than 35 degrees D. Between femoral neck and medial-lateral axes through the femoral condyles is less than 35 degrees

B. Between the axis of the shaft of the femur and the long axis of the femoral neck is less than 125 degrees The angle of inclination describes the angle within the frontal plane between the femoral neck and the medial side of the femoral shaft, normal angle is about 125 degrees. A change in the normal angle of inclination is referred to as either coxa vara or coxa valga. Coxa vara describes an angle of inclination markedly less than 125 degrees; coxa valga describes an angle of inclination markedly greater than 125 degrees.

A physical therapist completes a series of upper extremity resisted tests on a patient with suspected cervical spine pathology. Which myotome would BEST be assessed using the test shown in the image? A. C4 B. C5 C. C6 D. C7

B. C5 C4- diaphragm C5- shoulder abduction (primarily deltoids and supraspinatus) C6- elbow flexors or shoulder IR C7- elbow extensors or wrist flexors

A 55-year-old male patient had a right ischemic PCA infarct six months ago. Patient has moderate spasticity throughout the left UE, demonstrating a flexor synergy pattern. During the evaluation, he ran into the doorway on his left side, requiring moderate assist to prevent falling. Based on the location of CVA, he is exhibiting signs/symptoms of: A. Ipsilateral homonymous hemianopsia B. Contralateral homonymous hemianopsia C. Ideomotor apraxia D. Aphasia

B. Contralateral homonymous hemianopsia If it is the right PCA involved it will cause LEFT homonymous hemianopsia (left temporal and right nasal)

A therapist is assessing the gait of a patient with Parkinson's disease. Which of the following is MOST likely to be expected in this patient? A. Decreased steps per turn B. Decreased hip and knee flexion C. Decreased heel strike with decreased forefoot loading D. Decreased step-to-step variability

B. Decreased hip and knee flexion PD patients will require MORE steps when turning so rule out A. If you have decreased heel strike that will put MORE weight in forefoot loading so rule out C. The speed of movement is SLOWER with PD patients and when you walk slower the amount of hip and knee flexion needed is less.

While a patient is doing exercises in cardiac rehabilitation, a physical therapist is supervising the patient's vital signs. The physical therapist decides that the patient should STOP exercising immediately if: A. Increased systolic blood pressure up to 180 mm Hg B. Drop in systolic blood pressure >10 mm Hg C. First degree AV block D. Glucose level of 200 mg/d

B. Drop in systolic blood pressure >10 mm Hg You should not see a drop in SBP with exercise. It should increase.

A physical therapist observes a patient standing in a pool immersed in water to the level of the neck performing a number of upper extremity exercises. Which of the following active movements would be the MOST resisted by buoyancy with the patient starting with the upper extremity positioned at the side and the elbow in 90 degrees of flexion? A. Elbow flexion B. Elbow extension C. Shoulder abduction D. Shoulder medial rotation

B. Elbow extension There is an upward force on the body when immersed in water equal to the amount of water that has been displaced by the body. Elbow extension would go against this upward force. Therefore, elbow extension would be more difficult for the patient.

The patient's medical history includes hypercholesterolemia and type 2 diabetes. The patient's SBP is between 120-129 and DBP is less than 80. Which of the following categories MOST appropriately describes the type of hypertension? A. Normal B. Elevated C. Pre-hypertension D. Stage 1

B. Elevated This is partially a trick question. "Pre-hypertension" is an old term and is no longer used as an answer on the NPTE. So C is ruled out. Normal: <120/<80 Stage 1: 130-139 OR 80-89

A patient presents to PT with complaints of loss of sensation over the deltoid, radial forearm, and hand. On evaluation, there is weakness of the shoulder and elbow muscles however the intrinsic of the hand seems uninvolved. Which of the following is the MOST LIKELY diagnosis? A. Radial nerve palsy B. Erb's palsy C. Klumpke's palsy D. C6 radiculopathy

B. Erb's palsy Erb's palsy (C5-C6) is UPPER brachial plexus and Klumpke's palsy (C8-T1) is LOWER brachial plexus. Klumpke's produces a "Klaw" hand whereas Erb's palsy affects the shoulder.

A patient comes into an outpatient clinic with symptoms of anxiety, hyperventilation, and pain and cramping of the hand. Upon evaluation, the physical therapist notes a respiratory rate of 40. Based on the symptoms and findings, what is the MOST appropriate treatment? A. Use a bronchodilator to decrease respiratory acidosis B. Have the patient to breathe into a re-breathing mask to decrease respiratory alkalosis C. Paced breathing to decrease respiratory alkalosis D. Use a mechanical ventilator to decrease respiratory acidosis

B. Have the patient to breathe into a re-breathing mask to decrease respiratory alkalosis Anxiety is correlated with respiratory alkalosis. By breathing into a re-breathing mask you are increasing CO2 and decreasing the pH.

A 59-year-old teacher who is right hand dominant complains of stiffness in the neck and right hand (especially in the 1st CMC) upon waking in the morning. It subsides within thirty minutes as she moves around the house and performs some household chores. There is no associated history of other systemic symptoms, although she has noticed some bumps on her DIP joint. The presence of outgrowths can be documented as which of the following? A. Neurofibromatosis B. Heberden's nodules C. Rheumatoid nodules D. Bouchard's nodules

B. Heberden's nodules The patient's age and symptoms indicate OA. Nodules on the DIP are called Heberden's nodules and nodules on the PIP are called Bouchard nodules. Absence of systemic systems and morning stiffness lasting less than sixty minutes rule out rheumatoid arthritis, and thus rule out rheumatoid nodules. With neurofibromatosis, the nodules would not be restricted to the DIP joints.

During exams week, a physical therapy student observes the posture of her classmate. She notices that the classmate has slouched/rounded shoulders. Which of the following characterizes rounded shoulder posture? A. Stretched pectoralis minor B. Limited shoulder external rotation C. Increased shoulder abduction D. Inability to extend the elbow

B. Limited shoulder external rotation A forward head/slouched posture significantly alters scapular kinematics. There is decrease in posterior tilting and external rotation of the scapula and the scapula becomes protracted. This scapular posture also changes the posture of the humerus in the glenoid, which assumes a relatively abducted and internally rotated position with respect to the scapula. The GH internal rotators may become less flexible, and external rotators may weaken, resulting in limited shoulder external rotation.

All of the following are signs and symptoms of a later manifestation of cystic fibrosis EXCEPT: A. Anorexia B. Meconium ileus C. Clubbing D. Diarrhea

B. Meconium ileus Meconium Ileus is an early manifestation of cystic fibrosis as it seen in infants. Anorexia, clubbing and diarrhea are late manifestations.

A physical therapist works with a patient who experiences hyperfunction of the parathyroid glands secondary to a tumor. This condition would MOST likely contribute to the development of which of the following disorders? A. Cardiac arrhythmias B. Osteopenia C. Muscle spasms D. Obesity

B. Osteopenia Hyperparathyroidism results in the demineralization of bones due to the increased secretion of parathyroid hormone (which causes the release of calcium by the bones and a subsequent accumulation of calcium in the bloodstream) and subsequent loss of bone density and strength (i.e., osteopenia).

A 32-year-old male patient is referred to PT for gait training post right tibial plateau fracture. The patient is currently on non-weight bearing precautions for the right lower extremity. Which of the following is the MOST appropriate to be used to train this patient? A. Bilateral canes B. Parallel bar C. Hemiwalker D. Single crutch

B. Parallel bar Parallel bars are the only option that allow for non-weight bearing

A physical therapist reviews the medical record of a patient who has experienced recurrent angina. A recent entry indicated that an exercise stress test ordered by the physician was positive. What is the MOST accurate interpretation of this finding? A. Presence of balanced oxygen demand and supply B. Presence of ischemia C. Presence of normal vital signs D. Presence of cardiac arrhythmias

B. Presence of ischemia An exercise stress test is used to determine the presence of ischemia and evaluate the overall functional capacity of the patient. The patient is typically monitored through a 12-lead ECG and vital signs. Determination of the presence of ischemia is the goal of the exercise stress test.

A 21-year-old female volleyball player has limited ROM and pain in the right shoulder. The PT plans an intervention program but the patient refuses any treatment. Patient agrees that without intervention the symptoms might worsen. Which is the MOST appropriate PT response? A. Call the coach of the team and report that the injury might affect the player's performance B. Respect the player's decision to not use physical therapy services C. Start the treatment as the patient will most likely join in once she sees the benefits of the treatment D. Refer the patient to another therapist who is a sports specialist

B. Respect the player's decision to not use physical therapy services You can always educate them but you HAVE TO respect their decision. Now, if this was a child under 18 years it is up to the legal guardian. The child can say they don't want to but if the parent says to go to PT you have to listen to them.

A patient comes into an outpatient clinic with symptoms of anxiety, hyperventilation, syncope, and cramping of the wrist. Upon PT evaluation, the therapist finds increased respiratory rate and a decrease in blood pressure. Based on the symptoms and findings, what condition does the patient present with and what is the MOST appropriate treatment? A. Respiratory acidosis and bronchodilator B. Respiratory alkalosis and having the patient breathe into a paper bag C. Bronchitis and paced breathing D. Cystic fibrosis and endurance exercises

B. Respiratory alkalosis and having the patient breathe into a paper bag The patient is experiencing signs of respiratory alkalosis. Paper bag is a rebreathing device that allows the patient to inhale and rebreathe the exhaled CO2.

Which of the following conditions is MOST likely to cause a left tracheal deviation? A. Right pleural fibrosis B. Right pleural effusion C. Right lobar atelectasis D. Right pneumonectomy

B. Right pleural effusion A tracheal shift occurs away from the side of the abnormality when there is an increase in volume. A pleural effusion will cause an increase in volume. Therefore, a right pleural effusion will cause a left tracheal shift.

A patient develops a stage 2 pressure injury over the sacrum and is referred to physical therapy for wound care. Which of the following is the MOST appropriate initial application to clean the wound? A. Povidone-iodine solution B. Sterile normal saline C. Zinc oxide cream D. Nitrofurazone solution

B. Sterile normal saline The first step when you're in the clinic is almost always to use sterile normal saline to clean the wound! A stage 2 pressure injury is when the skin breaks open, the first two layers of skin are affected, superficial in nature.

A physical therapist observing a patient complete a leg curl exercise notices two prominent tendons visible on the posterior surface of the patient's knee. The visible medial and lateral tendons are MOST likely associated with which muscles? A. Semimembranosus and semitendinosus B. Semitendinosus and biceps femoris C. Popliteus and semitendinosus D. Semimembranosus and biceps femoris

B. Semiteninosus and biceps femoris The semimembranosus and semitendinosus muscles are both on the medial side so you can rule out A. While the semimembranosus is on the medial side and the biceps femoris on the lateral you could assume this is a valid answer; however, the semimembranosus is not visible with a leg curl so you can rule out D. The popliteus is a very deep structure and will not appear during a leg curl either so you can also rule out C.

A 12-year-old patient who has spastic diplegic cerebral palsy has full passive range of motion of the lower extremities, but demonstrates crouching with hip and knee flexion angles of 20° each in standing position. Which of the following interventions is BEST to achieve sustained improvements in lower extremity alignment during walking? A. Stretching of the iliopsoas B. Strengthening of the quadriceps and gluteals C. Stretching of the hamstrings and gastrocnemius D. Strengthening of the hamstrings and gastrocnemius

B. Strengthening of the quadriceps and gluteals This patient has full ROM so you don't need to stretch! Strength training has been shown to improve gait and muscle performance in patients who have cerebral palsy. The physical therapist should seek to create a balance of muscle activity across a joint. In this case, addressing quadriceps and gluteal muscles will be beneficial for improving knee and hip extension by counteracting the forces potentiating flexion.

Which of the following contain the cerebrospinal fluid and major arteries? A. Dura mater B. Subarachnoid space C. Pia mater D. Arachnoid

B. Subarachnoid space Meninges are just coverings so they cannot house anything. You need space to house arteries so you can rule out A, C, and D.

A patient reports to physical therapy after being fit for an upper extremity splint. Which type of splint would MOST likely be prescribed for a patient that demonstrates a positive Finkelstein's test? A. Ulnar gutter B. Thumb spica C. Radial gutter D. Dorsal forearm

B. Thumb spica To perform Finkelstein's test the patient is asked to make a fist with the thumb tucked underneath the fingers. The therapist stabilizes the patient's forearm and ulnarly deviates the wrist. A positive test is indicated by pain over the abductor pollicis longus and extensor pollicis brevis tendons and may be indicative of tenosynovitis in the thumb (i.e., de Quervain's disease). A thumb spica splint is a rigid splint that covers the radial side of the forearm and hand, as well as the thumb. The splint may cover the entire thumb or may stop at the proximal phalanx of the thumb, and thus allow for IP joint motion. This type of splint is used to immobilize the wrist and MCP joint of the thumb and is commonly used for treating gamekeeper's thumb, scaphoid fx, first metacarpal fx, and de Quervain's disease.

A 30-year-old female patient is being seen at an outpatient physical therapy clinic. She is a software engineer at a start-up company and uses public transit for her commute every day. The patient reports that she has difficulty maintaining her balance, especially when she is in crowded places at night. What aspect of balance should the physical therapist examine FIRST in this patient? A. Conscious proprioception B. Vestibular integrity C. Somatosensory integrity D. Visual integrity

B. Vestibular integrity When the patient is in a crowded environment at night or in public transport, there is visual conflict and unstable surface. In order to maintain balance she will depend on her vestibular system. Thus, if the patient is complaining of loss of balance, the vestibular integrity must be checked.

A physical therapist performs the manual muscle test shown in the photograph and notes weakness of the musculature. Which of the following nerve roots is MOST likely involved? A. L1 B. L3 C. L5 D. S1

C. L5 The therapist in the photograph is testing the tibialis anterior (dorsiflexion), which is innervated by the deep fibular (peroneal) nerve (L4- L5).

A PT examines the output from a single lead electrocardiogram of a patient in an outpatient clinic. The ECG strip is shown in picture below. The PT should determine the heart rate of the patient as? A. 110 beats per minute B. 70 beats per minute C. 100 beats per minute D. 50 beats per minute

C. 100 beats per minute A standard ECG strip will consist of 30 boxes. On the NPTE they will most likely be standard. You simply count the R waves seen and multiply by 10 to get your HR.

A 47-year-old mine-worker was admitted for burn injury. The PT observes that the anterior surface of the patient's trunk and both anterior lower extremities were affected. Which of the following BEST represents the percentage of body surface area involved? A. 18% B. 27% C. 36% D. 45%

C. 36% For an adult: 9% for anterior LE 18% for anterior trunk

A PT is ambulating a 75-year-old male who is in recovery from a prostatectomy procedure. The patient complains of palpitations, SOB, and fatigue. What should be the interpretation and immediate action according to the ECG strip shown below? A. Ventricular fibrillation; call for a defibrillator B. Premature ventricular contractions; take him to his bed and monitor for changes in ECG C. Atrial fibrillation; stop exercise and report to physician D. 3rd degree heart block; activate emergency

C. Atrial fibrillation; stop exercise and report to physician

A 45-year-old female patient is being evaluated in an outpatient clinic by a physical therapist. The PT performs the test shown in the picture. A positive result of the test likely indicates the presence of which of the following conditions? A. Radial collateral ligament tear B. Carpal tunnel syndrome C. De Quervain disease D. Restricted ulnar deviation

C. De Quervain disease This is an image of the Finkelstein's test.

A patient recovering from TBI is functioning at stage IV on the Rancho Los Amigos Level of Cognitive Functioning scale. During the therapist's initial examination, the patient becomes agitated and tries to bite the therapist. The BEST course of action is to: A. Postpone the examination until later in the day when the patient calms down B. Postpone the examination for one week and then try again C. Document the behaviors and engage in a calming activity D. Restructure the formal exam so the therapist can complete it in three very short sessions

C. Document the behaviors and engage in a calming activity Stage IV: confused/agitated

A patient is seen after an uncomplicated vaginal delivery of her third child. During the treatment the patient begins to complain of a headache and visual disturbances and then suddenly has a seizure. Which condition is MOST likely? A. Preeclampsia B. Gestational diabetes C. Eclampsia D. Ectopic pregnancy

C. Eclampsia Eclampsia is the only option that can occur after delivery Eclampsia: seizures that occur during a woman's pregnancy or shortly after giving birth due to hypertension

A physical therapist works with a patient diagnosed with congestive heart failure who presents with dyspnea during ambulation. The patient has an ejection fraction of 40%. Which of the following interventions would be the MOST appropriate? A. Instruction in pursed-lip breathing B. Progressive resistive exercises C. Education on energy conservation D. Instruction in diaphragmatic breathing

C. Education on energy conservation The ejection fraction is a measure of left ventricular contractility. It is determined by dividing stroke volume by left ventricular end-diastolic volume. Normal ejection fraction is 55-70%. Anything less than 55% of the blood pumped out of the ventricles with each heartbeat is abnormal and indicates impairment in left ventricular function. Ejection fraction is decreased in patients with left-sided congestive heart failure since the left ventricle is unable to maintain a normal cardiac output. The primary goals of treating a patient with congestive heart failure include improving exercise tolerance and increasing knowledge of the disease process. Since the heart is unable to meet the metabolic demands of the body, pacing and energy conservation techniques are necessary for the patient to improve their tolerance for activities of daily living and potentially exercise.

As a result of complications from habitually leaning forward on crutches, a patient will MOST likely have weakness of which of the following muscles? A. Elbow extensors and forearm pronators B. Scapular retractors and forearm pronators C. Elbow extensors and wrist extensors D. Scapular retractors and wrist extensors

C. Elbow extensors and wrist extensors Leaning on the axillary bar of the axillary crutches causes pressure at the radial groove of the humerus, potentially damaging the radial nerve and adjacent vascular structures in the axilla (O'Sullivan). The radial nerve innervates the triceps and wrist extensors.

While under the oversight of a PT, a patient performs a neurodynamic mobility exercise to reduce neural tension in the upper extremity. Following the therapy session, the patient calls the clinic to report increased neural symptoms with prolonged paresthesias. The MOST appropriate response by the physical therapist during the next session would be to: A. Explain that this sensation is normal and continue working in the same range B. Refer the patient back to the orthopedic physician C. Explain to the patient that this sensation is abnormal and decrease the range used D. Re-evaluate the patient and determine if there is a different source of their pain

C. Explain to the patient that this sensation is abnormal and decrease the range used When performing neuromobilization, increased symptoms should be accounted for by decreasing the range that the exercises are performed in. The therapist would aggravate the patient's symptoms if the same range was utilized. No further medical action is necessary at this time.

A family physician refers a patient to physical therapy for treatment of chronic low back pain. The patient is currently receiving treatment from a massage therapist for the same problem. Which of the following actions is MOST appropriate for the physical therapist? A. Ask the patient to discontinue the massage therapy B. Treat the patient on days the patient is not seen by the massage therapist C. Gain permission from the patient to contact the massage therapist to discuss the plan of care D. Discontinue the patient's physical therapy

C. Gain permission from the patient to contact the massage therapist to discuss the plan of care Obtaining the patient's permission to contact the massage therapist allows communication between healthcare providers and provides the most appropriate treatment for the patient.

You are performing a gait analysis of a patient whose primary complaint is right knee pain. The patient has a history of a right meniscectomy several years ago with minimal pain relief. Upon objective examination, the PT notes excessive dorsiflexion, eversion, and abduction of the right foot. Which of the following biomechanical adaptations is MOST likely to be true? A. Coxa valga B. Genu varum C. Genu valgum D. Genu recurvatum

C. Genu valgum Excessive dorsiflexion, eversion and abduction are the triplanar components of pronation at the ankle. Patients with ankle pronation are likely to have genu valgum and coxa vara.

A patient sustained a nerve injury just below the inguinal ligament. Which of the following gait deviations is the patient MOST likely to demonstrate? A. Increased posterior lean of the trunk in midstance B. Increased medial (internal) rotation of the thigh during midswing C. Hyperextension of the knee during stance phase D. Excessive ankle plantar flexion during foot flat (loading response)

C. Hyperextension of the knee during stance phase The nerve that lies below the inguinal ligament is the femoral nerve. The femoral nerve provides innervation for the quadriceps musculature. When the quadriceps are weak, there will be a compensatory motion of the femur by action of the gluteus musculature to pull the femur posteriorly. This will result in the knee ground reaction force being in front of the knee axis, thus providing an extensor moment.

A physical therapist examines the breath sounds of a patient diagnosed with pulmonary disease. The therapist identifies crackles during both inspiration and expiration. This finding is MOST representative of which of the following conditions? A. Pleural effusion B. Pulmonary fibrosis C. Impaired secretion clearance D. Localized stenosis

C. Impaired secretion clearance Crackles or rales are abnormal breath sounds heard during auscultation of the lungs with a stethoscope. Crackles are extra sounds caused by the "popping open" of small airways blocked by secretions or fluid and may be heard during both the inspiratory and expiratory phases of the breathing cycle.

A patient demonstrated painful ROM in horizontal abduction movement during ROM examination of shoulder. Which of the following joint mobilization techniques is MOST appropriate for the patient? A. Large amplitude oscillations performed at the beginning of the ROM in an anterior-inferior direction B. Small amplitude oscillations into tissue resistance up to the limit of available motion in a posterior-superior direction C. Large amplitude oscillations within the available ROM in an anterior-inferior direction D. Small amplitude oscillations into tissue resistance at the limit of available joint motion in a posterior-inferior direction

C. Large amplitude oscillations within the available ROM in an anterior-inferior direction With horizontal abduction you are rolling posteriorly so you must glide anteriorly so that rules out B and D. Now you must consider the different grades of mobilizations. You know that you will use either grade I or II because the pt has pain. Grade I is small amplitude so you know it must be grade II. C describes a grade II mobilization.

A patient can move her fingers to grasp objects but is unable to use her touch sensation to identify objects such as keys and pen placed in her hands, bilaterally. Which of the following is LEAST likely to be intact? A. Bilateral dorsal column medial lemniscus and corticospinal tract lesion B. Lesion of the right cerebral hemisphere C. Lesion of bilateral dorsal column medial lemniscus D. Lesion of bilateral radial nerves

C. Lesion of bilateral dorsal column medial lemniscus The patient is dealing with sensation problems so it will be an ascending/afferent pathway involved. You can automatically rule out A because the corticospinal tract is a MOTOR tract. We also know that there is bilateral involvement so you can rule out B. You can assume this would involve the fasciculus cuneatus (UE involvement) of the dorsal column medial lemniscus. Dorsal column medial lemniscus system: sensory pathway; fine touch, discriminating touch, pressure, vibration, proprioception

The PT is trying to provide the appropriate wheelchair measurements. Which of the following options provide the correct measurement for seat depth? A. Measure the length from posterior buttock to posterior aspect of popliteal fossa and add 2 inches to the measurement B. Measure the total area of both the hips and add 2 inches to the measurement C. Measure the length from posterior buttock to posterior of popliteal fossa and subtract 2 inches from the measurement D. Measure the trunk length and subtract it from lower leg length and add 2 inches to the measurement

C. Measure the length from posterior buttock to posterior of popliteal fossa and subtract 2 inches from the measurement

A patient uses patient-controlled analgesia with a lockout interval following an inpatient surgical procedure. Which medication would be MOST consistent with this delivery model? A. Atorvastatin (Lipitor) B. Baclofen (Lioresal) C. Meperidine (Demerol) D. Methotrexate (Trexall)

C. Meperidine (Demerol) Meperidine (Demerol) is a commonly used opioid agent. This class of pharmacological agents provides analgesia for acute severe pain management. The medication stimulates opioid receptors within the CNS to prevent pain impulses from reaching their destination. The potential serious side effects and the potential for physical dependence result in the medication often being administered with a lockout interval. A lockout interval refers to the period of time in which a patient-controlled analgesia system will not allow the patient to receive medication.

A physical therapist is teaching a patient an exercise in standing with the patient's right side of the body next to the wall and the patient's right arm resting on the wall for support. The patient is told to place the right leg behind the left in the extended, adducted and externally rotated position with the pelvis shifted to the right and a slight trunk lean to the left. Which of the following tests was MOST LIKELY positive for the therapist to recommend this intervention? A. Ely's test on the right B. Phelp's test on the left C. Ober's test on the right D. Piriformis test on the left

C. Ober's test on the right The intervention described is a stretch of the ITB/tensor fascia lata on the right. Ober's test is used to assess the tightness/contracture of the tensor fascia lata

The patient arrives to a PT clinic looking pale and is sweating profusely. His pulse is weak and his breath is shallow and rapid. The PT suspects heat exhaustion. Which of the following is NOT an appropriate response? A. Offer the patient some water B. Offer ice pack for forehead and the neck C. Offer the patient some salt tablets for electrolyte balance D. Ask the patient to remove any outer layer of clothing

C. Offer the patient some salt tablets for electrolyte balance Increased salt causes dehydration so you would not want to give this

Which of the following interventions would be the BEST choice to remove wound exudate and loose debris from a sacral wound that exhibits tunneling? A. Sharp debridement B. Wet-to-dry dressings C. Pulsatile lavage with suction D. Acetic acid wound cleanser

C. Pulsatile lavage with suction Pulsatile lavage with suction combines wound irrigation with suction and removes the irrigation fluid, wound exudate, and loose debris. It has been found to be advantageous over other interventions since it uses less water and requires less staff support, less cleanup, and less treatment time. It has been shown to increase healing time by rapid removal of contaminants, and it can be used to treat tunneling wounds using special cannula tips.

A patient has symptoms of muscle aches, cramps, soreness, and pain in back and extremities. Creatine kinase level is 10 times more than normal and the urine is tea colored. Which of the following is MOST consistent with the clinical findings? A. Hyponatremia B. Hypokalemia C. Rhabdomyolysis D. Myasthenia gravis

C. Rhabdomyolysis Rhabdomyolysis is a potentially fatal condition in which myoglobin and other muscle tissue contents are released into the bloodstream as a result of muscle tissue disintegration. This could occur with acute trauma, severe burns, overexertion, from alcohol abuse or alcohol poisoning, or with statins. It leads to muscle aches, cramps, weakness and soreness. Dark color of urine is due to liver failure. Normal sodium: 135-145 Normal potassium: 3.5-5.5

A physical therapist is assessing trunk strength of a healthy patient who requested a weight loss exercise program. The patient is lying supine and lifts his shoulders off the table to bring his right elbow towards his left knee. This movement is testing what muscle(s)? A. Right internal oblique and left external oblique B. Pectoralis major C. Right external oblique and left internal oblique D. Rectus abdominis

C. Right external oblique and left internal oblique During rotation to left side, internal oblique of side towards which the movement occurs (left side) work and the external oblique of the opposite side work (right side).

A patient has thin, bloody drainage from an abdominal surgical wound. This finding should be classified as which of the following types of drainage? A. Serous B. Chylous C. Sanguineous D. Purulent

C. Sanguineous Serous: clear, watery Chylous: milky, white Sanguineous: thin, bloody Purulent: thick

A PT is evaluating a 20-year-old male patient with a history of midback pain. The MRI reveals anterior wedging of the T7-9 vertebral bodies. What is the MOST appropriate diagnosis? A. Ankylosing spondylitis B. Osteogenesis imperfecta C. Scheuermann's disease D. Spondyloepiphyseal dysplasia

C. Scheuermann's disease "Juvenile kyphosis"; expect this in the thoracic spine; since you are flexing so much on the upper thoracic spine you'll get a narrowing and anterior wedging of the vertebral discs in the T7-9 area. Scheuermann's disease is diagnosed with an MRI so that helps to narrow down options. Ankylosing spondylitis- spine fuses, have a slight flexed posture Osteogenesis imperfecta- the formation of the bone is imperfect, easily fractured Spondyloepiphyseal dysplasia- the growth plate is not growing properly which will result in a short stature/dwarfism ALL OF THESE ARE DIAGNOSED WITH AN X-RAY

A 21-year-old female field hockey player has a history of several right leg hamstring strains. She has been performing frequent stretching exercises for the right hamstrings with minimal changes in ROM observed. What modality can the PT utilize to BEST improve her ROM and facilitate stretching? A. Ultrasound B. Iontophoresis C. Short wave diathermy D. TENS

C. Short wave diathermy SWD is a deep heating modality and can be used to cover a large area like the hamstrings unlike ultrasound, whose heating is limited by the effective radiating area of the sound head. SWD helps in increasing soft tissue extensibility which can facilitate stretching and help improve the ROM.

A PT is evaluating a patient who had a TBI with lesion in the left parietal lobe. On asking the patient to place their arms on their laps in the sitting position, the patient doesn't comply. When asked to point to their toes, the patient seems confused. Which of the following is the MOST appropriate way to document these findings? A. Right-left discrimination B. Anosognosia C. Somatognosia D. Unilateral neglect

C. Somatognosia You know that a lesion in the parietal lobe affects sensory and somatosensory so that should automatically push you towards C. Anosognosia- a lack of ability to perceive the realities of one's own condition; denies a diagnosis Somatognosia- unaware of their body parts and the relationship between their body parts. Patients with this deficit may display difficulty following instructions that require distinguishing body parts.

A 45-year-old obese male (BMI 33 kg/m^2) with type 2 DM is working out on the treadmill in the PT gym. While exercising, the patient suddenly develops light-headedness, dizziness, and instability. The MOST appropriate action is: A. Stop the treadmill and call the primary care physician B. Keep the treadmill moving and have the patient drink orange juice C. Stop the treadmill and have the nurse check blood glucose levels D. Slow the treadmill speed and have the patient drink cold water

C. Stop the treadmill and have the nurse check blood glucose levels You need to always check their blood glucose levels before doing anything. It is also dangerous having a dizzy and unstable patient on a treadmill.

Which of the following interventions is MOST appropriate to improve stability in long sitting for an individual with a complete T4 spinal cord injury? A. Stretch the gluteus maximus to 90 degrees of hip flexion B. Strengthen the abdominal muscles to grade Fair (3/5) C. Stretch the hamstrings to 110 degrees of straight leg raising D. Strengthen the erector spinae muscles to grade Good (4/5)

C. Stretch the hamstrings to 110 degrees of straight leg raising Stretching of the hamstring muscles prevents overstretching of the back during long sitting. Passive low back muscle tightness is important to develop for passive trunk stability. Straight leg raises less than 100° to 110° in long sitting put a passive pull on the pelvis, resulting in posterior pelvic tilt and stretching of the low back.

A PT student is assessing a patient with a chief complaint regarding the inability to completely raise the right arm. The student internally rotates the arm, slightly abducts, and brings the arm to 90 degrees of flexion. The student then asks the patient to resist the downward directed force on their arm. Which of the following responses would be MOST LIKELY if the patient is graded poorly on this test? A. Long thoracic nerve injury B. Radial nerve injury C. Suprascapular nerve injury D. Musculocutaneous nerve injury

C. Suprascapular nerve injury The student is assessing the supraspinatus muscle by performing the Empty Can Test. The test is considered to be positive if there is weakness or pain while resisting flexion. A positive finding is associated with a tear of the supraspinatus tendon or muscle, or injury to the suprascapular nerve.

A PT is called to assess a patient with traumatic brain injury. The therapist documents that the patient has decerebrate posturing pattern. Which of the following is MOST likely to be seen in this patient? A. Sustained contraction and posturing of the trunk and lower limbs in extension, and the upper limbs in flexion, fists clenched B. Sustained contraction and posturing of the trunk and lower limbs in flexion, and the upper limbs in extension, fists clenched C. Sustained contraction and posturing of the trunk and limbs in a position of full extension D. Sustained contraction and posturing of the trunk and limbs in a position of full flexion

C. Sustained contraction and posturing of the trunk and limbs in a position of full extension DEcErebratE: everything in extension Decorticate: LE's in extension, UE's in flexion

A patient who is not able to walk has developed an ischial tuberosity pressure injury. The patient is able to perform independent intermittent catheterization and demonstrates independence in bed-to-chair transfers. Which of the following factors has MOST likely contributed to the formation of the pressure injury? A. Friction B. Infection C. Tissue loading D. Tissue maceration

C. Tissue loading In this patient, prolonged sitting due to the inability to walk leads to tissue loading and risk for skin breakdown on the ischial tuberosities.

A 58-year-old patient has been diagnosed with Parkinson's disease. The patient leans forward and displays a festinating gait. What would be the MOST appropriate intervention? A. Increase cadence using a metronome B. Increase stride length using laser markers C. Use a toe wedge to displace the center of gravity backwards D. Use a heel wedge to displace the center of gravity forward

C. Use a toe wedge to displace the center of gravity backwards A festinating gait can sometimes be alleviated by the addition of modified heel or shoe wedges. A flat heel or toe wedge may slow down a propulsive gait.

The following EKG pictured below MOST LIKELY demonstrates which cardiac condition? A. Atrial fibrillation B. This is a normal electrocardiogram, there is no pathology present C. Ventricular tachycardia D. Supraventricular tachycardia

C. Ventricular tachycardia

A 22 year old patient complains of a gradual onset of thoracic and sacroiliac pain. Lumbar ROM is limited to 50%. Patient reports his stiffness is worse in the morning and gradually improves. He recently started a new job at Comcast and has been sitting for long periods of time. Which of the following would you MOST likely suspect? A. Thoracic hypomobility B. Pectoral muscle adaptive shortening C. Spinal tumor D. Ankylosing spondylitis

D. Ankylosing spondylitis Ankylosing spondylitis is a rheumatic disease characterized by chronic inflammation of ligaments in the lumbar spine, sacroiliac joint, thoracic spine, and shoulders that usually occur in the teenage years and 20's. Radiographs will show a bamboo spine.

A child with a unilateral hip disarticulation using a prosthesis works on advanced gait training activities. Which of the following activities would be the MOST difficult for the patient to perform? A. Rising from a wheelchair B. Ascending stairs with a handrail C. Descending stairs with a handrail D. Ascending a curb

D. Ascending a curb A child with a hip disarticulation would have the greatest difficulty ascending a curb during prosthetic training since there are no external supports to assist with the activity. Going up is harder than going down for a patient with hip disarticulation because they do not need to overcome gravity!

Occupational Safety and Health Administration (OSHA) is responsible for which of the following? A. Determining how much compensation the therapist should get early B. Assuring minimum standards are met at the hospital to maintain accreditation and prevent negligence in patient care C. Assuring minimum standards are met by the rehabilitation clinics to maintain accreditation and safety of patient D. Ensuring adequate steps taken to prevent exposure to harmful radiation to employees at a X-ray center

D. Ensuring adequate steps taken to prevent exposure to harmful radiation to employees at a X-ray center OSHA is concerned about the safety of the EMPLOYEES

A physical therapist is observing a 67-year-old female patient walk and immediately notes a forward trunk lean. A LIKELY contributing cause for this deviation is: A. Excessive ankle dorsiflexion B. Quadriceps spasticity C. Ankle plantar flexion weakness D. Ankle plantar flexion contracture

D. Ankle plantar flexion contracture Think about what women look like when they wear heels (ankle PF'd). They often have hyperextended knees and a forward lean.

A researcher is collecting ROM data on volleyball players during shoulder abduction motion. The results of the tests are BEST categorized as which of the following types of data? A. Interval data B. Ordinal data C. Nominal data D. Ratio data

D. Ratio data Ordinal and nominal data are used for qualitative data. ROM is always ratio data because there is a true zero- ROM can be 0 degrees. Interval data has no true zero.

A patient presents to the clinic with chief complaints of loss of balance. She has had 3 falls in the last 90 days. She described herself as pretty active but now she is afraid of falling. On probing further, the patient mentioned that all the falls happened at night when the patient would either wake up to go to the restroom or go to the other room to read when not able to fall asleep. During the assessment of balance using the CTSIB, in which of the conditions is the patient MOST likely expected to lose her balance? A. Conditions 2, 4 B. Conditions 4, 5 C. Conditions 1, 5 D. Conditions 2, 5

D. Conditions 2, 5 The patient does not fall when she has visual input but when it is taken away she falls. This means she is DEPENDENT on vision. So she would have difficulty with conditions that take away vision. Conditions 2, 3, 5, and 6 take away vision.

A single 22-year-old woman who is 3 months pregnant arrives at a therapist's private practice complaining of shoulder and leg pain. She has a black eye and some bruising at the wrists. The BEST course of action for the therapist is: A. Administer massage for bruising, TENS, and ice modalities for pain, as indicated by the examination findings B. Direct the patient to the nearest ambulatory care center for physician evaluation C. Refuse to examine the patient and send her to the nearest emergency room D. Examine the patient, and if abuse is suspected, report the findings to the appropriate authorities

D. Examine the patient, and if abuse is suspected, report the findings to the appropriate authorities If abuse was done and you ignore it you are not ensuring the safety of your patient

A physical therapist is observing a 50-year-old female patient in quiet standing and observes that the vertical ground reaction force is passing anterior to the knee joint in the sagittal plane. The ground reaction force is exerting which moment? A. Internal flexor moment B. External flexor moment C. Internal extensor moment D. External extensor moment

D. External extensor moment During ambulation, forces are applied under the surface of the foot every time a person takes a step. The forces applied to the ground by the foot are called foot forces. Conversely, the forces applied to the foot by the ground are called ground reaction forces. During gait, vertical ground reaction forces are created by a combination of gravity, body weight, and the firmness of the ground. If the GRF line is anterior to the knee, it produces an external knee extensor moment and if the GRF line is posterior to the knee, it produces an external knee flexor moment.

Which of the following combinations of orthotic and anatomical causes will MOST likely lead to the same gait deviation? A. Excessive height of medial upright of KAFO and gluteus maximus weakness B. Excessive height of lateral upright of KAFO and abduction contracture C. Inadequate dorsiflexion stop and pes equinus deformity D. Inadequate dorsiflexion assist and weak hip flexors

D. Inadequate dorsiflexion assist and weak hip flexors Both of these will lead to toe drag

A patient being treated for imbalance complains of sudden onset of mild hearing loss on the left side. Weber's test findings show lateralized to the right ear. Rinne test was consistent with air conduction greater than bone conduction on both sides. Which of the following is the MOST likely? A. Right side sensorineural hearing loss B. Left side conduction hearing loss C. Right side conduction hearing loss D. Left side sensorineural hearing loss

D. Left side sensorineural hearing loss The hearing loss is noted in the left ear. When the Weber's test is performed they hear it more in the unaffected ear. This would mean they have sensorineural hearing loss because when the loss is sensorineural the affected ear will hear nothing. Also, in order for conduction hearing loss, bone conduction needs to be greater than air conduction on the Rinne test. With this you can rule out B and C. Since the hearing loss is in the left ear you can rule out A.

When treating a geriatric patient with osteoporosis, which of the following will be MOST important to include in a treatment plan to improve bone density? A. Treadmill with high incline B. Lumbar flexion resistance training C. Seated rowing machine D. Leg press resistance training

D. Leg press resistance training Muscle contraction (strengthening exercises, resistance training) and mechanical loading (weight bearing) deform bone which stimulates osteoblastic activity and improves BMD. Since, leg press causes weight bearing it can cause bone formation (Wolf's Law). Seated row machine will not allow weight bearing, hence not beneficial. Treadmill with high incline and lumbar flexion should be avoided in an osteoporotic patient to prevent vertebral compression fracture.

A physical therapist working in a school system develops long-term goals as part of an Individualized Educational Plan for a child with Down syndrome. Which of the following timeframes is the MOST appropriate to attain these goals? A. One month B. Four months C. Six months D. One year

D. One year An Individualized Educational Plan (IEP) is designed for any school-aged child that requires therapy services. Long-term goals are based on a one-year plan.

A 55-year-old female patient is being seen by a PT 5 days status post unilateral mastectomy. The PT notes that the patient is unable to actively adduct the shoulder in the horizontal plane. Which of the following impairments is the MOST LIKELY origin of the patient's presentation? A. Stiffness secondary to lymphedema B. Pectoralis minor tightness C. Subscapularis weakness D. Pectoralis major weakness

D. Pectoralis major weakness Pectoralis major/ minor are muscles responsible for horizontal adduction. The pectoral muscles might have been damaged at the time of mastectomy causing weakness. Stiffness will affect all the movements and not only adduction. Subscapularis weakness leads to decreased internal rotation and adduction but is less affected with mastectomy.

A 65-year-old patient presents to the clinic post CVA. The PT review the angiogram and identifies a blockage of the upper division of the left MCA. Which of the following conditions is the therapist MOST likely to note upon performing an evaluation? A. Weakness in the left upper extremity B. Presence of non-expressive aphasia C. Left homonymous hemianopia D. Presence of expressive aphasia

D. Presence of expressive aphasia If you have a blockage on the left side it will affect your right side so you can automatically rule out A and C. The upper division of the middle cerebral artery supplies the frontal lobe which is where the Broca's region is. Broca's aphasia is expressive aphasia so the answer is D.

After gait training a patient with a new below knee prosthesis, you notice redness along the patellar tendon and medial tibial flare. This would indicate: A. The socket is too small and the residual limb is not seated properly B. The socket is too large and pistoning is occurring C. Improper weight distribution during stance D. Pressure tolerant weight bearing during stance

D. Pressure tolerant weight bearing during stance Patellar tendon and medial tibial flare are pressure tolerant areas so a little redness is expected ESPECIALLY if it is a new prosthesis! You will only refer to a physician if they have redness in a pressure SENSITIVE area! Pressure tolerant areas --> M-DPT ( masters/doctorate of physical therapy): medial tibia, distal end, patellar tendon, tibial and fibular shaft If the socket is too large the leg will have too much space and will be sliding in and out of the socket (pistoning). If the socket is too small you simply cannot get your leg into it. Neither A or B will cause redness.

A 27-year-old girl had a flip phone for the past 4 years and finally upgraded to an iPhone 8. She has been on her new phone constantly and is now having difficulty swiping upwards with her thumb. She explains this to her friend who is a PT student. To improve her thumb's range of motion without difficulty, what is the BEST mobilization or glide the PT student should perform? A. Inferior glide B. Superior glide C. Ulnar glide D. Radial glide

D. Radial glide Radial glide is used to improve thumb extension. At the CMC joint, the trapezium is convex and the proximal metacarpal is concave for flexion and extension. Flexion and Extension are frontal plane movements of the thumb and concave metacarpal moves over the convex trapezium so glide will be in the same direction. (Concave-convex rule)

A physical therapist is reviewing the laboratory report of a patient who received a diagnosis of pneumonia 2 weeks ago. The patient's white blood cell count is currently 9,000 cells/mm3. Which of the following conditions does this value indicate for the patient? A. Anemia B. Development of leukocytosis C. Immunosuppression D. Resolution of the pneumonia infection

D. Resolution of the pneumonia infection The patient's white blood cell count is within the normal range of 4,500-11,000/mm3, so the infection has resolved.

When performing active cervical rotation range of motion, a patient moves as shown in the photographs. Translation testing of the mid-cervical spine reveals no restrictions to the right and moderate restrictions to the left. Which of the following cervical motions will MOST likely be restricted in the patient? A. Forward bending B. Backward bending C. Side bending to the left D. Side bending to the right

D. Side bending to the right Cervical rotation follows the same arthrokinematic motions as side bending; therefore, side bending to the right would be limited. Side bending mechanics include a superoanterior glide of the left superior facet and a posteroinferior glide of the right facet, which would be restricted with decreased cervical motion and translation of the mid cervical spine to the left.

A 13-year-old patient reports groin pain that radiates into the anterior thigh. The patient exhibits an antalgic pattern, and the involved lower extremity is maintained in a laterally (externally) rotated position. Which of the following conditions is MOST likely present? A. Developmental hip dysplasia B. Osgood-Schlatter disorder C. Legg-Calve-Perthes disease D. Slipped capital femoral epiphysis

D. Slipped capital femoral epiphysis Slipped capital femoral epiphysis generally occurs in adolescents. Symptoms include antalgic gait and pain in the groin, knee, or medial thigh. This disorder is more likely to present with antalgic gait and a laterally (externally) rotated lower extremity. When the onset is acute, the adolescent will be unable to bear weight on the affected extremity. Obesity is often a factor in the development of this condition.

A physical therapist treats a child with cerebral palsy classified at Level V using the Gross Motor Function Classification System. Which of the following recommendations from the therapist is the MOST likely? A. Orthoses and assistive devices for community ambulation B. Orthoses and assistive devices for household ambulation C. Standing frame and orthoses and/or assistive devices for household ambulation D. Standing frame and wheelchair for community ambulation

D. Standing frame and wheelchair for community ambulation Level I- walking, climbing stairs, running, jumping Level II- walking in most settings, climb stairs with railing, difficulty walking long distances and balancing on uneven terrain Level III- walk with a hand-held mobility device in indoor settings, climb stairs with railing, use wheeled mobility for longer distances Level IV- can walk short distances at home with physical assistance, walker, of powered mobility. Use manual w/c or powered mobility in the community. Level V- transported via w/c in all settings, use a standing frame to assume an upright posture

A patient complains of loss of hand grip strength 11 weeks post Colles fracture. The PT notices that the patient has a stronger grip with wrist extension, but the grip strength decreases as the patient starts flexing the wrist. What is the MOST appropriate intervention to improve ROM? A. Stretch the flexor digitorum and strengthen the extensor digitorum B. Strengthen the extensor digitorum C. Stretch both flexor digitorum and extensor digitorum D. Stretch extensor digitorum and strengthen flexor digitorum

D. Stretch extensor digitorum and strengthen flexor digitorum Grip strength is decreasing when going into flexion so the flexors need to strengthen

A PT is evaluating a 34-year-old female patient with a vague diagnosis of low back pain. The patient displays a positive Thomas test. Which sub phase of the gait cycle will MOST likely show limitation in the hip range of motion? A. Loading response B. Initial contact C. Midstance D. Terminal stance

D. Terminal stance A positive Thomas test infers that the patient has tightness in the hip flexors. The hip flexors are stretched when in terminal stance so this deficit will impact that phase the most.

A PT is evaluating a 34 y/o female pt with a vague diagnosis of LBP. The pt displays a positive Thomas test. Which sub phase of the gait cycle will MOST likely show limitation in the hip ROM? A. Loading response B. Initial contact C. Midstance D. Terminal stance

D. Terminal stance The hip flexors will try to stop the hip from going into extension

A patient presents to the physical therapy clinic after complaining of anterior shin pain. During gait analysis, the PT notices that the patient is demonstrating drop foot. Despite the physician's diagnosis of shin splints, the PT notes the signs and symptoms are more consistent with anterior compartment syndrome. To confirm this diagnosis, what clinical findings MUST be present? A. Weak plantarflexors and paresthesia along the anterior lower leg B. Weak dorsiflexors and paresthesia along the anterior lower leg C. Weak plantarflexors and paresthesia between the 1st and 2nd toes D. Weak dorsiflexors and paresthesia between the 1st and 2nd toes

D. Weak dorsiflexors and paresthesia between the 1st and 2nd toes Deep peroneal nerve is most commonly injured (compressed) in anterior compartment syndrome in the leg. Compression may be caused by trauma, tight shoelaces, ganglion, or pes cavus. Motor loss includes an inability to dorsiflex the foot (drop foot), which results in a high steppage gait. Sensory loss is a small triangular area between the first and second toes.

A physical therapist prepares to measure a patient's blood pressure prior to treatment. Which of the following values describes the MOST appropriate rate to release the pressure when obtaining the blood pressure measurement? A. 2-3 mm Hg per second B. 3-5 mm Hg per second C. 5-7 mm Hg per second D. 8-10 mm Hg per second

A. 2-3 mm Hg per second After inflating the cuff to 20 mm Hg above the estimated systolic pressure, the therapist should carefully open the valve and deflate the bladder no more than 2-3 mm Hg per second while listening for the Korotkoff sounds

A patient has decreased eccentric control of the elbow flexors. Which of the following proprioceptive neuromuscular facilitation (PNF) techniques is MOST appropriate to perform? A. Agonist reversals (combination of isotonics) B. Dynamic reversals (isotonic reversals) C. Rhythmic stabilization D. Rhythmic initiation

A. Agonist reversals (combination of isotonics) Agonist reversals include resisted concentric, isometric, and eccentric contraction tasks. Therefore, a patient who has decreased eccentric control would benefit from this technique

After examining a 25-year-old individual with a BMI of 31, the physical therapist designed an exercise training program. Which of the following is the BEST initial exercise prescription? A. 40% to <60% HRR, five days per week for 30-45 minutes B. 60% to <80% HRR, five days per week for 15-20 minutes C. 40% to <60% HRR, three days per week for 15-20 minutes D. 60% to <80% HRR, three days per week for 30-45 minutes

A. 40% to <60% HRR< five days per week for 30-45 minutes A body mass index of 31 can be defined as obese. As per the ACSM guidelines, the initial exercise training for obese individuals should be moderate (40% to <60% of heart rate reserve/ HRR). Eventual progression to more vigorous exercise intensity (i.e. ≥ 60% HRR) may result in further health/fitness benefits. Frequency should be ≥ 5 days to maximize caloric expenditure and duration should be a minimum of 30 minutes per day progressing to 60 min per day of moderate intensity, aerobic activity. 15 to 20 minutes is not a long enough duration and 3 days per week is not a high enough frequency for obese individuals

A physical therapist participates in a research study that formally measures an individual's maximum oxygen consumption. Which of the following individuals would be expected to have the largest maximum oxygen consumption? A. A 23-year-old male (weight: 240 pounds; height: 72 inches) B. A 25-year-old female (weight: 160 pounds; height: 66 inches) C. A 53-year-old male (weight: 210 pounds; height: 69 inches) D. A 47-year-old female (weight: 130 pounds; height: 62 inches)

A. A 23-year-old male (weight: 240 pounds; height: 72 inches) Males have larger maximum oxygen consumption than females. Younger, taller, and heavier individuals also have greater oxygen consumption. SO...the answer is typically a young, large male.

A physical therapist is assessing a 7-year-old boy for gait deviations. On examination, the patient was found to have hip anteversion. The physical therapist would expect to see which of the following first? A. An increase in the amount of hip internal rotation B. An increase in the amount of hip external rotation C. An increase in the amount of genu varus D. An increase in the amount of tibial lateral torsion

A. An increase in the amount of hip internal rotation Hip anteversion= in-toeing and increased hip internal rotation

Which of the following hematological conditions is MOST likely to be found in a patient who has chronic kidney disease? A. Anemia B. Leukopenia C. Neutropenia D. Polycythemia

A. Anemia Anemia is a significant hematologic problem associated with chronic kidney disease due to decreased erythropoietin production, decreased red blood cell lifespan, and reduced iron absorption

Which of the following postural characteristics are MOST likely to be seen in a patient who has lower crossed syndrome? A. Anterior pelvic tilt and slight hip flexion B. Anterior pelvic tilt and slight hip extension C. Posterior pelvic tilt and slight hip flexion D. Posterior pelvic tilt and slight hip extension

A. Anterior pelvic tilt and slight hip flexion A patient who has lower crossed syndrome will have tight erector spinae and iliopsoas muscles and weak abdominal and gluteus maximus muscles. This results in an anterior pelvic tilt, an increased lumbar lordosis, and a slight flexion of the hip.

What landmark is shown in this picture during the auscultation: A. Apical pulse B. Aortic valve C. Tricuspid valve D. Pulmonic valve

A. Apical pulse Apical pulse or point of maximal impulse (PMI) palpate at the 5th interspace, midclavicular vertical line (apex of the heart)

A 30-year-old male patient presents with right lower abdominal pain. PT performs the "pinch-an-inch" test which is positive. Which condition below is MOSTLY related to the symptoms described? A. Appendicitis B. Diverticulitis C. Chron's disease D. Irritable bowel syndrome

A. Appendicitis Diverticulitis and irritable bowel syndrome both present with pain in the LEFT lower quadrant so you can rule those out. Appendicitis is the inflammation of the appendix. Pinch-an-inch test can be used for both appendicitis and diverticulitis; however, diverticulitis would be tested on the left side.

A patient has jaundice, dark urine, and ascites. Which of the following findings is MOST likely to be present during the physical therapy examination? A. Asterixis B. Pronator drift C. Hoffman D. Rebound tenderness

A. Asterixis Jaundice, darkened urine, and ascites are all clinical signs of liver disease. Asterixis, or liver flap, is also likely to be present as a result of ammonia imbalance, which causes this neurologic symptom.

Which of the following is INCORRECT regarding the cardiopulmonary response to acute aerobic exercise from moderate to maximum intensity of work? A. Cardiac output decreases B. Stroke volume slightly decreases C. Systolic blood pressure increases D. Heart rate increases

A. Cardiac output decreases Cardiac output and work rate are directly related so as work rate increases so does cardiac output

A patient is referred to a physical therapist for treatment of chronic neck pain. During the examination, the therapist notices that the patient has a marked ulnar drift in both hands at the MCP joints. Which of the following treatments should the therapist perform? A. Cervical stabilization exercises with the patient in good postural alignment B. Gentle cervical mobilization for 8 minutes C. Intermittent cervical traction at 8% of the patient's body weight for 10 minutes D. Moist hot pack on the patient's cervical spine with the patient in sitting position for 15 minutes

A. Cervical stabilization exercises with the patient in good postural alignment Marked ulnar drift is a hallmark sign of rheumatoid arthritis (O'Sullivan, p. 1000). Because cervical spine ligaments can be affected in this population, cervical stabilization exercises in neutral are appropriate for managing neck pain in patients who have rheumatoid arthritis (Brody).

A physical therapist enters a private treatment area and observes a patient collapsed on the floor. The patient appears to be moving slightly; however, seems to be in need of medical assistance. Which of the following actions should the therapist perform FIRST? A. Check for unresponsiveness B. Monitor airway, breathing, and circulation C. Position the patient D. Phone emergency medical services

A. Check for unresponsiveness The first step is to always check for responsiveness! To check for responsiveness, tap the victim on the shoulder and ask, "Are you alright?" If the patient is unresponsive, the therapist should phone 911, get an AED, provide cardiopulmonary resuscitation, and use the AED, if necessary.

A patient sustained an injury to his knee. While in the hospital, imaging was ordered, and the patient was given a diagnosis of popliteal artery occlusion. What is the BEST treatment option for this patient in the acute care setting? A. Complete bed rest B. Ankle pumps in bed C. Use of compression stockings D. Application of heat for pain relief

A. Complete bed rest Complete bed rest is advised in this situation, as the patient has an active occluded artery. Increased BP via the pumping exercises may increase the likelihood of a cardiovascular event, or increase the risk of tissue damage via hypoxia. Increasing pressure or heat is contraindicated in arterial deficiencies.

While reading the Methods section of a research report, a physical therapist notes the investigators used a repeated measures design. This type of experimental design is used to accomplish which of the following outcomes? A. Controls for differences between subjects B. Keeps the subjects "blind" to the identity of the treatment group C. Ensures that subjects with similar characteristics are assigned to different treatment groups D. Selects a homogenous group of subjects

A. Controls for differences between subjects In a repeated measures design all subjects experience all levels of the independent variable. This provides an efficient method for controlling differences between subjects because characteristics that may affect the outcomes, such as gender, age, and physical characteristics, remain constant for each subject. Since each subject acts as their own control, a repeated measures design is also called a within-subjects design.

A patient with Addison's disease reports fatigue and extreme weight loss. What mechanism is the MOST likely cause of this patient's symptoms? A. Decreased production of cortisol B. Hyperfunction of the thyroid gland C. Decreased absorption of nutrients within the intestines D. Hyperfunction of the adrenal gland

A. Decreased production of cortisol Addison's disease is characterized by hypofunction of the adrenal glands. The adrenal glands are responsible for the production of cortisol and aldosterone. Decreased production of these hormones results in fatigue and weight loss. Addison's disease can be treated by the administration of exogenous cortisol. Hyperfunction of the adrenal gland is Cushing's syndrome!!!

Which of the following mobilizations to the temporomandibular joint would MOST likely improve a limitation in mouth opening? A. Distraction with anterior glide B. Distraction with posterior glide C. Compression with anterior glide D. Compression with posterior glide

A. Distraction with anterior glide Distraction with anterior glide to the temporomandibular joint is best for improving a patient's ability to achieve greater opening of the mouth. Compression causes irritation so you can rule out C and D. A posterior glide helps with CLOSING.

Which of the following findings is MOST common in patients with COPD? A. Dorsal kyphosis B. Hemoptysis C. Cor pulmonale D. Decrease respiratory rate

A. Dorsal kyphosis COPD is characterized by airflow obstruction, air entrapment and pulmonary hyperinflation. In this, the thorax appears enlarged owing to loss of lung elastic recoil and hyperinflation. In COPD, the anterior-posterior diameter of the chest increases, resulting in dorsal kyphosis. Cor pulmonale occurs in later stages of COPD. Respiratory rate can be increased due to increased work of breathing. Hemoptysis occurs in some patients but dorsal kyphosis is the most common finding.

A patient reports complete sensation loss in the C6 dermatome, but has Normal (5/5) muscle strength in all upper extremity muscles. The patient MOST likely has a lesion in which of the following locations? A. Dorsal root ganglia of C6 B. Ventral root ganglia of C6 C. Posterior interosseous nerve D. Medial antebrachial cutaneous nerve

A. Dorsal root ganglia of C6 The dorsal roots are purely afferent nerves with no motor component (Kisner). Dorsal- sensory Ventral- motor

A physical therapist is developing an exercise program for a patient who has upper extremity lymphedema. Which of the following exercises should the patient perform LAST? A. Elbow flexion B. Cervical rotation C. Shoulder abduction D. Shoulder circumduction

A. Elbow flexion Begin with your proximal and move distal for exercises!

A physical therapist is examining a 25-year-old male who had a recent fall while fixing the roof. The patient seems distracted and the PT has difficulty maintaining the patient's attention. Which area of the brain is MOST likely injured? A. Frontal lobe B. Temporal lobe C. Parietal lobe D. Occipital lobe

A. Frontal lobe

The rating of perceived exertion scale is considered to be an acceptable method for determining exercise intensity because it correlates well with: A. Heart rate B. Respiratory rate C. Systolic blood pressure D. Diastolic blood pressure

A. Heart rate

A physical therapist attempts to design an exercise program for a patient with a BMI of 34.5 kg/m^2. Which potential complication of exercise is the MOST relevant for this patient? A. Heat intolerance B. Asthma C. Gastroesophageal reflux D. Orthostatic hypotension

A. Heat intolerance A BMI of 30 kg/m^2 or greater is considered obese. The excessive adipose tissue in obese patients can acts as an extra layer of insulation and not allow heat to dissipate as expected. Obese patients are at high-risk for heat intolerance since they are often unable to appropriately respond to the thermal challenges of exercise.

All of the following are signs and symptoms of Tethered Cord Syndrome in an adult EXCEPT: A. Increased flaccidity B. Scoliosis C. Severe back pain D. Urinary incontinence

A. Increased flaccidity Tethered spinal cord syndrome is a neurological disorder caused by tissue attachments that limit the movement of the spinal cord within the spinal column. These attachments cause an abnormal stretching of the spinal cord. Symptoms: Back pain, worsened by activity and relieved with rest Leg pain, especially in the back of legs, Leg numbness or tingling, Changes in leg strength, Deterioration in gait, Spine tenderness, Scoliosis, Bowel and bladder problems.

A softball pitcher who uses an underarm throw presents in clinic with an outpatient complaint of pain on the medial aspect of her elbow. She is overusing her shoulder external rotators. Analyze why she experiences pain in her elbow? A. Increased valgus stress on the elbow B. Increases varus stress on the elbow C. Leaning backwards at the time of release D. Excessive trunk rotation at the time of release

A. Increased valgus stress on the elbow Repetitive stress in sports especially throwing (involving shoulder external rotation) can create a severe valgus force to the elbow causing a sprain of the medial collateral ligament and a lateral side compression injury.Leaning backwards at the time of release and excessive trunk rotation will not cause valgus forces at the elbow joint.

A patient who has a spinal cord injury reports having spastic (reflex) bowel function. Which of the following descriptions BEST characterizes the patient's neurologic injury? A. Injury above spinal segments S2-S4, leaving spinal defecation reflexes intact B. Injury at or below spinal segments S2-S4, leaving spinal defecation reflexes intact C. Injury above spinal segments S2-S4, abolishing spinal defecation reflexes D. Injury at or below spinal segments S2-S4, abolishing spinal defecation reflexes

A. Injury above spinal segments S2-S4, leaving spinal defecation reflexes intact Above S2-S4: spastic bowel function, spinal defecation reflexes intact At or below S2-S4: flaccid bowel function, loss of spinal defecation reflexes

You are assessing a patient in the ICU who opens eyes to pain, has an abnormal flexion response to motor stimuli, and is oriented verbally. You would MOST likely categorize this patient on the Glasgow Coma Scale as having what type of brain injury? A. Moderate B. Very severe C. Mild D. Severe

A. Moderate This patient would score a 10 on the Glasgow Coma Scale. <8: severe 9-12: moderate 13-15: mild

A PT is performing cranial nerve testing on a 58-year-old male patient. The patient has ptosis, lateral strabismus, and a dilated pupil in the right eye. Which of the following cranial nerve test results is MOST likely to be abnormal? A. Oculomotor nerve (CN III) B. Optic nerve (CN II) C. Facial nerve (CN VII) D. Trigeminal nerve (CN V)

A. Oculomotor nerve (CN III) Lateral strabismus is when the eye moves down and laterally

A patient had a cerebrovascular accident 6 months ago. The patient is unable to perform volitional fractionated movements in the hemiparetic lower extremity. The patient's goal is to be able to walk short distances. Which of the following treatments is MOST appropriate for the patient? A. Perform body-weight-supported treadmill training B. Use assistive devices to maximize functional gait C. Perform quadriceps strengthening in the hemiparetic leg D. Perform static standing balance activities

A. Perform body-weight-supported treadmill training Body-weight-supported treadmill training has been shown to be effective in allowing intensive practice of gait in individuals who have a cerebrovascular accident. Use of a harness allows for a safe environment to practice a complex motor task for a patient with severe neuromuscular impairment.

A PT aide was treating a patient who suffered from a CVA and asked the patient to complete bridging exercises that were not in the original POC. During the bridging exercises, the patient suffered a back injury. Who will need to take the responsibility for the injury? A. Physical therapist B. Physical therapy assistant C. Physical therapy aide D. Physical therapy student

A. Physical therapist Unfortunately, even though you did not approve the exercise in the POC the physical therapist will be responsible. The responsibility always falls on licensed practitioners. So, a PT student and PT aide will never be at fault.

A physical therapist completes a respiratory assessment on a patient in an acute care hospital. The examination reveals decreased breath sounds and decreased fremitus. This finding is MOST indicative of which of the following medical conditions? A. Pleural effusion B. Pulmonary edema C. Consolidation D. Atelectasis

A. Pleural effusion Decreased breath sounds and decreased fremitus are most likely caused by pleural effusion or pneumothorax. Pulmonary edema, consolidation, and atelectasis are associated with decreased breath sounds but INCREASED fremitus.

Upon examination, the PT finds the following range of motion values: right elbow flexion 10-100, forearm supination 0-75, forearm pronation 0-45. What is the MOST appropriate intervention to help increase range of motion on the right side? A. Posterior glide of the radius at the proximal radio-ulnar joint B. Dorsal glide of ulna at the distal radio-ulnar joint C. Lateral glide of radius at the distal radio-ulnar joint D. Medial glide of the radius at the radio-carpal joint

A. Posterior glide of the radius at the proximal radio-ulnar joint The patient is limited in both elbow flexion and forearm pronation. For pronation you will have an anterior roll and posterior glide at the proximal radio-ulnar joint. Pronation is usually around 75 degrees and supination is around 80 degrees.

A physical therapist assesses several superficial reflexes as part of a neurologic examination. Which grading system is the MOST appropriate when documenting the obtained results? A. Present, absent B. Ordinal scale from 0-4+ C. Hypoactive, normal, hyperactive D. Zero, trace, poor, fair, good, normal

A. Present, absent Superficial reflexes are involuntary muscle contractions that follow stimulation of the skin usually by stroking or scratching. Superficial reflexes are graded as present or absent, although asymmetry should be noted. True asymmetry is almost always pathological.

A PT is developing a treatment plan for a 32-year-old male patient 4 weeks post arthroscopic debridement of the L knee joint. The PT decides to use electrical stimulation to improve the strength of the quadriceps. Which of the following parameters is the MOST APPROPRIATE for this patient? A. Pulse duration- 250-300 microseconds; 10 seconds on, 50 seconds off; ramp up/down time- 2 seconds B. Pulse duration- 150-170 microseconds; 10 seconds on, 20 seconds off; ramp up/down time- 2 seconds C. Pulse duration- 250-300 microseconds; 10 seconds on, 20 seconds off; ramp up/down time- 6 seconds D. Pulse duration- 150-170 microseconds; 10 seconds on, 50 seconds off; ramp up/down time- 6 seconds

A. Pulse duration- 250-300 microseconds; 10 seconds on, 50 seconds off; ramp up/down time- 2 seconds When a muscle is weak you want a longer pulse duration but you want to be gentle so you have them rest more. Therefore, a on:off ratio of 1:5 is appropriate. Ramp time needs to be between 1-4 seconds (this is standard). The bigger the muscle the longer the duration!

The photograph shows a patient performing active shoulder range of motion. Which of the following exercises will BEST address the patient's problem? A. Push-ups against a wall B. Abduction in the scapular plane C. Lateral (external) rotation with elastic resistance D. Scapular retraction in prone position

A. Push-ups against a wall The photograph depicts scapular winging, which can occur with weakness of the serratus anterior (Magee). The push-up against a wall assists with serratus anterior strengthening (Kisner, p. 609).

A 2-year-old child with Down syndrome is being treated by a physical therapist in an outpatient clinic. The MOST appropriate intervention should include: A. Pushing a toy cart while standing B. Locomotor training using body weight support C. Rolling and somersault activities D. Rhythmic stabilization of postural extensors in sitting

A. Pushing a toy cart while standing Children with Down syndrome have hypotonicity; so weight bearing activities should be encouraged. Hence, standing is preferred over sitting. Pushing a toy cart is a functional activity which will help in improving the gross and fine motor skills. Somersault should not be done due to atlantoaxial joint instability.

Which of the following substitution patterns should be prevented when measuring active forearm pronation? A. Shoulder medial (internal) rotation and shoulder abduction B. Shoulder medial (internal) rotation and shoulder adduction C. Shoulder lateral (external) rotation and shoulder abduction D. Shoulder lateral (external) rotation and shoulder adduction

A. Shoulder medial (internal) rotation and shoulder abduction Lateral rotation and adduction should be avoided during SUPINATION

A patient is semi-conscious and opens his eyes to pain. The best motor response elicited is withdrawal and he is producing incomprehensible sounds. Which of the following is the MOST APPROPRIATE indicator of the level of consciousness and severity of injury on the Glasgow Coma Scale (GCS)? A. The patient has a GCS score of 8/15 indicative of severe brain injury B. The patient has a score of 5/15 indicative of severe brain injury C. The patient has a score of 12/15 indicative of mild brain injury D. The patient has a score of 10/15 indicative of moderate brain injury

A. The patient has a GCS score of 8/15 indicative of severe brain injury Opens eyes to pain: 2 Normal flexion (withdrawal): 4 Incomprehensible sounds: 2

A patient sustained a grade II strain to the iliopsoas muscle. Which of the following phases of the gait cycle should the physical therapist expect to be the MOST impacted by this injury? A. Toe off (pre-swing) and acceleration (initial swing) B. Heel strike (initial contact) and acceleration (initial swing) C. Foot flat (loading response) and deceleration (terminal swing) D. Midstance and deceleration (terminal swing)

A. Toe off (pre-swing) and acceleration (initial swing) The iliopsoas works eccentrically from midstance to toe off (pre-swing) and then switches quickly to a concentric contraction to advance the limb forward at acceleration (initial swing). Therefore, weakness of the hip flexors is best observed during the toe off (pre-swing) and acceleration (initial swing) phases.

A physical therapist recognizes that a child has significant difficulty flexing the neck while in a supine position. Failure to integrate which of the following reflexes would BEST explain the child's difficulty? A. Tonic labyrinthine B. Moro C. Asymmetrical tonic neck D. Symmetrical tonic neck

A. Tonic labyrinthine The tonic labyrinthine reflex promotes a tendency for extension when a patient is in supine and serves to limit the child's ability to flex the neck when in a supine position. The child should lie in sidelying or in supine with hip flexion and/or knee flexion in order to decrease the influence of the reflex.

Which of the following tests is MOST accurate for assessing volume reduction in a patient who has lymphedema? A. Water displacement B. Limb circumference C. Bioelectrical impedance D. Optoelectronic volumetry

A. Water displacement Water displacement has been regarded as the most sensitive and accurate standard for volume measurement.

A 27-year-old patient arrives at the clinic for evaluation and treatment post partial nerve injury of long thoracic nerve. On examination which of the following is most likely to be true? A. Winging and weakness when lifting arm above 90 degrees shoulder flexion B. Winging and weakness when lifting arm above 90 degrees of shoulder abduction C. Winging and weakness when lifting arm beyond 60 degrees of shoulder flexion D. Winging and weakness when lifting arm beyond 60 degrees of shoulder abduction

A. Winging and weakness when lifting arm above 90 degrees shoulder flexion The long thoracic nerve innervates the serratus anterior so you know the serratus anterior is the cause of the winging. The serratus anterior typically causes winging into shoulder flexion (medial winging) whereas the trapezius causes winging into abduction (lateral winging).

A 75-year-old individual has limited endurance. There is no history of cardiorespiratory problems. After an exercise tolerance test, which was negative for coronary heart disease, the BEST initial exercise prescription for this individual would be: A. 80-90% HR max B. 60-70% HR max C. 40-50% HR max D. 30-40% of HR max

B. 60-70% HR max There is nothing wrong with this patient, they are just out of shape and older. 40-50% HR max is nearly a resting heart rate for this patient. (220-age= HR max)

The PT is treating a 23-year-old male patient who sustained left hip fracture. The patient is having difficulty gaining hip flexion ROM. How much minimum hip flexion should the PT try to obtain so that the client can ambulate with a normalized gait pattern? A. 15 degrees B. 25 degrees C. 35 degrees D. 45 degrees

B. 25 degrees Use the rule of extremes to rule out 15 and 45 degrees. You need approximately 30 degrees of hip flexion for a normal gait pattern but you don't need to surpass that number.

A 10-year-old child presents with burns to the trunk, anterior left arm, anterior left leg, and genitals. What percentage of the child's body is suffering from burns, based on the Lund Browder chart? A. 29% B. 30% C. 31.5% D. 32.5%

B. 30% Trunk: 18% Anterior leg: 6.5% Anterior arm: 4.5% Genitals: 1%

A PT is working for a home health company and mean annual income for all PTs is $100,000. Last year, the physical therapist's salary was 2 standard deviations more than the mean. What percentage of his salary falls ABOVE $100,000? A. 68% B. 47.5% C. 95% D. 34%

B. 47.5% Imagine a bell curve. The middle of the curve is $100,000. You are concerned about 2 standard deviations ABOVE the mean so you only care about the right side of the graph. -2 SD to 2 SD is 95% but since you only care about the right side you will divide that number by 2 to get 47.5%.

A PT is treating a child in an outpatient clinic. The PT notices that the child is unable to transfer a toy from one hand to another hand. What age would be considered normal for a child to be able to transfer an object from one hand to another hand? A. 4 months B. 9 months C. 12 months D. 18 months

B. 9 months NORMAL MILESTONES 4 months: supine to side lying transfer (not able to sit independently to transfer objects back and forth) 9 months: quadruped creeping, cruising, can stand alone, transfer objects between hands 12 months: walk unassisted, in and out of squatting, floor to stand transfers, pincer grasp

A PT is performing a sensory examination of a patient post-SCI. When the PT strokes lightly on the patient's arm with a camel brush, the patient doesn't recognize the sensation. On pricking the patient with a neurological pin, the sensation is recognized. What other sensation is most likely recognized by the patient? A. Letters drawn on the hand B. A warm test tube placed on the skin C. The shape of an object placed in his hand D. Vibration from a tuning fork

B. A warm test tube placed on the skin The spinothalamic tract is responsible for pain and temperature (lateral spinothalamic tract) and crude touch and pressure (anterior spinothalamic tract). Since the patient is unable to feel the camel brush you can assume there is an issue with the dorsal column medial lemniscus system. A- dorsal column medial lemniscus B- spinothalamic tract C- stereognosis; dorsal column medial lemniscus D- dorsal column medial lemniscus

Which of the following is NOT an expected sign seen in a pt with facial nerve (CN VII) palsy? A. Hyperacusis B. Absence of sensation in the anterior 2/3 of the tongue C. Absence of corneal reflex D. Decrease lacrimation

B. Absence of sensation in the anterior 2/3 of the tongue Rationale: Sensation in anterior 2/3 of the tongue is by the trigeminal nerve and not the facial nerve. Facial nerve is for TASTE on anterior 2/3 of the tongue. A- defined as the collapsed tolerance to normal environmental sounds. Facial nerve controls the excessive movements of the stapedius bone and dampens the sound and its injury can cause hyperacusis.

A physical therapist treats a patient with a peroneal tendon subluxation. To observe the subluxation, which of the following motions is the MOST appropriate to reproduce the subluxation? A. Active plantar flexion and inversion B. Active ankle dorsiflexion and eversion C. Passive ankle plantar flexion and inversion D. Passive ankle dorsiflexion and eversion

B. Active ankle dorsiflexion and eversion The peroneus longus and brevis act as strong evertors of the ankle. Ankle dorsiflexion and eversion are often used to reproduce the subluxation; however, active motion is required since the tendon is a contractile structure.

A physical therapist administers a special test to a patient with suspected hand pathology. The pinch test shown in the image would be used to assess the integrity of which nerve? A. Posterior interosseous B. Anterior interosseous C. Deep radial D. Ulnar

B. Anterior interosseous The anterior interosseous nerve is a branch of the median nerve that innervates the deep muscles on the anterior forearm, except the medial half of the flexor digitorum profundus. Inability to maintain tip-to-tip pinch is most likely due to weakness of the flexor pollicis longus.

A physical therapist is evaluating a patient with Cauda Equina syndrome. Which of the following correctly describes the typical bowel and anesthesia distribution pattern for this population? A. Flaccid bowel and bladder, sciatic anesthesia B. Areflexic bowel and bladder, saddle anesthesia C. Spastic bowel and bladder, sciatic anesthesia D. Spastic bowel and bladder, saddle anesthesia

B. Areflexic bowel and bladder, saddle anesthesia Cauda Equina syndrome is characterized by a flaccid (or areflexic) bowel and bladder and saddle anesthesia. It presents like a LMN lesion!

What acute change in arterial oxygenation will a person who resides close to sea level experience immediately after ascending to an altitude of 5,000 ft above sea level or higher? A. Arterial oxygenation is reduced due to reduced oxygen concentration in the air B. Arterial oxygenation is reduced due to reduced air pressure C. Arterial oxygenation is reduced due to increased air pressure D. Arterial oxygenation is increased due to increased air pressure

B. Arterial oxygenation is reduced due to reduced air pressure

PT is treating a 6-year-old child with autism in outpatient pediatric physical therapy. Which of the following is the MOST appropriate intervention? A. Tumbling on mat to promote core strength B. Ball toss, hopping, and running with a partner with verbal cues for turn taking C. Perform activities in a fast-paced environment with long explanations for sequencing D. Performing standing dance routine in different rooms to improve generalizability of a skill

B. Ball toss, hopping, and running with a partner with verbal cues for turn taking Tumbling is not structured enough for patients with autism. They need very structured activities and do really well with verbal cueing.

A patient has contralateral hemianopsia and hemiplegia (greater in the face and arm rather than leg), along with that the patient demonstrates hesitant speech and perceptual problems such as depth perception and agnosia. Which of the following arteries is MOST likely affected? A. Basilar B. Middle cerebral C. Posterior cerebral D. Anterior cerebral

B. Middle cerebral Middle- UE and face Anterior- LE Posterior- vision issues

In a restaurant, a therapist sees someone choking. He approaches the individual, and asks to help. The patient refused, and proceeds to collapse. What is the MOST appropriate next action for the therapist? A. Call EMS and wait for them, since patient didn't give consent to the PT B. Call EMS and begin CPR C. Give abdominal thrusts D. Position the patient in sidelying

B. Call EMS and begin CPR With an unresponsive patient, activation of the emergency response system, and initiation of CPR should be done. When a patient is unresponsive consent is considered unless the patient has signed a DNR.

A patient who has right hemiparesis following a cerebrovascular accident is habitually positioned in right sidelying position. Which of the following problems may result from this positioning and should be of GREATEST concern to the physical therapist? A. Left gaze preference B. Chronic right shoulder pain C. Trunk shortening on the right D. Skin breakdown on the medial aspect of left knee

B. Chronic right shoulder pain Hemiplegic shoulder pain is a common complication after stroke. Poor positioning of the more affected upper extremity has been implicated in producing joint microtrauma and pain. Prolonged soft tissue injury can result in complex regional pain syndrome.

A 68-year-old patient is performing a Bruce protocol on an inclined treadmill. The PT is monitoring the patient using ECG leads. During the exercise, the PT sees the first-degree AV block ECG pattern as shown in the picture. The PT's BEST response should be: A. Stop the treadmill session immediately and call the cardiologist B. Continue without any modifications and monitor ECG C. Reduce the treadmill speed and monitor ECG D. Stop the treadmill, have the patient rest, and then resume at a lower intensity

B. Continue without any modifications and monitor ECG The Bruce protocol is meant to test their maximal abilities so there is no point in it if you reduce the speed and you can rule C out. A first-degree block is not a big deal but you should monitor it.

78-year-old patient was admitted with a diagnosis of acute exacerbation of COPD. The patient has a FEV1 of 40% with FEV1/FVC < 70% and has SOB during ambulation. According to the GOLD classification, the patient would MOST likely be classified as: A. Very severe B. Severe C. Moderate D. Mild

B. Severe FEV1= amount of air you get out in the first second of expiration Very severe: FEV1 <30% Severe: FEV1 30-50% Moderate: FEV1 50-80% Mild: FEV1 >80%

A physical therapist documents in the medical record that a patient has moved from stage 5 to stage 6 of Brunnstrom's Stages of Recovery. This type of transition is characterized by which of the following changes in movement patterns? A. Absence of associated reactions B. Disappearance of spasticity C. Voluntary movement begins outside of synergy patterns D. Return of normal motor function

B. Disappearance of spasticity Brunnstrom's stages are used for recovery following a stroke 1: flaccidity 2: spasticity begins to develop 3: peak of spasticity 4: spasticity declines 5: spasticity still decreasing, out of synergies 6: disappearance of spasticity, isolated joint movement 7: normal

A physical therapist reviews a patient coverage form that lists the following parameters used during a recent ultrasound treatment: 1.5 W/cm^2, pulsed 20%, 1 MHz, 6 minutes. If the objective of the ultrasound treatment was to increase tissue temperature, what parameter would be the MOST critical for the therapist to alter? A. Time B. Duty cycle C. Frequency D. Intensity

B. Duty cycle Duty cycle is the ratio of on time to the total time. When ultrasound is used in a pulsed mode with a 20% or lower duty cycle, the heat produced during the on time of the cycle is dispersed during the off time and as a result there is no measurable net increase in temperature. To increase the temperature it would be necessary to significantly increase the duty cycle or use a continuous mode.

A physical therapist treats a patient post CVA who presents with speech and language deficits. The therapist incorporates phonetics into the plan of care. This intervention would be MOST essential for a patient diagnosed with which of the following disorders? A. Broca's aphasia B. Dysarthria C. Verbal apraxia D. Dysphagia

B. Dysarthria Dysarthria is a motor disorder of speech that is caused by an UMN lesion. The condition affects the muscles that are used to articulate words and sounds. Speech is often noted as "slurred" and there may also be an effect on respiratory or phonatory systems due to weakness. Treatment focuses on improving the intelligibility of speech by strengthening all aspects of speech production through phonetics. Exercises will also work on coordination and articulatory precision.

You are treating a patient with a cerebellar infarct that is affecting the UE. The patient has trouble coordinating rapid, alternating movements of the UE. Finger to nose testing reveals minimal aberrations, and toe tapping is within functional limits. What BEST describes the aforementioned symptoms? A. Dysmetria B. Dysdiadochokinesia C. Asthenia D. Resting tremor

B. Dysdiadochokinesia Dysdiadochokinesia is tested with alternating movement.

A physical therapist works with a patient who has left-sided hemiparesis as depicted in image B. As the therapist facilitates an anteriorly directed weight shift, the patient shifts weight onto the left lower extremity through which of the following mechanisms? A. Concentric activity of the left quadriceps and soleus B. Eccentric activity of the left quadriceps and soleus C. Concentric activity of the right quadriceps and soleus D. Eccentric activity of the right quadriceps and soleus

B. Eccentric activity of the left quadriceps and soleus An anterior weight shift would promote increased hip flexion, knee flexion, and dorsiflexion on the LLE which are the opposite of the quadriceps and soleus muscle actions hence it will be eccentric. The right leg will be maintaining proximal control when shifting anteriorly.

A physical therapist examines a patient post transtibial amputation. The patient resides in a retirement community and is socially active. The patient is presently using a temporary prosthesis consisting of a plastic socket, a pylon, and a SACH foot. The patient expresses concern that the permanent prosthesis will look awful and will be obvious to everyone. Which type of prosthesis would be the MOST appropriate for the patient? A. Endoskeletal shank and single-axis articulated foot-ankle assembly B. Endoskeletal shank and solid ankle cushion heel (SACH) foot C. Exoskeletal shank and single-axis articulated foot-ankle assembly D. Exoskeletal shank and solid ankle cushion heel (SACH) foot

B. Endoskeletal shank and solid ankle cushion heel (SACH) foot An endoskeleton is designed to incorporate a synthetic foam cover shaped like the opposite leg. As a result, the device is more cosmetically attractive and would likely make the patient more socially comfortable. Since the patient is of retirement age and expect slow impact on the prosthesis, a SACH foot will be appropriate.

Which of the following exercise guidelines is MOST appropriate for a patient who has diabetes? A. Exercise during peak insulin times B. Exercise at the same time each day C. Avoid exercise if the blood glucose level is less than 150 mg/dL (8.3 mmol/L) D. Avoid exercise if the blood glucose level is greater than 200 mg/dL (11.1 mmol/L)

B. Exercise at the same time each day Patients who have diabetes should exercise regularly and consistently AVOID EXERCISING DURING PEAK INSULIN TIMES

A 62-year-old patient came to an outpatient clinic. He wants to start aerobic exercise. Which of the following is the MOST appropriate value suggesting aerobic exercise intensity? A. Exercise at 4-5 MET B. Exercise with heart rate between 106 to 145 bpm C. Exercise at 158 bpm D. Exercise at 85% to 95% of VO2max

B. Exercise with heart rate between 106-145 bpm Karvonen formula: THRR = HRrest + (0.6 [HRmax - HRrest]) HRmax = 220 - 62 = 158 bpm HRrest is not provided so we use 60-85% of max HR. 158 * 0.6 = 95 158 * 0.85 = 134 THRR = 95 + (0.6 [158-95]) = 133 bpm THRR = 134 + (0.6 [158-134]) = 148 bpm So, the approximate target HR is 133 bpm - 148 bpm. B is the closes to this.

To prevent contractures in a patient with an above knee (transfemoral) amputation, emphasis should be placed on designing a positioning program that maintains range of motion in hip: A. Flexion and abduction B. Extension and adduction C. Adduction and lateral rotation D. Flexion and medial rotation

B. Extension and adduction When a limb is cut it has a tendency to go into flexion and abduction so you want to move in the opposite direction of this to prevent a contracture.

In a PT session, the PT commands the patient to wear a shirt but the patient is unable to complete the task due to inability to find buttons on his shirt. Which of the following is the MOST APPROPRIATE diagnosis and the best possible strategy to address this deficit? A. Form discrimination, patient should be encouraged to use touch to identify objects B. Figure-ground discrimination, patient should be encouraged to use touch to identify objects C. Position in space impairment, orienting the patient with one object in relation to another D. Ideational apraxia, having the patient perform one part of the task at a time and guiding the patient with the task if needed

B. Figure-ground discrimination, patient should be encouraged to use touch to identify objects The patient is unable to see his buttons on the background of his shirt.

A patient with low back pain has L4 nerve root impingement. The patient will MOST likely demonstrate which of the following gait deviations? A. Trendelenburg gait B. Foot slap C. Posterior thrust of the trunk at heel strike (initial contact) D. Toe walking

B. Foot slap The L4 nerve root is the main segmental innervation to the tibialis anterior. The L4 nerve root is also the myotome for ankle dorsiflexion. Impingement of the L4 nerve root would result in foot slap.

A patient underwent surgical repair of a superior labrum tear in the shoulder 1 week ago. Which of the following resisted motions should be AVOIDED during physical examination of the patient? A. Forearm pronation B. Forearm supination C. Scapular retraction D. Scapular protraction

B. Forearm supination Resisted forearm supination is performed by active contraction of the biceps brachii and supinator muscles (Dutton, p. 718). The long head of the biceps brachii has firm attachments to the superior labrum of the shoulder (Dutton, p. 595). Resisted contraction of the biceps brachii during the first 3-6 weeks post surgery, when the labrum tear has not yet fully healed, can place excessive strain on the labrum and can cause severe damage to the repaired labrum. Therefore, resisted forearm supination should be avoided during the first 6 weeks, which is the time needed for proper healing of the labrum (Dutton, pp. 653-654).

A physical therapist reviews the parameters of several pain modulation theories using transcutaneous electrical nerve stimulation (TENS). When comparing sensory stimulation to motor stimulation, sensory stimulation requires which of the following modifications to the parameters? A. Greater phase duration B. Greater frequency C. Stronger amplitude D. Shorter treatment time

B. Greater frequency Frequency is significantly greater with sensory level stimulation compared to motor level stimulation. High frequency, short phase duration = sensory stimulation

A physical therapist is asked to provide a presentation on the dangers of anabolic steroid use to a group of adolescent male athletes. Which risk factor associated with steroids would be unique to the target audience? A. Deepened voice B. Growth cessation C. Liver damage D. Testicular atrophy

B. Growth cessation A deepened voice is seen in females taking steroids. Liver damage is a risk for ALL individuals taking steroids. Testicular atrophy is a risk for males of all ages, not just adolescents. Rising levels of testosterone and other hormones trigger the growth spurt associated with adolescence. When these hormones reach a specific level, they signify the conclusion of growth. Adolescents using anabolic steroids exhibit artificially increased levels of sex hormones that may trigger the cessation of bone growth prematurely.

A physical therapist works with a patient who has right hemiplegia post CVA Which of the following therapeutic positions would be the MOST difficult for this patient to maintain? A. Half-kneel with involved leg anterior B. Half-kneel with involved leg posterior C. Bilateral tall kneeling D. Bilateral lower extremity bridge

B. Half-kneel with involved leg posterior Half-kneel with the involved leg posterior is the most difficult position for the patient to maintain since the posterior leg is responsible for increased body weight as compared to tall kneeling. The overall increased stability demands placed on the affected posterior limb provides additional challenge to all of the supporting muscles in order to maintain hip extension and lower trunk control.

A PT student is working on her research project to study the effects of strengthening hamstring muscles along with core muscle strengthening in patients with low back pain. For the study to be ethical and unbiased, which of the following is the MOST APPROPRIATE way to randomize the subjects into control and experimental groups? A. PT asks the subject's family which group the subject should be in but does not tell the patient B. Have the patients choose an envelope which will determine which group they will be placed in but do not inform the patient which group they are in C. SPT divides the patients into groups based on the severity and chronicity of the back pain with the experimental group consisting of more chronic LBP patients D. SPT asks the subject which group he/she prefers to be in

B. Have the patients choose an envelope which will determine which group they will be placed in but do not inform the patient which group they are in Blinding and randomization are essential in randomized control trials (RCT). Having the patient, PT, or family member choose which group the subject should be in is not true randomization and creates a bias. Having a patient choose an envelope with a pre-determined group will be more appropriate for randomization, and it will not be biased.

A physical therapist is examining a 54-year-old patient and diagnoses a right pelvic drop during walking at a self-selected walking speed. Which of the following is a potential cause of right pelvic drop? A. Weak hip abductors on right side B. Hip abductor contracture on right side C. Weak hip adductors on left side D. Hip abductors contracture on left side

B. Hip abductor contracture on right side With a pelvic drop you will either see a weakness in the hip abductors (stretched) on the contralateral side or a contracture of the hip abductors on the ipsilateral side.

A patient with a diagnosis of Parkinson's Disease is referred to an outpatient PT clinic. He is beginning to show signs of bilateral involvement, but has had no falls so far. Which of the following is the MOST APPROPRIATE stage of Hoehn-Yahr classification? A. Hoehn and Yahr stage V B. Hoehn and Yahr stage II C. Hoehn and Yahr stage I D. Hoehn and Yahr stage III

B. Hoehn and Yahr stage II Symptoms of rigidity and postural instability are cardinal symptoms of Parkinson's disease. Parkinson's disease is classified with the Hoehn and Yahr scale. Hoehn and Yahr Level II indicates minimal bilateral or midline involvement with no balance impairment.

All of the options below are potential complications of obesity during exercise EXCEPT: A. Precipitation of angina pectoris or myocardial infarction B. Hyperhydration and increased circulating blood volume C. Ligamentous injuries D. Chafing

B. Hyperhydration and increased circulating blood volume Hyper hydration is not seen with obesity since obese people are susceptible to dehydration. They have increased heat intolerance as that they are less able to adapt to temperature changes, risk of hyperthermia and heat exhaustion.

Use of a pneumatic compression pump in the lower extremity is CONTRAINDICATED for a patient who has which of the following findings? A. Ankle-brachial index of 0.9 B. Hypoproteinemia measured at less than 2 g/dL C. Resting blood pressure of 140/90 mm Hg D. Fasting blood glucose value of 118 mg/dL (6.5 mmol/L)

B. Hypoproteinemia measured at less than 2 g/dL Hypoproteinemia less than 2 g/dL is a contraindication for compression as this intervention can increase intravascular fluid thus further lowering serum protein concentration, which can adversely affect cardiac or immunologic function

31-year-old pregnant obese female, in her third trimester, is diagnosed with damage of the pudendal nerve. The patient is referred to physical therapy clinic for pelvic floor strengthening. What is the BEST initial exercise prescription for the involved muscles? A. In a supine position; 5 contractions held for 3 seconds each B. In left side-lying; 10 contractions held for 5 seconds each C. In right side-lying; 10 contractions held for 5 seconds each D. In a seated position; 10 contractions held for 10 seconds each

B. In left side-lying; 10 contractions held for 5 seconds each For muscle strengthening, initial position can be supine or side-lying but as the patient is pregnant (in the third trimester), supine position should be avoided. Sitting will help strengthen the muscles against gravity and can be used in a more advanced phase, hence, left side lying with ten contractions held for 5 seconds each should be done. *Doctors recommend pregnant women only lay on their left side because it helps prevent your uterus from laying on your liver (right side)!

A patient complains of R knee discomfort while using their BiCAAL device. The PT observed the R knee collapse when the foot makes contact to the ground during early stance. Which of the following is MOST likely causing this impairment? A. Adequate knee lock B. Inadequate dorsiflexion stop C. Inadequate plantarflexion stop D. Excessive dorsiflexion stop

B. Inadequate dorsiflexion stop "R knee collapse" means the knee is going into knee flexion. If you have an inadequate dorsiflexion stop you will be able to go into a lot of dorsiflexion which causes knee flexion. Too much PF= hyperextension Too much DF= hyperflexion

Which of the following test results is MOST consistent with a T1 spinal cord injury (ASIA Impairment Scale B)? A. Intact sensation on the apex of the axilla, active movement of the elbow flexors against gravity, and absence of anal sensation B. Intact sensation on the medial side of the antecubital fossa, palpable muscle activity of the little finger (5th digit) abductors, and presence of anal sensation C. Intact sensation on the dorsal surface of the proximal phalanx of the middle finger (3rd digit), palpable muscle activity of the wrist extensors, and absence of anal sensation D. Intact sensation on the dorsal surface of the proximal phalanx of the thumb (1st digit), active movement of the wrist extensors against gravity, and presence of anal sensation

B. Intact sensation on the medial side of the antecubital fossa, palpable muscle activity of the little finger (5th digit) abductors, and presence of anal sensation The intact sensation on the medial side of the antecubital fossa and contraction of finger abductors is consistent with the T1 dermatome and myotome. In order for an injury to be considered incomplete (ASIA Impairment Scale B), either deep anal sensation or some sensation in the anal mucocutaneous junction must be present.

The PT will measure the distance between spinous process to inferior angle at 0, 60, and 100 degrees of abduction at three different time points. One of the MOST important factors for the success of this study is: A. Face validity B. Intra-rater reliability C. Inter-rater reliability D. Content validity

B. Intra-rater reliability Inter-reliability is between two different individuals so you can rule that out immediately.

Which of the following rehabilitation activities will cause the MOST strain to an ACL following allographic reconstruction? A. Isometric quadriceps contraction at 60 degrees B. Isometric quadriceps contraction at 15 degrees C. Isometric hamstring contraction at 15 degrees D. Simultaneous quadriceps and hamstrings contraction at 60 degrees

B. Isometric quadriceps contraction at 15 degrees Isometric exercise of quadriceps at 15 degrees will cause open chain (end- range) terminal extension. Contraction of the quadriceps in this position and range causes anterior tibial translation and can create excessive stress to the graft during the early stage of healing. So, open chain terminal knee extension is not advised post ACL repair.

During assessment of skin sensation, which of the following structures are responsible for transmission of the cold sensation? A. Meissner corpuscles B. Krause end bulbs C. Golgi tendon organs D. Ruffini endings

B. Krause end bulbs "Kold" sensation Meissner corpuscles: fine, discriminative touch; vibration Golgi tendon organs: senses tension in the muscle Ruffini endings: hot sensation

A patient describes sharp elbow pain over the origin point of the common extensor tendon of the wrist extensors. The pain is alleviated with rest and the Mill's test is positive. Which of the following disorders is MOST likely present? A. Medial epicondylitis B. Lateral epicondylitis C. Anconeus tendonitis D. Olecranon bursitis

B. Lateral epicondylitis In lateral epicondylitis, a patient complains of pain over the lateral aspect of elbow. The pain is related to activities that involve wrist extension/grasp, as it is the wrist extensors that must contract during grasping activities to stabilize the wrist. Diffuse achiness and morning stiffness are also common complaints. And Cozen's test, Mill's test will be positive.

A 50-year-old female patient presents with right-sided arm weakness and sensory loss. The PT also notes that homonymous hemianopsia and global aphasia are also present. The MOST likely location for her brain injury is that: A. Left MCA superior division B. Left MCA stem C. Right MCA superior division D. Right MCA stem

B. Left MCA stem Occlusion of Left MCA stem causes right side hemiparesis, global aphasia, and contralateral homonymous hemianopsia. Superior division of MCA causes Broca's aphasia and inferior division of MCA causes Wernicke's Aphasia.

A 28-year-old professional mountain bike rider presents to cardiac rehabilitation after suffering a MI 6 months ago. Which exercise test would be MOST beneficial in order to assess aerobic capacity and create an aerobic conditioning program? A. Bruce protocol B. Lower extremity ergometry C. Six minute walk test D. Step test

B. Lower extremity ergometry You want to test their aerobic condition in a similar position as what they do often.

A 24 y.o. male was training for baseball and hurt his finger while trying to catch the ball. He experienced a sudden sharp pain in his finger, and his finger was swollen immediately. He finished his training and went to the ER with the following presentation. Which of the following deformities does this patient MOST LIKELY have? A. Zig-Zag deformity B. Mallet finger deformity C. Claw finger deformity D. Trigger finger deformity

B. Mallet finger deformity A mallet finger deformity is the result of a rupture or avulsion of the extensor tendon where it inserts into the distal phalanx of the finger. The distal phalanx rests in a flexed position.

A patient that is eight days status post ACL reconstruction (patellar tendon autograft) is being examined by a physical therapist. What is the MOST appropriate exercise to implement into the patient's home exercise program? A. Leg press on operated side B. Mini squats C. Limited range isokinetic at 30 degrees D. Active knee extension in short sitting

B. Mini squats You want to avoid open-chain terminal knee extension because it produces shear forces that stretch the ACL. So you can rule out D. Option C is nearly terminal knee extension so you would avoid that too. Leg press on the operated side is too advanced at this point.

A 35-year-old female complains of muscle weakness. Upon examination, the physician detected ptosis, diplopia, limited facial expression, and significant weakness in bilateral upper extremities that is greater than weakness in the bilateral lower extremities. The physician concluded that the patient is MOST likely diagnosed with which condition? A. Guillian Barre Syndrome B. Myasthenia gravis C. Multiple sclerosis D. Parkinson's disease

B. Myasthenia gravis MS is typically a UNILATERAL problem so you can rule out C. Guillain Barre Syndrome presents as a LMN lesion and MS as an UMN lesion. Myasthenia gravis is a neuromuscular junction disorder characterized by weakness that worsens during periods of activity and improves after rest. The weakness ranges from ptosis or diplopia to respiratory weakness. Muscles that control speech, facial expression, mastication, swallowing, breathing, and neck and limb movements may be involved.

Which of the following is NOT a common side effect of chemotherapy? A. Skin rash B. Myopathies C. Thrombocytopenia D. Ulcers

B. Myopathies Side effects of chemotherapy include anorexia, nausea, vomiting, diarrhea, ulcers, hemorrhage, bone marrow suppression, anemia, leukemia, thrombocytopenia, fatigue, skin rashes, neuropathies, and phlebitis and hair loss. Myopathy is not a side effect of chemotherapy.

A physical therapist consults with the teacher of a nine-year-old child with dyspraxia. Which of the following school-based activities would likely be the MOST challenging for the child? A. Maintaining upright sitting posture in a classroom chair B. Negotiating a crowded hallway between classrooms C. Opening and closing a locker D. Writing their name

B. Negotiating a crowded hallway between classrooms Dyspraxia (developmental coordination disorder) is a common disorder that affects movement and coordination. These children often present with slow movement times, poor motor sequencing, poor motor memory, and perceptual problems. A child with dyspraxia will have difficulty maintaining their balance in environments with challenging surfaces and many obstacles, such as in a crowded hallway. To accommodate for this, a student may be allowed to leave class a few minutes early in order to transition between classrooms in a less crowded hallway.

A 45-year-old female presents to PT with a recent diagnosis of MS. Upon initial evaluation, she complains of eye pain that is worsened with movement, vision loss in one eye, and visual field loss. Which of the following cranial nerves listed below is MOST likely involved? A. Facial (VII) B. Optic (II) C. Abducens (VI) D. Oculomotor (III)

B. Optic (II) Facial nerve looks at the muscles of the eye and how well they can keep their eyes shut when you try to open them so you can rule out A. Abducens innervates the lateral rectus (LR6) so it is motor and moving the eye to the side so you can rule out C. Oculomotor is concerned with the MOTOR aspect of the eye so you can rule that out. The optic nerve is concerned about the sensory aspect so pain will be related to that.

After a stroke, a patient's visual field is as shown in the picture. Which of the following is the MOST LIKELY location of the lesion? A. L occipital lobe B. Optic chiasm C. R optic tract D. L optic nerve

B. Optic chiasm

A 65-year-old male patient has a history of diabetes mellitus for the past 15 years. The patient is unable to hold in urine and states that pelvic floor muscle strengthening is not helpful. What condition does the patient exhibit? A. Stress incontinence B. Overflow incontinence C. Urge incontinence D. Functional incontinence

B. Overflow incontinence If it was stress incontinence pelvic floor strengthening would help so you can rule A out. Nothing was mentioned about infections so you can rule out C. It also did not mention anything about the patient being unable to get to the bathroom so you can rule out D. With overflow incontinence there is an overflow in the bladder (bladder is full) but the patient does not feel the urge to urinate leading to leakage. This is a common side effect of ongoing DM.

Two PT students are reviewing the literature for the effects of cupping technique to increase hamstring flexibility. According to levels of evidence, which studies provide the BEST evidence for support of cupping in increasing the hamstrings flexibility? A. Clinical case report B. Randomized controlled trials C. Clinical case series D. Cross sectional studies

B. Randomized controlled trials The only research that would be higher than this is a systematic review which is not an option. Randomized controlled trials > cross-sectional studies > clinical case series > clinical case report

A patient underwent a surgical repair of a full thickness rotator cuff tear 6 days ago. The PT wants to perform passive and assisted movement. Which position will be MOST appropriate for the patient? A. Patient is sitting with arm abducted 70 degrees with slight flexion B. Patient is supine with arm abducted 45 degrees with slight flexion C. Patient in sitting with arm abducted 15 degrees with slight flexion D. Patient in supine with arm abducted 90 degrees with slight flexion

B. Patient is supine with arm abducted This is early in the recovery so you don't want gravity playing a factor yet. With that you can rule out A and C. 90 degrees too much abduction this early so the answer is B. These answers use the "rule of extremes". When you have four options of numbers line them up in order (15, 45, 70, 90) and more than likely the correct number are one of the middle ones.

Following a stroke, a PT is giving a 62-year-old patient education about positioning in bed. Which is the BEST position to place the lower extremity when lying in supine? A. Pelvis retracted, knee in extension, ankle in neutral with nothing against soles of foot B. Pelvis protracted, knee on small towel roll, ankle in neutral with nothing against soles of foot C. Pelvis retracted, knee on small towel roll with support against soles of feet D. Pelvis protracted, knee in extension, ankle in neutral with support against soles of feet

B. Pelvic protracted, knee on small towel roll, ankle in neutral with nothing against soles of feet Best position in supine for more affected LE following a stroke-The pelvis should be kept in protraction. The hip should be slightly abducted and flexed. The knee should be slightly flexed (to prevent hyperextension) and the ankle should be in neutral with nothing against the soles of feet. Foot splint can be used to maintain ankle in neutral in case of heel cord tightening and break down.

Two trained rescuers are performing CPR on an unresponsive adult with diabetes. They started with compressions, airway clearance, and then breathing. After two minutes, the AED delivered a shock. What is the rescuers' next step? A. Wait for one more shock because the AED delivers two shocks to reset the heart rhythm B. Perform CPR starting with compressions and follow the sequence C. Check the pulse to see if the AED has reset the heart's normal rhythm D. Remove the AED and let it charge again for reuse

B. Perform CPR starting with compressions and follow the sequence As soon as the shock is delivered; CPR should be resumed and continued until prompted by AED to allow rhythm check. Minimize interruptions in chest compressions before and after shock.

Marta is experiencing chronic pain and tenderness in her cervical spine at the level of C5-C6 vertebrae. The physician diagnoses reduced space between the vertebrae. What will be the MOST appropriate physical therapy intervention? A. Provide an ice pack to permanently relieve pain B. Perform cervical mobilization by moving the C5 vertebrae anteriorly C. Perform cervical mobilization by moving the C6 vertebrae anteriorly D. Applying traction at the level of C7

B. Perform cervical mobilization by moving the C5 vertebrae anteriorly Mobilizing upper vertebra i.e. C5 anteriorly increases the space between C5-C6 causing flexion. Traction at C7 level will not be effective. Ice pack will not increase the intervertebral foraminal space.

A therapist documents that a patient presents with UMN signs. Presence of which of the following would cause the therapist to come to this conclusion? A. Positive Babinski, absent cutaneous reflexes, disuse atrophy, presence of synergy B. Positive Babinski, exaggerated cutaneous reflexes, disuse atrophy, presence of synergy C. Positive Babinski, exaggerated cutaneous reflexes, neurogenic atrophy, absence of synergy D. Negative Babinski, absent cutaneous reflexes, disuse atrophy, presence of synergy

B. Positive Babinski, exaggerated cutaneous reflexes, disuse atrophy, presence of synergy Neurogenic atrophy and absence of synergy are characteristics of a LMN lesion so you can automatically rule out C. Everything is greater with an UMN lesion, so hyperreflexia is characteristic. You can rule out A and D because they say reflexes are absent.

A physical therapist examines a patient with a suspected lesion of the common fibular nerve. Which objective finding would be the MOST useful to rule out the possibility of a sciatic nerve lesion? A. Inability to actively dorsiflex the foot B. Preservation of the Achilles reflex C. Presence of a steppage gait D. Weakness of the quadriceps muscle

B. Preservation of the Achilles reflex Absence of the Achilles reflex is characteristic of a tibial nerve or sciatic nerve injury. Preservation of the Achilles reflex means that a sciatic nerve injury could not be present though the common fibular nerve could still be affected since this nerve is not responsible for the Achilles reflex.

A patient who is 6 months pregnant has sharp pain in and around the sacrum and buttocks. The patient reports a recent urinary tract infection treated with antibiotics. Over a 3-week course of physical therapy treatment, the patient's pain does not respond to joint or soft tissue mobilization; however, the patient does report some improvement with the use of external stabilization. Which of the following actions would be MOST appropriate for the physical therapist? A. Initiate postural stability exercises B. Refer the patient to her obstetrician for medical imaging C. Incorporate modalities for pain control D. Refer the patient to an orthotist for a custom pregnancy support belt

B. Refer the patient to her obstetrician for medical imaging The patient's symptoms of continued pain and history of urinary tract infection are consistent with sacroiliitis and require a referral for additional diagnostics.

A patient has right midfoot pain and stiffness. Recent medical history includes treatment for urethritis and conjunctivitis, and blood test results indicate that the rheumatic factor is absent. The patient MOST likely has which of the following conditions? A. Osteoarthritis B. Reiter syndrome C. Psoriatic arthritis D. Rheumatoid arthritis

B. Reiter syndrome Reiter syndrome is a seronegative spondyloarthropathy that presents with asymmetrical extremity arthritis, conjunctivitis, and urethritis

Following surgery of the right hip, a patient ambulates as shown in the picture. As part of the intervention, the PT opts to include FES to help improve the gait pattern. Stimulation should be initiated for: A. Right abductors during swing on the right B. Right abductors during stance on the right C. Left abductors during stance on the left D. Left abductors during swing on the left

B. Right abductors during stance on the right If the patient had surgery on the right hip you will use FES on the right hip. So, you can automatically rule out C and D. Now if the right abductors are weak the deficits will be noted during right stance phase by a left hip drop. Also, you always choose closed-chain strengthening before open-chain.

A young adult underwent right Achilles tendon repair 6 weeks ago and is now able to fully weight bear. The PT is giving him advice on proper show modification. Which of the following would be BEST for the patient to utilize? A. Normal shoes B. Shoes with 1 - 1.5 cm heel lift C. Shoes with lower than regular heel D. Shoes with 5 cm heel lift

B. Shoes with 1 - 1.5 cm heel lift You need to keep your foot in some plantarflexion to not put strain on the healing Achilles tendon so that automatically rules out A and C. Option D is too high of a heel.

A PT examines the gait of a 62-year-old male patient. The patient exhibits right early heel off during stance phase of gait as shown in the picture. Which of the following impairments is MOST likely associated with this finding? A. Shortening of the hamstrings B. Shortening of the gastrocnemius C. Weakness of the tibialis anterior D. Weakness of the iliopsoas

B. Shortening of the gastrocnemius With weakness of the tibialis anterior you would see a foot drop or foot slap. Shortening of the hamstrings would affect the knee or hip. Weakness of the iliopsoas would affect the hip.

A physical therapist works with a patient who sustained burns to the left axilla and shoulder. The therapist would most likely fabricate a splint for the patient in order to avoid which of the following deformities? A. Shoulder abduction contracture and webbing of the axillary folds B. Shoulder adduction contracture and webbing of the axillary folds C. Shoulder flexion and medial rotation contracture D. Thoracic kyphosis secondary to flexion forces

B. Shoulder adduction contracture and webbing of the axillary folds Imagine the burns shortens tightens the skin on the shoulder and armpit which pulls the arm in. You want to avoid this by splinting the arm into shoulder abduction and lateral rotation.

During evaluation, the PT asks the patient with a diagnosis of CVA to lift her left arm as high as she is able. The patient can lift her arm off her lap; however, she is only able to move it with elbow flexion, shoulder elevation, and wrist flexion with significant increased tone. Which of the following Brunnstrom's Stages of Recovery is the MOST appropriate description for this patient? A. Stage 1 B. Stage 3 C. Stage 5 D. Stage 6

B. Stage 3 Stage 1: flaccidity, no active movement Stage 3: voluntary control of movement synergy, peak of spasticity Stage 5: complex movement, greater independence from limb synergies Stage 6: individual joint movement, coordinated movement

A physical therapist is designing an independent home program for a patient who has a 10-year history of recurrent low back pain. The goal of the program is to reduce the recurrence rate and improve the patient's function. Which of the following recommendations is MOST appropriate? A. Rest whenever pain is increased B. Start an aerobic conditioning program C. Place an order for a home traction unit D. Use a lumbar brace regularly

B. Start an aerobic conditioning program Aerobic exercise combined with specific strengthening may decrease the frequency of low back pain recurrence

A 44-year-old male patient is being evaluated by a physical therapist. The patient underwent a hernia repair 3 weeks ago and should AVOID which of the following activities? A. Walking at a metabolic equivalent of 4 three weeks after surgery B. Stretching of the anterior spinal and hip musculature before the incision is fully healed C. Stretching of the posterior spinal and hip musculature before the incision is fully healed D. Wall sits performed in an upright position

B. Stretching of the anterior spinal and hip musculature before the incision is fully healed Hernias are on the anterior part of the body so you need to avoid stretching that to help with healing!

A 29-year-old patient presents to the clinic with pain on the lateral aspect of the shoulder. The physical therapist performs a manual muscle tests and finds weak shoulder abductors (3+/5). Based on patient presentation, which of the following nerve is MOST likely affected? A. Long Thoracic nerve B. Suprascapular nerve C. Thoraco-dorsal nerve D. Median nerve

B. Suprascapular nerve The suprascapular nerve provides the motor supply to the supraspinatus and infraspinatus muscles which help in shoulder abduction movement. So, if this nerve is injured the patient can have weak shoulder abduction.Long thoracic nerve supplies Serratus Anterior muscle. Thoraco-dorsal nerve supplies Latissimus dorsi.

A physical therapist monitors the vital signs of a patient running on a treadmill at a series of steadily increasing speeds. A change in which variable would be MOST responsible for an observed increase in pulse pressure during the exercise session? A. Heart rate B. Systolic blood pressure C. Diastolic blood pressure D. Cardiac output

B. Systolic blood pressure Systolic blood pressure is the maximum arterial pressure during systole. Systolic pressure initially increases with exertion in a linear progression. The relative increase in systolic blood pressure, combined with stable diastolic blood pressure, results in an increased pulse pressure.

A rock climber injured his right foot and right hand. He comes to a PT clinic and reports a bowstring injury. Which of the following structures are most likely to be injured? A. The Cruciform pulleys B. The Annular pulleys C. Lumbrical muscles D. Wrist extensors crossing the elbow

B. The Annular pulleys Annular pulleys along with other flexor pulleys share a similar function of holding the underlying tendons at a relatively close distance to the joints. With injury to these structures, the force of a strong contraction of the extrinsic finger flexors causes the tendon to pull away from the joint's axis of rotation, a phenomenon referred to as "bowstringing" of the tendon.

A patient with T4 paraplegia is learning how to transfer his wheelchair up onto a curb by doing a wheelie. Which of the following verbal cues is LEAST helpful to complete this task? A. Lean backward initially B. Throw your weight forward as you advance the wheels forward C. Place your hands at the 11 o'clock position for initial forward propulsion D. Maintain the center of gravity over the rear wheels

B. Throw your weight forward as you advance the wheels forward Throwing the weight forward as we advance the wheel is not necessary. First step involves leaning back. After the initial propulsion, maintaining the center of the body over the rear wheel is essential. Placing the hands at 11'0 clock position assists in the initial forward propulsion.

A PT is treating a patient with Epley maneuver starting with the head in 30 degrees of extension and right ear towards the ceiling. Which of the following tests is MOST likely expected to be positive for the therapist to choose this intervention? A. Torsional upbeating nystagmus with right dix hallpike test B. Torsional upbeating nystagmus with left dix hallpike test C. Geotropic nystagmus on supine roll test D. Ageotropic nystagmus on supine roll test

B. Torsional upbeating nystagmus with left dix hallpike test Epley maneuver is used to treat canalisthiasis and typically posterior BPPV. With posterior BPPV you will see a torsional upbeating ("post up bruh"). The ear being treated will begin facing the ground so you are treating the left ear.

Documenting the care provided to the patient is essential and must be completed in a timely manner. Which of the following is NOT appropriate with respect to documentation? A. When a charting error is made, it must be clearly indicated that a change was made without deleting the original record B. When a charting error is made, use white-out material to correct the text C. Mistakes must be crossed out with a single line through the error and then both initialed and dated by the therapist D. Medically approved symbols or abbreviations can be used for documentation

B. When a charting error is made, use white-out material to correct the text

Prior to initiating an ultrasound treatment a physical therapist formally measures the target area as 12 cm^2. Based on the therapist's measurements, what is the MOST appropriate size of soundhead to utilize during the treatment? A. 1 cm^2 B. 3 cm^2 C. 5 cm^2 D. 8 cm^2

C. 5 cm^2 Typically, you use a head that is approximately half the size of the target area. So, for this half of the area would be 6 cm^2. Therefore, 5 cm^2 is the closest to this value. 5 and 10 cm^2 are the most common sizes.

A physical therapist would like to clear the airway secretions of a patient who is graded stage III per the Ranchos Los Amigos scale. Which of the following techniques is LEAST APPROPRIATE for this patient? A. Gravity assisted positioning B. Percussions C. Activity cycle of breathing techniques D. Vibrations

C. Activity cycle of breathing techniques The patient is not in a state where they can being doing active exercises yet.

A patient diagnosed with Cushing's syndrome is referred to physical therapy. Which of the following signs and symptoms is NOT consistent with this syndrome? A. Distension of the abdomen B. Swelling in the facial area C. Adrenal hypoplasia D. Cardiac hypertrophy

C. Adrenal hypoplasia Cushing's syndrome is a condition characterized by HYPERfunction of the adrenal cortex. Addison's disease is characterized by HYPOplasia of the adrenal cortex.

A 40-year-old male was admitted to an acute rehabilitation unit with neurological impairments, including altered mental status, dysarthria, and motor weakness. During the objective examination, the PT found that the patient also elicited hyperreflexia. What clinical condition is MOST appropriate considering the above findings? A. Cervical myelopathy B. Myasthenia gravis C. Amyotrophic lateral sclerosis D. Multiple sclerosis

C. Amyotrophic lateral sclerosis ALS is a motor neuron disease characterized by the degeneration and loss of motor neurons in the spinal cord, brainstem, and brain, resulting in a variety of UMN and LMN signs and symptoms. LMN signs - muscle weakness, hyporeflexia, hypotonicity, atrophy, muscle cramps, fasciculations, UMN signs- spasticity, hyperreflexia, muscle weakness, Bulbar signs - dysphagia, dysarthria, sialorrhea, pseudobulbar affect due to the involvement of cranial nerves 5,7,9,10,12. Cervical myelopathy will not be associated with dysarthria, spasticity and cognitive impairment. Myasthenia gravis is not associated with spasticity and cognitive impairment. Multiple sclerosis has sensory impairment present along with the other UMN signs and motor weakness.

Based on the mechanism of injury shown in the photograph, which of the following tests should a physical therapist perform to confirm the suspected diagnosis? A. Fibular translation test B. Lateral (external) rotation test C. Anterior drawer test of the ankle D. Distal tibiofibular compression test

C. Anterior drawer test of the ankle The photograph shows an inversion sprain of the ankle. Inversion sprains can cause injury to the anterior talofibular ligament, which can be tested by the anterior drawer test of the ankle. (p. 1132)

Which of the following scenarios BEST describes the effect of climatic conditions on an individual who has exercise-induced asthma? A. Bronchospasm is facilitated by exercise in a humid environment, compared with a dry environment B. Bronchospasm is facilitated by exercise in a warm environment, compared with a cold environment C. Bronchospasm is blunted when exercising in a humid environment, compared with a dry environment D. Bronchospasm is blunted when exercising in a cold environment, compared with a warm environment

C. Bronchospasm is blunted when exercising in a humid environment, compared with a dry environment Exercise-induced asthma or bronchospasm is exacerbated in cold and dry environments and is blunted when exercising in a humid environment.

A patient sustained a proximal humerus fracture that is non-displaced. Which of the following clinical findings would provide the BEST support for the patient being cleared to perform active-assisted exercise? A. Hematoma formation B. Diminished pain C. Callus formation D. Remodeling

C. Callus formation Callus formation is one of the first indications that healing has occurred. The presence of a callus identified through diagnostic imaging allows the patient to progress to active-assisted exercise.

What is the MOST appropriate method to confirm an L4 nerve root lesion? A. Check sensation over lateral foot B. Check Achilles reflex C. Check sensation over great toe D. Check medial hamstring reflex

C. Check sensation over great toe L4 dermatome is medial aspect of foot, L5 is dorsum of foot and S1 is lateral aspect of foot. Few dermatomal diagrams depict great toe is supplied by L4. For reflexes: L5, S1- medial hamstrings reflex, S1, S2- Achilles reflex

A 65 year old patient with diabetic neuropathy presents to the clinic with a complaint of frequent falls especially at night. Which of the following conditions of the Sensory Organization Test is this patient MOST LIKELY to score poorly? A. Conditions 2 and 4 B. Conditions 1 and 3 C. Conditions 5 and 6 D. Conditions 4 and 6

C. Conditions 5 and 6 Due to peripheral neuropathy, this patient is more dependent on their vision and vestibular systems. Patients depending on vision become unstable in conditions 2, 3, 5 & 6 where we either close the eyes, or have a conflict between vision and the vestibular system. With conditions 1 and 4, the patient will have the opportunity to rely on the visual system and maintain balance.

A patient complains of the inability to slip the belt through the belt loops at the back of his pants. He is unable to reach his right hand to the left side on the back to complete the task. Which of the following mobilization techniques would be MOST APPROPRIATE to complete this task? A. Glenohumeral caudal glide B. Glenohumeral anterior glide C. Glenohumeral posterior glide D. Glenohumeral superior glide

C. Glenohumeral posterior glide The shoulder joint follows the concave-convex rule. The convex humerus moves on the concave glenoid fossa. To be able to reach the back of the pant on the opposite side, the IR range has to be improved. Thus, posterior glide of the GH joint is appropriate.

A PT examines a pt complaining of tingling into the 4th and 5th digits with muscle wasting over the hypothenar eminence. The PT suspects ulnar neuropathy and decides to examine the integrity of the nerve. Which of the following testing procedures would be the best? A. Have the pt flex both wrists while holding them for one minute B. Have the pt make a fist around the thumb and perform ulnar deviation C. Have the pt grasp a piece of paper between their first and second fingers while the examiner pulls the paper and monitors the first finger D. Have the pt perform extension of the 3rd digit of the hand against examiner resistance

C. Have the pt grasp a piece of paper between their first and second fingers while the examiner pulls the paper and monitors the first finger This is a test for the Froment's sign which checks the ulnar nerve. If the pt flexes their thumb when resisting the pull they may have a problem with the ulnar nerve.

A physical therapist is facilitating an in-service on BLS by educating on the Do's and Don'ts of adult high-quality CPR. Which of the following SHOULD the rescuer do during the training? A. Include pauses between compressions for greater than 10 seconds B. Compress chest to depth of less than 2 cm C. Minimize pauses in between compressions D. Compress chest at a rate of 123/min

C. Minimize pauses in between compressions Compress the chest to a depth of AT LEAST 2 cm. Compress the chest at a rate of 100-120/min.

A patient is being treated for neck and shoulder pain by a physical therapist. The physical therapist notes that the patient has a forward head, rounded shoulders posture that cannot be corrected with active movement. Which of the following interventions is MOST likely to correct the problem? A. Initiate cervical flexor and extensor strengthening and progress as tolerated B. Perform thrust manipulation to the midthoracic spine and apply modalities as needed C. Initiate pectoralis minor stretching and scapular retractor strengthening and progress as tolerated D. Initiate cervical traction to reduce the stress on the cervical spine and begin a program of active range of motion movements for the cervical and thoracic spine

C. Initiate pectoralis minor stretching and scapular retractor strengthening and progress as tolerated Treatment of forward head posture includes strengthening of the muscles that have become inhibited and stretching of the facilitated muscles. Stretching of the pectoralis minor muscle will reduce the protraction improving the rounded posture and strengthening of retractors will help maintain the retracted shoulder posture.

A patient 1 month post myocardial infarction is being seen in a clinic. When increasing the patient to 5 MET's of workload, the therapist notes that the ECG reads 1 unifocal PVC. The PT's IMMEDIATE action should be: A. Continue to exercise and increase the intensity B. Stop exercise because patient is undergoing ischemia C. Keep exercising at a lower intensity, consultation with physician is not required here D. Stop exercise and consult with physician before starting any exercise

C. Keep exercising at a lower intensity, consultation with physician is not required here Increasing the intensity when you notice something abnormal makes no sense so you can rule out A. The question states the patient is post myocardial infarction NOT ischemia so you can rule out B. 1 unifocal PVC is not a major concern so you can continue to exercise at a lower intensity.

A 55-year-old male was admitted with a diagnosis of head trauma. Upon further examination, patient seems to respond to simple commands consistently, responses are not purposeful, random or fragmented as well as verbalization appearing to be inappropriate at times with confabulatory present. The PT would MOST likely classify this patient on the LOCF as: A. Level VI B. Level IV C. Level V D. Level III

C. Level V LOCF= Ranchos Los Amigos Level of Cognitive Functioning scale Level III: localized response Level IV: confused-agitated Level V: confused, inappropriate, non-agitated Level VI: confused, appropriate

A 48-year-old male patient reports of SOB and swelling in LE's. During the baseline examination, the PT examines patient's heart sounds before starting an exercise program. Which valve is being auscultated at #5 in the picture? A. Tricuspid valve B. Pulmonary valve C. Mitral valve D. Aortic valve

C. Mitral valve "All Physical Therapists Make $2245 per week" Aortic- 2nd intercostal on R Pulmonary- 2nd intercostal on L Tricuspid- 4th intercostal on L Mitral- 5th intercostal on L

A PT is looking at squat activity in 20 obese and 20 normal weight females. Both groups performed 10 bilateral squats at 100-degrees of knee flexion. The PT hypothesizes that knee moments will be higher in obese. Which is the MOST appropriate test? A. Chi-Square test B. Mann-Whitney U test C. One tailed t-test D. Two-tailed t-test

C. One tailed t-test This is looking at two DIFFERENT groups of EQUAL size which is characteristic of a parametric test. The T-test is a parametric test. If there is a hypothesis then the test is a ONE tailed t-test (no hypothesis with two-tailed).

A 40-year-old male patient presents with dyspnea upon exertion and low endurance during ambulation and activities of daily living. What is the BEST breathing technique to enhance this patient's clinical presentation? A. Diaphragmatic breathing B. Lateral costal breathing C. Paced breathing D. Pursed lip breathing

C. Paced breathing Paced breathing is indicated when a person becomes dyspneic during an activity. In this, by breaking activities down into component parts and interspersing rest periods between each component, the total activity can be completed without dyspnea or undo fatigue. Pursed lip breathing is used to decrease respiratory, reduce dyspnea and facilitate relaxation for patients with COPD. Lateral costal breathing is used to expand one segment of the lung. Diaphragmatic breathing is used for relaxation.

A 49-year-old patient presents to a clinic with swollen legs. More swelling is seen in the right LE than the left LE. The PT suspects lipedema as a diagnosis. Which of the following is NOT likely seen as the patient's presentation? A. Patient is susceptible to bruising of the affected area B. Patient would have a negative Stemmer sign C. Patient has a high likelihood of developing cellulitis D. Patient would report pain on pressure

C. Patient has a high likelihood of developing cellulitis Patients with LYMPHedema have a high likelihood of developing cellulitis

A physical therapist is assessing muscle strength of a 20-year-old male who reports having shoulder pain ever since he started lifting weights at the gym as part of his New Year's resolution. The therapist positions the patient in the supine position with 120 degrees of shoulder abduction and moves the shoulder diagonally down and inward towards the patient's opposite hip. Resistance is given above the wrist in an up and outward direction. Which muscle is being tested? A. Pectoralis major clavicular head B. Pectoralis major both heads C. Pectoralis major sternal head D. Pectoralis minor

C. Pectoralis major sternal head MMT of Pectoralis major sternal head - the motion begins at 120° of shoulder abduction and moves diagonally down and in toward the patient's opposite hip. Resistance is given above the wrist in an up and outward direction

A patient post CVA requires an orthosis due to occasional dragging of the toe during the swing phase of gait The patient presents with weakness of the dorsiflexors and has good medial/lateral stability at the ankle. Which of the following orthotic options is the MOST appropriate for this patient? A. Solid ankle-foot orthosis B. Tone reducing foot orthosis C. Posterior leaf spring orthosis D. Custom articulating ankle-foot orthosis with anterior trim lines

C. Posterior leaf spring orthosis A posterior leaf spring orthosis is a type of ankle-foot orthosis that provides a DF assist during swing phase. The PLS orthosis is designed with flexibility so that both DF and PF can occur during the gait cycle. The PLS allows for improved biomechanics during gait secondary to its flexibility and therefore would be the most appropriate type of orthosis for the patient given their level of medial/lateral stability.

A physical therapist orders a wheelchair for a patient who has C4 tetraplegia. Which wheelchair would be the MOST appropriate for the patient? A. Manual wheelchair with friction surface handrims B. Manual wheelchair with handrim projections C. Power wheelchair with sip and puff controls D. Power wheelchair with joystick controls

C. Power wheelchair with sip and puff controls This person would have very little UE innervation so there is no way they can self-propel a manual chair. You can rule out A and B. Anything above a C5 will not use the UE's!!!

A physical therapist works with a patient who has been instructed to take non-steroidal anti-inflammatory drugs (NSADs) to help control the symptoms of arthritis. The therapist educates the patient that overuse of NSAIDs can result in gastrointestinal damage. This side effect is caused by the inhibition of the production of which hormone? A. Angiotensin B. Erythropoietin C. Prostaglandins D. Gastrin

C. Prostaglandins Prostaglandins are a group of hormones that help protect the lining of the stomach by inhibiting gastric acid secretion and increasing the production of mucous in the stomach lining. NSAIDs inhibit the production of these protective prostaglandins, which results in the stomach becoming more susceptible to damage from the gastric acids.

A PT is treating an adolescent patient who has severe ankle pain. The patient has been advised by the physician to rest the ankle, but the patient's dad insists that the patient play in an upcoming basketball game. Which of the following actions is MOST appropriate for the PT? A. Discuss concerns about the patient's condition with the patient's mother B. Discuss concerns about the patient's condition with the patient's basketball coach C. Report concerns about the patient's treatment adherence to the referring physician D. Report the concerns to the State Physical Therapy Board

C. Report concerns about the patient's treatment adherence to the referring physician Automatically rule out B because that is a violation of HIPAA.

A 58-year-old male underwent a total hip arthroplasty on the left side using a postero-lateral approach. The PT is working on a gait training program for the patient. The therapist should instruct the patient to hold the crutch or cane in the: A. Left hand to decrease activity in the left hip abductors B. Left hand to facilitate activity in the left hip abductors C. Right hand to decrease activity in the left hip abductors D. Right hand to facilitate activity in the left hip abductors

C. Right hand to decrease activity in the left hip abductors The cane is almost always held on the unaffected side so you can rule out A and B. The muscle on the left side is trying to heal so you are shifting the weight to the right side to reduce the amount of stress on it.

A patient with an above knee prosthetic limb is displaying right lateral trunk bending while ambulating. Which of the following would be the MOST likely cause of this gait abnormality? A. Right anterior wall is too high B. Right medial wall too low C. Right lateral wall too low D. Right posterior wall is too high

C. Right lateral wall too low Stick to your plane!!! You can automatically rule out A and D. When you think of a right lateral trunk lean that tells you that there is a left pelvic drop that you are correcting for. So, there is either tightness in the left abductors or weakness in the right abductors. If a wall is low that correlates with muscle weakness. So a low wall on the lateral side would correlate with weak abductors. Anterior wall too high- tight hip flexors Medial wall too low- weak adductors Lateral wall too low- weak abductors Posterior wall too high- tight hip extensors

A 39-year-old comes to an outpatient clinic complaining of hip problems. The PT notices a drop of the left hip during right midstance. The MOST LIKELY cause of this impairment can be the injury of: A. Right inferior gluteal nerve B. Right femoral nerve C. Right superior gluteal nerve D. Right obturator nerve

C. Right superior gluteal nerve This will be due to weak gluteus medius on the R side. The superior gluteal nerve innervates the gluteus medius. The inferior gluteal nerve innervates the gluteus maximus.

A patient recently underwent a radical lymph node dissection secondary to prostate cancer. According to the patient's history, what is the MOST important for the physical therapist to educate the patient on? A. Routinely perform volumetric measurements of right lower extremities B. Routinely use a home compression pump with pressure >80 mm Hg C. Routinely monitor any changes in the fitting of their socks or shoes D. Routinely perform circumferential measurements of both lower extremities

C. Routinely monitor any changes in the fitting of their socks or shoes Lymphedema happens following a lymph node dissection. Checking the fitting of their socks is more functional and educational. The patient is also more qualified to do this than the other options.

A physical therapist treats a 30-year-old individual who was admitted to the hospital with insidious respiratory issues and small, red granulomas on their face, particularly around their mouth. These clinical findings are typical of which of the following conditions? A. Systemic sclerosis B. Bronchiectasis C. Sarcoidosis D. Phenylketonuria

C. Sarcoidosis Sarcoidosis occurs primarily between the ages of 20-40 years. Women > men. African Americans > Caucasians. The impact of sarcoidosis is dependent on the magnitude and region of the granulomas. Prognosis is typically favorable; however, there are some instances where it can be life-threatening.

A physical therapist treats a patient with a colostomy that is capable of producing solid stool on a fairly regular schedule. Which type of colostomy would be the MOST consistent with this description? A. Ascending B. Descending C. Sigmoid D. Transverse

C. Sigmoid The sigmoid colon is the final portion of the large intestine and serves as a connection to the rectum. A sigmoid colostomy is the most common type of colostomy, located a few inches lower than a descending colostomy. As a result, this type of colostomy has additional working colon. A sigmoid colostomy produces normal stool consistency and discharge can be regulated. Ascending- primarily liquid Transverse- soft or loose stool Descending- firm but somewhat irregular Sigmoid- normal stool consistency

Which of the following is MOST appropriate to screen for in a patient with an acute exacerbation of Guillain-Barre Syndrome? A. Fatigue B. Spasticity C. Skin integrity D. Respiratory function

C. Skin integrity Skin integrity is most important to check as loss of sensation is dangerous. Fatigue is common, but will not be as detrimental to patient. Respiratory function will be preserved initially. Spasticity is not seen with GBS.

A patient has medial ankle pain, a pronated foot with a calcaneal valgus deformity, pain with passive ankle eversion, and weakness of the great toe flexors. The patient MOST likely has which of the following conditions? A. Common fibular (peroneal) nerve injury B. Deltoid ligament strain C. Tarsal tunnel syndrome D. Tibialis posterior tendinitis

C. Tarsal tunnel syndrome Pain only with passive ankle eversion, a pronated foot, valgus deformity, and weak toe flexion strength are associated with tarsal tunnel syndrome

A 58-year-old patient is lifting a wine box using a stoop lift at a winery. Which of the following is the MOST significant factor in increasing compression forces on the spine in addition to the weight of the wine box? A. The width of the wine box B. Performing the lift with the lumbar spine in a neutral position C. The distance of the wine box from the base of the spine D. The muscle strength of the lower extremities

C. The distance of the wine box from the base of the spine As the wine box moves further away from the body there will be greater compressive forces on the spine. Think of it like a moment arm, the higher the moment arm the greater the force.

Which of the following positions would MOST accurately place a joint in a loose-packed position? A. Hip placed in 0 degrees of abduction B. Glenohumeral joint placed in 15 degrees of abduction C. Tibiofemoral joint placed in 25 degrees of flexion D. Humeroulnar joint placed in 30 degrees of flexion

C. Tibiofemoral joint placed in 25 degrees of flexion Loose-packed positions: Hip: 30 degrees of flexion Glenohumeral joint: 55 degrees of abduction Tibiofemoral joint: 25 degrees of flexion Humeroulnar joint: 70 degrees of flexion

A teenager with idiopathic scoliosis has been wearing a well-fitting thoracolumbar orthosis. Brace tolerance is good, but the patient stands with the trunk flexed forward on the lower extremities. This standing posture is MOST likely caused by which of the following conditions? A. Excessive lumbar lordosis B. Shortened hamstrings C. Tight hip flexors D. Leg length discrepancy

C. Tight hip flexors Tight hip flexors can cause trunk flexion because the orthosis prevents lumbar lordosis and anterior pelvic tilt from occurring

You are treating a patient with spastic cerebral palsy in outpatient physical therapy clinic. The patient has difficulty with ambulation in the community and the patient's mother reports concern about falling. When quickly extending the right knee joint, there is a catch followed by minimal resistance through the rest of the remaining range (about 25% of the total range of motion). Which of the following statements is MOST ACCURATE? A. 3 on the Modified Ashworth Scale and increased tone in the hamstrings B. 3 on the Modified Ashworth Scale and increased tone in the quadriceps C. 1 on the Modified Ashworth Scale and increased tone in the quadriceps D. 1+ on the Modified Ashworth Scale and increased tone in the hamstrings

D. 1+ on the Modified Ashworth Scale and increased tone in the hamstrings The Modified Ashworth Scale is a tool used to grade spasticity. A slight increase in muscle tone manifested by a catch, followed by minimal resistance throughout the remainder of movement (less than half of the range of motion) is classified as 1+.

Patient underwent incident of myocardial infarction 5 weeks ago. PT has to give strength training to the patient. Which is the MOST appropriate protocol that can be given to a post MI patient at 5 weeks? A. PT should wait 2 more weeks for strength training B. Start using 5 lbs free weights or dumbbells with UE and LE training C. Use 5-10 lbs for UE and 10-15 lbs for LE D. 1-3 lbs with light weights and elastic bands

D. 1-3 lbs with light weights and elastic bands After 5 weeks post MI begin with light weight 1-3 lbs and elastic bands for 12-15 reps

A PT is treating a patient that is four weeks post CHF in a cardiac rehabilitation unit. While monitoring the exercise session and EKG the therapist observed the following rhythm. Upon asking the pt how he is doing the patient replies with "I'm doing fine". What should the therapist suspect here and what would be the therapist's immediate action? A. 2nd degree Wenckebach rhythm, stop exercise and alert emergency system B. Bradycardia, stop exercise immediately and make the patient rest C. Prolonged PR segment, 1st degree block, continue exercising without any modification D. 3rd degree AV block, activate emergency system

D. 3rd degree AV block, activate emergency system There is no consistency between atrial and ventricular contractions. This patient is at a high risk of arrest. Immediate medical intervention is warranted.

A researcher is collecting data on 100 tennis players with overhead injuries. Out of the 100 patients with rotator cuff injuries, 60 patients had a positive special test and 40 had a negative special test. What is the sensitivity of the special test? A. 20% B. 80% C. 40% D. 60%

D. 60% Sensitivity = TP/(TP + FN) Sensitivity = 60/(60 + 40)= 60%

A 50-year-old male patient with a secondary-degree heart block (Mobitz type I Wenckebach) presents his ECG report to the physical therapist. What would a physical therapist expect to find on his EKG? A. An increase in PR interval lengths B. No relationship between P waves and QRS complexes C. Normal PR intervals in all the beats preceding a dropped beat D. A gradual increase in PR interval length in all the beats preceding a dropped beat

D. A gradual increase in PR interval length in all the beats preceding a dropped beat A- first degree B- third degree C- second degree Mobitz type II

A physical therapist treats a patient with a sacral pressure ulcer by applying a foam dressing impregnated with charcoal. This type of dressing would be MOST beneficial in treating which type of wound? A. A dry wound that is infected B. A dry wound that has a fetid odor C. A heavily exuding wound that is infected D. A heavily exuding wound that has a fetid odor

D. A heavily exuding wound that has a fetid odor A foam dressing would be beneficial for treating a heavily exuding wound due to its absorptive properties. The charcoal in the dressing is beneficial for decreasing fetid odors associated with the wound.

A PT examines a patient who complains of foot pain while running. The examination shows that the patient has excessive forefoot pronation. Which of the following would be the MOST appropriate orthotic insert? A. A lateral forefoot post under the fifth metatarsal head B. A lateral rearfoot post under the calcaneus placing it in an everted position C. A middle post just proximal to the third metatarsal head D. A medial post just proximal to the first metatarsal head

D. A medial post just proximal to the first metatarsal head

All of the following are components of treatment for Phase I lymphedema EXCEPT: A. Decongest the trunk quadrants first before addressing the lymphedematous extremity B. Exercise should always be performed with a compression bandage C. Decongesting the proximal portions of the limb first and then working distally with the direction of flow always towards the trunk D. Affected limbs are bandaged with high-stretch compression bandages

D. Affected limbs are bandaged with high-stretch compression bandages Low (short)-stretch compression bandages are used for lymphedema!!!

A physical therapist is observing a 67-year-old female patient walk and immediately notes a forward trunk lean. A LIKELY contributing cause for this deviation is: A. Excessive ankle dorsiflexion B. Quadriceps spasticity C. Hip extensor contracture D. Ankle plantar flexion contracture

D. Ankle plantar flexion contracture Individuals with an ankle plantar flexion contracture will make initial contact with the ground with the forefoot region. At mid stance, bringing the heel to the ground will result in knee hyperextension. Forward lean of the trunk occurs in terminal stance as a strategy to maintain forward progression of the center of mass.

During a patient interview, the physical therapist becomes concerned that a patient might be contemplating suicide based on several verbal statements. The patient was recently prescribed a new medication to treat depression. Which of the following actions would be the MOST appropriate initial step by the therapist? A. Remind the patient it normally takes a period of time for this type of medication to be helpful B. Use the impact suicide would have on the patient's family as a deterrent C. Recommend the patient make an appointment to return to the physician D. Ask questions to determine if the patient is considering suicide

D. Ask questions to determine if the patient is considering suicide Asking questions to determine if the patient is considering suicide is recommended to determine the seriousness of the concern for suicide. Health professionals should not hesitate to ask whether a person is considering suicide or even if the patient has a plan. The QPR model for suicide prevention involves "Q" (question the person about suicide), "P" (persuade the person to get help), and "R" (refer).

A physical therapist would be required to wear a surgical face mask when working on transfer training with a patient who has which of the following conditions? A. Rubeola B. Tuberculosis C. Varicella zoster D. Bacterial pneumonia

D. Bacterial pneumonia A-C are all airborne precautions which would require a particulate respirator.

A 50-year-old male fainted in the office and was transferred to the ER for further evaluation. The medical history includes DM II and hypertension. The arterial blood gas results are as follows: Blood pH- 7.43, PaCO2- 28 mmHg, and HCO3- 18 mEq/L. This is known as: A. Partially compensated respiratory alkalosis B. Uncompensated respiratory alkalosis C. Compensated metabolic alkalosis D. Compensated respiratory alkalosis

D. Compensated respiratory alkalosis pH is normal (7.35-7.45) PaCO2 is abnormal (35-45) HCO3 is abnormal (22-26) For partially compensated all three values should be abnormal so you can rule out A. If it is uncompensated then pH would be abnormal so rule out B. PaCO2 is 7 off of the normal value and HCO3 is 4 off of the normal value. Since the PaCO2 is more off that means it is compensated RESPIRATORY alkalosis. IT IS ALWAYS COMPENSATED IF THE pH IS NORMAL

A note in the medical record indicates that a patient was recently prescribed Lasix (furosemide). Which of the following medical conditions is MOST commonly associated with the use of this medication? A. Atrial flutter B. Deep vein thrombosis C. Hyperlipidemia D. Congestive heart failure

D. Congestive heart failure Lasix is a medication that is commonly prescribed to help lessen the edema associated with chronic heart failure

A physician is assessing how well a baby is doing immediately after birth and how well the newborn is doing outside the womb. Which of the following findings indicate the BEST result? A. Crying baby, HR <100 bpm, some muscle tone, grimacing present, pink body with blue extremities B. Entire body is pink, active motion of extremities, grimacing present, HR <100 bpm, irregular and slow respiration C. Crying baby, HR <100 bpm, pink body, flexed arms and legs, grimacing present and a cough D. Crying baby, HR >100 bpm, pink body and blue extremities, prompt response to stimulus, active movements of extremities

D. Crying baby, HR >100 bpm, pink body and blue extremities, prompt response to stimulus, active movements of extremities Use the Apgar Scoring for this Normal child: hands and feet are pink; HR >100 bpm; pulls away, sneezes, coughs, or cries with stimulation; active, spontaneous movement; normal RR and effort, good cry

A PT is examining a 46-year-old patient who underwent a radical mastectomy with axillary node removal. Which of the following signs would indicate initial development of lymphedema in the patient? A. Non-pitting edema of lower leg and foot B. Shallow wound beds on the forearm C. Atrophy of the biceps muscle D. Decreased flexibility of the fingers

D. Decreased flexibility of the fingers Non-pitting edema happens further along in the process so you can rule out A. You may see atrophy in the muscles eventually but it will not be early on so you can rule out C. B would not be impacted by lymphedema. Warmth, decreased flexibility in the fingers, and PITTING edema would be early signs.

A 74-year-old individual reports experiencing increased urinary incontinence over the past year. What physiological change is MOST commonly associated with this condition in older adults? A. Reduced kidney filtration capacity B. Increased reservoir capacity of the bladder C. Spasm of the detrusor muscle D. Decreased urge sensation

D. Decreased urge sensation Decreased urge sensation is one of the leading reasons for incontinence in older adults. The bladder becomes full, but due to decreased bladder sensitivity the older adult may not recognize this and as a result experiences episodes of incontinence.

A patient presents to the clinic with complaints of knee pain. On evaluation the PT notices that the patient has excessive ankle plantar flexion along with excessive anterior pelvic tilt. What could be the correlated motion at knee joint? A. Genu valgum B. Genu varum C. Medial tibial torsion D. Genu recurvatum

D. Genu recurvatum Stick to the plane! The only option that is within the correct plane is D! Anterior pelvic tilt is equivalent to a forward lean.

The shoulder is the most mobile joint in the body, but because of its wide range of movements it is one of the most commonly injured. Which of the following is the MOST commonly dislocated joint in the shoulder? A. Acromioclavicular joint B. Sternoclavicular joint C. Scapulothoracic joint D. Glenohumeral joint

D. Glenohumeral joint Glenohumeral joint is the most mobile joint. It is a multiaxial, ball-and-socket, synovial joint that depends primarily on the muscles and ligaments rather than bones for its support, stability. It has high mobility but little articular stability making it the most commonly dislocated joint of the shoulder complex.

A male patient with a BMI of 38 kg/m^2 presented to an outpatient clinic for a regular visit. The physical therapist observes ichthyosis on the lower extremities as shown in the picture. What would be the MOST likely cause in this patient? A. Addison's disease B. Graves disease C. Cushing's syndrome D. Hashimoto's disease

D. Hashimoto's disease Ichthyosis is dry, itchy, "fish-scale" skin. Graves disease is a hyperthyroid issue which is characterized by SWEATY skin and weight LOSS (a hyper person is moving so much they are sweaty and lose weight). You can rule out B. Cushing's syndrome ("cushion's syndrome") is a hyper-adrenal condition characterized by everything getting bigger and soft NOT hard and dry so you can rule that out. Addison's disease is a hypo-adrenal condition characterized by weight loss (opposite of Cushing's) so they would not be obese. That leaves you Hashimoto's disease. Hashimoto's disease is a hypothyroid disease characterized by weight gain and dry skin.

A physical therapist assesses a patient's sensation of light touch in the lower extremities. The therapist documents that the patient has impaired sensation of the entire right leg, yet normal sensation of the left leg. Which injury is MOST likely present? A. Compression of a lower extremity peripheral nerve B. Compression of a lumbar nerve root C. Spinal cord transection D. Left hemisphere stroke

D. Left hemisphere stroke A and B would only impact a small area of the skin, not the whole leg. A PARTIAL spinal cord transection would be a plausible answer; however, if there is a complete spinal cord transection like this answer you would have impaired sensation bilaterally. Patients status post stroke often have impairments of superficial and/or deep sensations. A patient with a left hemisphere lesion will have sensory impairments on the right side of the body and can involve the face, upper extremity, and lower extremity.

Lymphedema occurs due to disruption of the normal circulation of lymphatic fluid in the body. All of the following are causes of secondary lymphedema EXCEPT: A. Infection by Wuchereria Bancrofti B. Kaposi's sarcoma C. Traumatic injuries D. Milroy's disease

D. Milroy's disease Wuchereria Bancrofti is a parasitic worm that infects the lymphatic system and causes lymphatic filiaris. Milroy's disease is a congenital abnormality of the lymphatic system that is characterized by lymphedema in the legs. Primary: congenital or hereditary causes Secondary: injury to one or more components of the lymphatic system

A patient with a transfemoral amputation is unable to wear a total contact prosthesis for the past 4 days. The patient reports shooting pain at the end of the residual limb. Examination of the residual limb does not show any erythema. The MOST LIKELY cause is: A. Cellulitis B. Dermatitis C. Impetigo D. Neuroma

D. Neuroma The other three options have erythema so you can rule them out.

A patient admitted to the hospital 3 days ago presents with moderate to high amounts of drainage and edema of the left lower extremity. Which of the following characteristics BEST describes the patient's presentation? A. Decrease pulse, no granulation tissue, and ulcer on the lateral malleolus with smooth edges B. Normal pulse, no granulation tissue, and ulcer on the lateral malleolus with smooth edges C. No pulse, good granulation tissue, and shallow ulcer on the medial malleolus D. Normal pulse, good granulation tissue, and dark pigmented ulcer on the medial malleolus

D. Normal pulse, good granulation tissue, and dark pigmented ulcer on the medial malleolus In a venous ulcer- the pulses are normal, drainage is moderate to high, granulation tissue is present in the wound bed, swelling of unilateral or bilateral LEs, dark pigmentation due to hemosiderin deposition and is usually located proximal to medial malleolus.

A note in a patient's medical record indicates a specific drug is taken through enteral administration. Which of the following methods of drug delivery is an example of enteral administration? A. Inhalation B. Injection C. Topical D. Oral

D. Oral Enteral administration of drugs involves the esophagus, stomach, and intestines. The most common routes of enteral administration are oral, sublingual, and rectal.

A 10-year-old male complains of RLE pain specifically in the thigh region that worsens at night and is relieved by aspirin. PT palpated site as it feels warm and tender to touch; however, there is no mass and no swelling present. Which of the following conditions is MOST likely suspected? A. Ewing sarcoma B. Chondrosarcoma C. Osteosarcoma D. Osteoid osteoma

D. Osteoid osteoma It is unilateral, worse at night, and there is NO MASS. With a sarcoma there will be a mass so this is more related to the bone. It also does not spread so it can stay unilateral. Osteoid osteoma is a benign tumor of the bone which is usually present in patients ages 5-20 years.

A patient is referred to physical therapy with a C6 nerve root injury. Which of the following clinical findings would be the LEAST expected with this type of injury? A. Diminished sensation on the anterior arm and the index finger B. Weakness in the biceps and supinator C. Diminished brachioradialis reflex D. Paresthesias of the long and ring fingers

D. Paresthesias of the long and ring fingers Paresthesias of the long and ring fingers are commonly associated with the C7 nerve root. Other findings of a C7 nerve root injury include weakness of the triceps and wrist flexors, and a diminished triceps reflex.

While evaluating the gait cycle of a 68-year-old male patient, the PT observes a right pelvic hike during the swing phase of the right gait cycle. Which of the following conditions is LEAST likely to cause the problem? A. Reduced right hip flexion B. Inadequate right knee flexion C. Lack of right ankle dorsiflexion D. Right ankle plantar flexor weakness

D. Right ankle plantar flexor weakness Plantar flexor weakness would affect the push off but NOT the swing phase

A physical therapist uses the Modified Ashworth Scale when assessing a patient post stroke. Which of the following assessment procedures would be the MOST appropriate when using the Modified Ashworth Scale? A. Strength testing B. Deep tendon reflex testing C. Active range of motion D. Passive range of motion

D. Passive range of motion The MAS is an assessment tool used to grade spasticity. Spasticity is an increased resistance to PASSIVE stretch that is velocity-dependent.

Which of the following interventions would be MOST appropriate for a patient with a spinal cord lesion to the anterolateral sensory system? A. Tactile stimulation using tuning forks and vibrators of varying frequencies B. Active movement using visual feedback for facilitation of position sense C. Sensory re-education utilizing objects of various sizes, shapes, and textures D. Patient education concerning protection from hot/cold injuries

D. Patient education concerning protection from hot/cold injuries With lesions to the anterolateral system, a patient may exhibit sensory deficits in both light touch and hot/cold discrimination. Failure to distinguish extremes in temperature could result in the patient sustaining thermal injuries. Instruction in techniques to protect against these injuries would be of primary importance.

A PT works with a 75-year-old female patient following a bone marrow transplant. The patient's platelet count is 25,000 cell/mm3. Which of the following interventions would be MOST appropriate? A. Log roll training and breathing exercises B. Progressive resistance exercises and infection control C. Bicycling and lower extremity stretching D. Patient education on fall prevention and progressive ambulation

D. Patient education on fall prevention and progressive ambulation For patients with platelet count 20,000-30,000- Light exercises (no PROM; light AROM permitted; walking as tolerated) and fall prevention is recommended. Below 20,000 - ADLs, AAROM (AROM but no antigravity or resistive exercise). Resistive exercises are contraindicated.

A physical therapist treats a patient who sustained an acute wound to the anterior surface of the forearm. Which of the following types of cells would have been the FIRST to arrive at the injury site immediately following the incident? A. Endothelial cells B. Fibroblasts C. Leukocytes D. Platelets

D. Platelets Platelets are the first cells to arrive at the wound site. These cells attach to exposed collagen at the injury site and release chemicals that attract more platelets. Collectively, the platelets form a platelet plug to temporarily stop the bleeding. Platelets are most active in the hemostasis phase of wound healing.

A physical therapist assesses the vital signs of a patient with a blood disorder. The therapist records the patient's blood pressure as 150/85 mm Hg. Which of the following conditions is the MOST likely cause of the abnormal blood pressure measurement? A. Anemia B. Thrombocytosis C. Leukopenia D. Polycythemia

D. Polycythemia Polycythemia is an increase in the number of red blood cells and concentration of hemoglobin. This condition results in increased blood viscosity and increased blood volume, thus resulting in elevated blood pressure measurements.

A 54-year-old female patient is seen by a PT for gait training. The patient performs excessive hip flexion while ascending a ramp. Which of the following should the PT focus FIRST on mobilizing? A. Anterior glide on hip joint B. Posterior glide on hip joint C. Anterior glide on ankle joint D. Posterior glide on ankle joint

D. Posterior glide on ankle joint This person most likely has limited ankle DF and they are compensating for this by excessive hip flexion.

A patient has a negative result on a stress test for the medial collateral ligament. A radiograph shows the coronoid process is inferior to the trochlea. The patient MOST likely has which of the following injuries? A. Varus displacement B. Valgus displacement C. Anterior displacement D. Posterolateral displacement

D. Posterolateral displacement Displacing the coronoid process inferior to the trochlea requires the ulna to be displaced posterolateral to the humerus. A- LCL instability B- MCL instability C- olecranon is displaced anteriorly

A PT student is constantly on their new phone and is having difficulty swiping upwards with their thumb. What is the BEST mobilization to improve the thumb range of motion so that the student can swipe happily thereafter? A. Inferior glide at the CMC joint B. Superior glide at the CMC joint C. Ulnar glide at the CMC joint D. Radial glide at the CMC joint

D. Radial glide at the CMC joint The motion of sliding your thumb up is CMC extension. With CMC flexion/extension roll and slide are in the SAME direction. Flexion/extension is a frontal plane motion at the thumb. So both roll and slide would be radially.

A patient with SCI T5 level arrived to an outpatient clinic for an individual treatment session. The physical therapist plans to incorporate challenging skills in transitioning into prone on elbows as this focuses on increasing muscular strength at the head, neck, shoulders, and scapula region. Which of the following PNF technique would be MOST appropriate to utilize in this training? A. Slow reversal hold B. Normal timing skill C. Rhythmic rotation D. Rhythmic stabilization

D. Rhythmic stabilization If you want MORE STABILITY you would use rhythmic STABILIZATION. Your target is to work on stabilizing the patient in this position.

The definition of atherosclerosis is characterized by the thickening of the blood vessel walls from focal accumulation of all of the following except: A. Lipids B. Platelets C. Monocytes D. Sodium and potassium

D. Sodium and potassium

A patient has an acute lumbar disc herniation and lumbar spondylolisthesis. Which of the following interventions is MOST appropriate? A. Passive trunk extension B. High-velocity manipulation C. Active trunk flexion exercises D. Spinal stabilization exercises

D. Spinal stabilization exercises Although trunk flexion is indicated for individuals who have spondylolisthesis, active trunk flexion is contraindicated for patients who have an acute disc lesion, such as a herniated disc (pp. 464-465, 473). Spinal stabilization exercises are indicated to increase trunk stability for patients who have an acute herniated disc and for patients who have spondylolisthesis (pp. 465, 473).

A 25-year-old male soccer player had to undergo an ACL reconstruction surgery. It has been 3 weeks since the surgery and he is undergoing PT treatment. Which of the following is LEAST likely to be contraindicated while exercising this patient? A. Short arc quads between 30 degrees to full extension B. Closed chain strengthening exercises of quadriceps between 60-90 degrees of knee flexion C. 5 lbs weights at ankle during quadriceps strengthening D. Squatting with knees staying posterior to the toes

D. Squatting with knees staying posterior to the toes When squatting the knees SHOULD NOT come anterior to the toes because it increases the shear forces of the tibia and puts stress on the graft.

A 45-year-old male patient presents with minimal symptoms of heaviness and stiffness which appears unilateral. A TI weighted image demonstrated lesions in the medulla oblongata. According to Hoehn-Yahr disability scale, the patient would MOST LIKELY be classified as: A. Stage IV B. Stage II C. Stage III D. Stage I

D. Stage I Stage I affects ONE side with only minimal symptoms.

A patient arrives at a clinic with right shoulder pain. Upon evaluation, the patient described of having increased difficulty with brushing the back of the head for the past 3 months with increased pain at rest. The PT observed that ROM is limited in all directions. Which of the following stages and movement patterns would MOST LIKELY be associated with this scenario? A. Stage III, IR>Abd>ER B. Stage I, ER>Abd>IR C. Stage IV, IR>Abd>ER D. Stage II, ER>Abd>IR

D. Stage II, ER>Abd>IR The capsular pattern for the shoulder is ER>Abd>IR so you can automatically rule out A and C. Use posterior-inferior glides for adhesive capsulitis!!! Use iodine (sclerotic scars) or acetic acid (calcium deposits) if treating this with iontophoresis! People with adhesive capsulitis often also have DM and hyperthyrodism. Stages of adhesive capsulitis: Stage 1- gradual onset of pain that increases with movement and is present at night. Loss of ER with intact RTC strength. This stage usually lasts less than 3 months. Stage 2- "Freezing stage". Persistent and more intense pain even at rest. Motion is limited in all direction and cannot be fully restored with an injection. 3-9 months after onset. Stage 3- "Frozen stage". Pain only with movement, significant adhesions, and limited GH motions. Compensates with excessive scapulothoracic movement. Atrophy of shoulder muscles. 9-15 months after onset. Stage 4- "Thawing stage". Minimal pain and no synovitis but significant capsular restrictions from adhesions. Motion may improve. 15-24 months after onset.

Excessive upward rotation of the left scapula is noted as the patient attempts shoulder abduction. Which of the following exercises is MOST appropriate to help correct the excessive scapular rotation? A. Forearm wall slides to strengthen serratus anterior B. Shoulder shrugs to strengthen upper trapezius C. Standing wall push-ups to strengthen serratus anterior D. Standing rows to strengthen the rhomboids

D. Standing rows to strengthen the rhomboids Serratus anterior and upper trapezius are both upward rotators of the scapula so you are left with option D. The rhomboids help with downward rotation.

The physical therapist is testing the active shoulder range of motion of a 45-year-old female patient. The physical therapist asks the patient to move the shoulder to 150 degrees of adduction. According to Concave-Convex rule, during adduction at the shoulder joint (GH joint), the humerus will slide: A. Anteriorly B. Inferiorly C. Posteriorly D. Superiorly

D. Superiorly In glenohumeral Joint, the concave glenoid fossa articulates with the convex humeral head. According to the concave- convex rule, with motions of the humerus, roll and slide will occur in opposite direction. So, during adduction of shoulder, humerus roll inferiorly and will slide superiorly.

A PT is performing an exercise stress test on a 45-year-old male patient. The patient's resting values are: BP 130/90 mm Hg, HR is 75 bpm, and RR is 24 breaths/min. Which of the following is an ABNORMAL response to vigorous aerobic exercise? A. Diastolic blood decreases to 88 mm Hg B. Diastolic blood pressure increases to 100 mm Hg C. Respiratory rate increases to 34 breaths/minute D. Systolic blood pressure decreases to 108 mm Hg

D. Systolic blood pressure decreases to 108 mm Hg Systolic blood pressure should increase with exercise! AND If the diastolic changes by more than 10 and systolic change by more than 20 you need to stop. RR is only of concern when it surpasses 40 breaths/minute

A 58-year-old patient presents to the clinic with stage 2 lymphedema. The patient's right brachial pressure is 100 mm Hg and ankle pressure is 50 mm Hg. Which of the following statements is CORRECT about use of intermittent compression therapy? A. The PT should use intermittent compression therapy because it decreases peripheral resistance B. The PT should not use intermittent compression therapy because it decreases peripheral resistance C. The PT should use intermittent compression therapy because it increases peripheral resistance D. The PT should not use intermittent compression therapy because it increases peripheral resistance

D. The PT should not use intermittent compression therapy because it increases peripheral resistance Stage 2 lymphedema is spontaneously irreversible. Hard swelling is present with "brawny" edema. There is a positive Stemmer sign. Tissue is fibrosclerotic; proliferation of adipose tissue. An ABI of 0.5 is pretty severe and you SHOULD NOT use intermittent compression therapy. Increasing peripheral resistance will only make it worse for the patient. The blood in the ankles is already low and compression would worsen the blood flow even more. This patient ABI would be 50/100= 0.5 which is moderate/severe. Normal: 1.19-0.95 Mild: 0.94-0.75 Moderate: 0.74-0.5 Severe: < 0.5

A PT is assessing the integrity of the right optic nerve. Which of the following is LEAST appropriate to assess the function of this sensory nerve? A. Confrontation test B. Reading the Snellen's eye chart with left eye shut C. Shining light in the eye of the right eye D. Touching the cornea of the right eye with cotton

D. Touching the cornea of the right eye with cotton Confrontation test- checking peripheral vision C tests both oculomotor and optic because the optic nerve should be sensitive to the light whereas the oculomotor nerve will constrict the pupils

A physical therapist applies a dressing to an area of skin on a patient's heel. The therapist decided to use the dressing as a prophylactic measure to reduce the risk of skin breakdown in an area that was determined to be particularly susceptible. Which of the following dressings would the therapist have MOST likely used? A. Calcium alginate B. Hydrocolloid C. Hydrogel D. Transparent film

D. Transparent film A calcium alginate is used for wounds that produce moderate to heavy exudate which would not apply here so you can rule that out. A hydrocolloid is used for partial and full-thickness wound not a superficial wound so that would not be appropriate. A hydrogel would not be used as a prophylactic measure and is instead used to prevent a wound from dehydrating. A transparent film would be appropriate. Since the film is transparent it allows for frequent assessment of the wound and offers some protection. These films are permeable to oxygen but are impermeable to microorganisms and moisture.

A physical therapist reads in the medical record that a patient developed a hematoma in their lower leg. Which diagnostic imaging method would have MOST likely been used to identify the presence of the hematoma? A. X-ray B. Bone scan C. Arteriography D. Ultrasound

D. Ultrasound A hematoma is a collection of blood OUTSIDE of blood vessels. An arteriography looks at the arteries themselves not outside of them.

A 63-year-old patient is 7 weeks out of right rTSA (reverse total shoulder arthroplasty) surgery. When educating the patient for the post operative precautions, which of the following activities can be performed? A. Lifting objects up to 6 lbs B. Reaching for the wallet in the back pocket C. Combing hair with a brush D. Use overhead pulley with patient facing the doorway

D. Using overhead pulley with patient facing the doorway If an overhead rope-pulley system is used for assisted elevation of the arm, initially have the patient face the doorway and pulley apparatus, so shoulder elevation occurs only within a limited range A- need to wait until 12 weeks B- this involves a lot of extension and IR which you want to avoid 6 weeks: protection 12 weeks: active phase 6 months: return to sports

A physical therapist is examining a patient who is recovering from a motor vehicle accident that injured the left forearm. The patient appears to have damaged the posterior interosseous nerve (branch of radial nerve). Which of the following motions will be MOST impaired? A. Forearm pronation B. Ulnar deviation C. Wrist flexion D. Wrist extension

D. Wrist extension The major disability associated with radial nerve injury is a weak grip, which is weakened because of poor stabilization of the wrist and the finger joints. In addition, the patient demonstrates an inability to extend the thumb, the wrist, and the elbow, as well as the proximal phalanges. The radial nerve innervates the extensors!

A physical therapist reviews a patient's medical history prior to administering intermittent compression. Which of the following conditions would be considered a contraindication when using this type of mechanical device? A. Venous stasis ulcer B. Acute pulmonary edema C. Intermittent claudication D. Lymphedema

B. Acute pulmonary edema Acute pulmonary edema should not be treated with intermittent compression since the shift of fluid from the peripheral to the central circulation may significantly increase stress on the heart

A patient who is 3 months post right ankle fracture has an AROM of 0-30 degrees of DF and 0-10 degrees of PF. To restore motion required for normal walking, which of the following joint mobilization techniques should the PT perform? A. Anterior glide of the talus B. Lateral glide of the calcaneus C. Medial glide of the calcaneus D. Posterior glide of the talus

A. Anterior glide of the talus PF is limited so you are restoring that. Always stick to your plane for glides. You can rule out B and C because they are not in the correct plane for PF. With PF you are moving down and posteriorly. The ankle is convex on concave so you know that roll and glide are opposite. So since the roll is posterior you know the slide is anterior.

During a cranial nerve examination, PT asks the patient to open his mouth. The PT notices that the patient's uvula is angled towards the right side. Which cranial nerve is most likely affected and which side is most likely weak? A. CN 10, weak left B. CN 10, weak right C. CN 9, weak left D. CN 12, weak right

A. CN 10, weak left When cranial nerve X of left side is affected, it causes left side weakness and uvula deviates to the right side. Intact CN X (Vagus) maintains midline position of uvula, CN IX (glossopharyngeal) maintains the gag reflex. CN XII maintains position of the tongue.

An 81-year-old female patient has poor posture and osteoporosis. To decrease the patient's risk for vertebral fracture, which of the following interventions is MOST appropriate to perform? A. Hamstring stretches with trunk flexion B. Back extensor strengthening exercises C. Postural exercise to decrease lumbar lordosis D. Instruction to carry loads in front of the body

B. Back extensor strengthening exercises Patients who have osteoporosis and who want to prevent a spinal compression fracture should perform daily exercise to increase muscular endurance in spinal extensors

A physical therapist is reading the lab report of a patient admitted in the hospital. In which of the following scenarios would the patient be advised to NOT exercise? A. Platelet count: 22,000/mm3; Hb: 9g/dL; WBC: 6000/mm3 B. Hb: 7g/dL; Hematocrit: 23%; WBC: 4800/mm3 C. Hematocrit: 25%; Platelet count: 20,000/mm3; WBC: 5200/mm3 D. Platelet count: 29,000/mm3; Hb: 8g/dL; Hematocrit: 30%

B. Hb: 7g/dL; Hematocrit: 23%; WBC: 4800/mm3 Guidelines for exercise based on laboratory values suggest no exercises for the following findings: WBC <5000/mm3; Hematocrit <25%; Hb <8g/dL; Platelets <20,000- AAROM or AROM but no antigravity or resisted exercise.

A 62 year old male with a 20 year history of smoking, DM II, HTN stage 1 and COPD presents with cough and expectoration. On auscultation the therapist finds presence of crepitus. Which of the following lobes can be drained MOST EFFICIENTLY in the sitting position? A. L apical segment of upper lobe and R middle lobe B. L apical segment of upper lobe and R posterior segment of upper lobes C. R apical segments of upper lob and R superior segments of lower lobes D. R apical segment of upper lobe and R anterior segment of upper lobe

B. L apical segment of upper lobe and R posterior segment of upper lobes Apical and Posterior segments of the upper lobes can be drained in the sitting position.

A physical therapist treats a patient who has benign paroxysmal positional vertigo. Which of the following physical therapy treatments would MOST benefit this patient? A. Dix-Hallpike maneuver B. Canalith repositioning maneuvers C. Singular neurectomy D. Gaze stability exercises

B. Canalith repositioning maneuvers Dix-Hallpike is a DIAGNOSTIC tool NOT a treatment.

A physical therapist treats a patient with Parkinson's disease who has been receiving levodopa therapy for two years. Which of the following side effects would MOST likely be present due to the chronic use of this medication? A. Bradykinesia B. Choreoathetosis C. Shuffling gait D. Rigidity

B. Choreoathetosis Choreoathetosis is a type of dyskinesia characterized by uncontrolled, involuntary movements. The onset of dyskinesias can occur as soon as three months after first receiving levodopa therapy. The other three options are common characteristics of PD that would IMPROVE with levodopa.

20-year-old female, after an uncomplicated delivery has a 3 cm diastasis recti and weak abdominal muscles. Which of the following exercises is MOST appropriate for the patient? A. Sitting on a Swiss Ball B. Head lifts with arms bracing the abdomen C. Double leg lifts D. Deep breathing exercises in bridging position

B. Head lifts with arms bracing the abdomen Sitting on a Swiss Ball will strengthen the abdominals but it is too low level. Double leg lifts asks for too much contraction of the abdominals. Deep breathing exercises won't help too much with strengthening. Head lifts with the arms bracing the abdomen allows you to strengthen the abdominals without putting too much strain on the diastasis recti. Diastasis recti: <2 cm= normal, continue with regular exercises 2-3 cm= perform abdominal strengthening exercises with caution 4 cm or more= do not exercise; deep breathing exercises

A 20-year-old male athlete presents to a PT clinic five days post allograft ACL reconstruction. The patient is independent in ambulation with crutches. He has 53 degrees of active knee flexion and 67 degrees of passive knee flexion. He lacks 10 degrees of active knee extension and 5 degrees passively. What is the MOST significant deficit on which the PT should focus treatment? A. Lack of active knee extension B. Lack of passive knee extension C. Lack of passive knee flexion D. Lack of active knee flexion

B. Lack of passive knee extension After ACL reconstruction, the main goal of the therapist is the prevention of extension lag. So, passive extension must be achieved first (by the end of the first 1 to 2 weeks). Once passive knee extension is achieved active knee extension can be achieved to prevent the Lag.

You are treating a patient s/p total hip arthroplasty anterior approach 3 days post-op in inpatient acute care. The patient complains of 4/10 pain and pain in the anterior hip region. The patient's goal is to be able to navigate stairs to safely enter and exit the home. Which of the following is the LEAST appropriate intervention? A. AROM of knee and ankle joints of the affected extremity in sitting B. Stair climbing with UE support and a focus on hip hyperextension to improve reciprocal stair negotiation C. Standing marching on ground with UE support and emphasis on weight transfers D. Stair tapping with UE support to improve stair negotiation tolerance

B. Stair climbing with UE support and a focus on hip hyperextension to improve reciprocal stair negotiation The patient had an anterior approach s/p hip replacement. Contraindications for the anterior hip approach are excessive hip extension and external rotation and hip flexion greater than 90 degrees.

A patient was diagnosed with a tumor on the left glossopharyngeal nerve which affected the function of the nerve. Which of the following statements is MOST appropriate regarding this patient? A. No response seen on touching L posterior pharyngeal wall B. Uvula deviates to R on touching L posterior pharyngeal wall C. Symmetrical elevation of palate with touching L posterior pharyngeal wall D. Uvula deviates to L on touching L posterior pharyngeal wall

B. Uvula deviates to R on touching L posterior pharyngeal wall The uvula always deviates to the opposite side of the lesion or tumor (opposite of hypoglossal nerve)

A physical therapist applies a transparent film dressing to a patient's forearm. Which objective finding would MOST warrant the use of this type of dressing? A. Bacterial infection B. Fragile skin C. Minimal drainage D. Peri-wound maceration

C. Minimal drainage Transparent film dressings are used for wounds with minimal to no drainage. This type of dressing is permeable to water vapor but not water, and therefore moderate or heavy drainage will accumulate under the dressing, resulting in maceration and excessive dressing changes.

A patient presents with intermittent claudication for 5 minutes. Upon examination, the patient has a 0.78 ABI score. What does a score of 0.78 ABI indicate? A. Severe arterial disease B. Normal- no arterial disease C. Mild arterial disease D. Moderate arterial disease

C. Mild arterial disease Normal: 1.19-0.95 Mild: 0.94-0.75 Moderate: 0.74-0.5 Severe: < 0.5

A 40-year-old male patient presents with right homonymous hemianopsia and right hemiparesis. He is unable to read, but he can communicate through writing. The MOST likely location of infarct is: A. Right MCA B. Right PCA C. Left PCA D. Left MCA

C. Left PCA Left PCA infarcts can cause dyslexia i.e. difficulty with reading (calcarine lesion and posterior part of corpus callosum), homonymous hemianopsia (damage to primary visual cortex or optic radiation) and right hemiparesis (damage to cerebral peduncle of midbrain).

You are treating a patient in inpatient rehabilitation with deficits in pupillary light reflex in the right eye and a failure to look at the nose with the right eye. When testing lateral deviation of the eye, the patient is able to follow the stimulus. No deficits noted in chewing. Which anatomical location is MOST likely affected? A. Medulla B. Cerebellum C. Midbrain D. Pons

C. Midbrain The pupillary light reflex is associated with CN III. The inability to look at the nose with the right eye is associated with CN IV (remember SO4). So now you have to remember where these nerves are in the brain. Remember "CE MI PONS MEDU". So, if you count to 3 and 4 you will find that CN III and IV are in the midbrain.

A patient who is taking warfarin (Coumadin) has an international normalized ratio (INR) of 5. What is the MOST appropriate interpretation of this value? A. This value is too low, indicating there is increased risk of excessive anticoagulation B. This value is too low, indicating the blood is too thick and there is increased risk for clots C. The value is too high, indicating there is increased risk of excessive coagulation D. This value is too high, indicating the blood is too thick and there is increased risk for clots

C. The value is too high, indicating there is increased risk of excessive anticoagulation International normalized ratio is used to assess the adequacy and effectiveness of anticoagulant therapies, such as warfarin (Coumadin). An acceptable ratio during anticoagulant therapy is 2-3.5. If a higher number is noted, then the patient is at risk for excessive bleeding. This can be dangerous and should be noted in individuals undergoing anticoagulation therapy.

A physical therapist is evaluating a patient, who works as a maid in a hotel. Her work requires her to make beds, vacuum, clean bathrooms, and manipulate a fairly heavy cart with cleaning supplies and linen. She is right hand dominant and suffered a severed biceps tendon on the right side (which has not been repaired) in an incident involving a boyfriend, another woman, and a knife. As a result of this incident, she has experienced a secondary inflammation of the interosseous membrane (between the radius & ulna) that causes pain when it is under moderate tension. During the physical examination, the physical therapist is likely to see: A. An increase in range of motion and strength for elbow flexion B. A decrease in strength and increase in range of motion for elbow flexion C. A decrease in strength but no decrease in active range of motion for elbow flexion D. A decrease in strength and active range of motion for elbow flexion

D. A decrease in strength and active range of motion for elbow flexion The function of the biceps muscle is elbow flexion and forearm supination. When the tendon is ruptured, the patient will have weak or absent elbow flexion and supination, swelling, ecchymosis, palpable gap in the biceps tendon.

A 65-year-old patient arrives at the PT clinic. Patient reports that since this morning he has felt pain down both arms, has shortness of breath, and has continued to perspire. What is the MOST appropriate intervention? A. Evaluate the cervical spine and provide pectoral stretches to decrease bilateral arm pain B. Evaluate the cervical spine and provide scapular strengthening exercises to decrease bilateral arm pain C. Treat the patient with TENS and recommend that the patient follow up with a physician within 3 days D. Immediately contact emergency medical services

D. Immediately contact emergency medical services This could be something to do with the heart or something else very serious. When in doubt save your license and put the problem on someone else who is more knowledgeable.

Which of the following instruments is MOST appropriate for measuring patients' quality of life? A. Dynamic Gait Index (DGI) B. Oswestry low back pain disability index (ODI) C. Timed up and go test (TUG) D. Medical Outcomes Study 36-item short form (SF-36)

D. Medical Outcomes Study 36-item short form (SF-36) The MOS is a 36-item short form for measuring the patient's quality of life. It is a self-report that covers eight domains of physical function, role limitation due to physical problems, role limitations due to emotional problems, fatigue, and general health perceptions.

A PT is treating a 22-year-old male patient who has been classified as ASIA C. Which of the following statements is true about this patient? A. Motor function is preserved with only 1/3 of the key muscles below the neurological injury have a grade of less than 3 B. Motor function is preserved with at least 1/2 of the key muscles below the neurological level of injury have a muscle grade of more than 3 C. Motor function is preserved with all of the key muscles below the neurological level of injury have a muscle grade of more than 3 D. Motor function is preserved with more than 1/2 of the key muscles below the neurological level of injury have a muscle grade of less than 3

D. Motor function is preserved with more than 1/2 of the key muscles below the neurological level of injury have a muscle grade of less than 3 A: complete; no motor or sensory B: sensory but no motor C: less than 1/2 of key muscles have a muscle grade of 3 or more D: more than 1/2 of key muscles have a muscle grade of 3 or more E: normal

A patient comes to the clinic with tightness of lumbricals. In order to stretch the lumbricals, the physical therapist needs to: A. Flex the wrist B. Flex the wrist and MCP joints C. Extend MCP and flex IP joints D. Flex MCP and extend IP joints

C. Extend MCP and flex IP joints The action of lumbricals is MCP flexion and IP extension. So, to stretch it we can perform MCP extension and IP flexion.

You are treating a 35 year old patient with the following Electrocardiogram in inpatient rehabilitation. Which cardiac condition is MOST LIKELY represented below? A. Atrial fibrillation B. Ventricular tachycardia C. Second degree AV block D. Third degree AV block

C. Second degree AV block This is a second degree AV block type I. There is a progressive increase in the PR interval until the beat drops.

A physical therapist working in the home care setting attempts to obtain the body temperature of a patient. Which of the following methods for obtaining temperature would likely result in the LOWEST obtained value? A. Tympanic membrane B. Rectal C. Oral D. Axillary

D. Axillary Axillary temperature is obtained by placing the tip of the thermometer in the armpit. The arm is then brought to the patient's side holding the elbow against the chest for 4-5 minutes. The axillary temperature is typically 0.5-1.0 degree Fahrenheit lower than the oral temperature value.

A patient sustains a deep partial-thickness burn to the anterior surface of the right upper extremity and a superficial partial-thickness burn to the anterior surface of the trunk. According to the rule of nines, the patient has burns over what percentage of total body surface area? A. 13.5% B. 22.5% C. 27% D. 36%

B. 22.5% Anterior surface of the trunk: 18% Anterior surface of the UE: 4.5%

A 35-year-old patient has difficulty turning door knobs. Which of the following muscles is an antagonist to the muscle responsible for this action? A. Biceps B. Triceps C. Supinator D. Extensor pollicis longus

B. Triceps Turning door knobs require the biceps to supinate with the elbow in flexion, while the triceps counteracts /neutralizes the flexion action.

Which of the following motions will close the right cervical facet joints from a neutral position? A. Cervical extension, right side flexion, left rotation B. Cervical extension, left side flexion, left rotation C. Cervical flexion, right side flexion, left rotation D. Cervical extension, right side flexion, right rotation

D. Cervical extension, right side flexion, right rotation Extension closes the gap. You always side-bend to the same side to close even more and rotation is to the same side in the C-spine.

A patient has pain in the mid lower abdominal area and low back that is not of musculoskeletal origin. Which of the following diagnoses is MOST likely? A. Enlarged liver B. Inflamed pancreas C. Ruptured gallbladder D. Dissecting aortic aneurysm

D. Dissecting aortic aneurysm Pain in the abdominal and lower back region can be referred by a dissecting aortic aneurysm. Liver and gallbladder: right shoulder

In a research study on obese THA patients, a correlation coefficient (r) of +0.80 was found for the relationship between weight and BMI. Which of the following interpretations of this finding is MOST appropriate? A. Weight and BMI have a high positive correlation B. 80 percent of the variability in BMI can be accounted for by weight C. 80 percent of the variability in weight can be accounted for by BMI D. There are no significant difference between the weight and BMI levels

A. Weight and BMI have a positive correlation

Which of the following is LEAST LIKELY to be present in a patient with congestive heart failure? A. Weight loss B. Weight gain C. Peripheral edema D. Orthopnea

A. Weight loss

A 53-year-old patient has an ulcer on the foot as shown in the picture. What is the MOST likely diagnosis? A. Arterial ulcer B. Venous ulcer C. Diabetic foot ulcer D. Pressure ulcer

B. Venous ulcer The wound is just proximal to the medial malleolus which is characteristic of a venous ulcer.

A patient is participating in a total body aerobic exercise program during Phase III cardiac rehabilitation. Given a fixed power output during exercise, which of the following physiological variables will MOST likely occur? A. Heart rate will be lower during upper extremity exercise as compared to lower extremity exercise B. Stroke volume will be lower during lower extremity exercise as compared to upper extremity exercise C. Systolic blood pressure will be lower during upper extremity exercise as compared to lower extremity exercise D. Rate pressure product will be lower during lower extremity exercise as compared to upper extremity

D. Rate pressure product will be lower during lower extremity exercise as compared to upper extremity exercise The rate pressure product will be lower for leg exercise and higher for arm exercise due to the effect of higher heart rate and higher systolic pressure. RPP= HR x SBP

A 35 year old male patient has been diagnosed with acromioclavicular joint arthritis. He will most likely have the GREATEST discomfort when performing which of the following tasks? A. Pulling a wallet out of the back pocket on the same side as the involved extremity B. Bending forward to pick up a pen that is lying on the floor C. Pushing a heavy shopping cart using the involved extremity D. Reaching across the body to scratch the back of the opposite shoulder

D. Reaching across the body to scratch the back of the opposite shoulder Reaching across the body to scratch the back of the opposite shoulder requires horizontal adduction movement at the shoulder joint. Pain with horizontal adduction indicates A-C joint pathology.

A physical therapist is conducting a wheelchair assessment of a patient with bilateral trans-femoral amputation. Which of the following wheelchair modifications is most appropriate based on the patient's presentation? A. Rear wheels are positioned two inches anterior to the normal position B. Seat is lowered two inches to the normal position C. Castor wheels are positioned two inches posterior to the normal position D. Rear wheels are positioned two inches posterior to the normal position

D. Rear wheels are positioned two inches posterior to the normal position In patients with bilateral lower extremity amputation, the rear wheel axles are positioned approximately 2 inches posterior to their normal position to widen the base of support of the chair and compensate for the loss of the weight of the user's lower extremities.

A patient sustained a nondisplaced midshaft radial and ulnar fracture 12 weeks ago. The patient was casted in mid-range elbow flexion with the forearm in a neutral position. Which of the following muscle pairs would MOST likely demonstrate contractile tissue shortening following the cast removal? A. Brachialis and flexor pollicis B. Brachioradialis and triceps C. Biceps brachii and triceps D. Biceps brachii and brachioradialis

D. Biceps brachii and brachioradialis The biceps brachii and brachioradialis flex the elbow and supinate the forearm. The cast position would shorten both muscles.

All of the following are guidelines in treating patients with GERD EXCEPT: A. Any intervention requiring a supine position should be scheduled before meals and avoided just after eating B. Encourage the patient to sleep on left side for nocturnal reflex C. Modification of position towards a more upright posture may be required if symptoms persist during therapy D. Encourage the patient to sleep on right side for nocturnal reflex

D. Encourage the patient to sleep on right side for nocturnal reflex Sleeping on the right side will encourage the contents of the stomach to enter the esophagus.

A 62-year-old patient has ascites and bilateral pedal edema. The patient's pulse rhythm is regular. The patient's history is negative for any liver, kidney, or metabolic disease. Which of the following conditions is MOST likely present? A. Hypotension B. Hypertension C. Left ventricular failure D. Right ventricular failure

D. Right ventricular failure Right ventricular failure results in backup of blood into the systemic venous circulation, manifested by edema systemically, including jugular venous distention, ascites, and bilateral pedal edema

A patient presents with dizziness with rolling in bed and bending forward to load the dishwasher. On assessment, the patient tests positive for dix hallpike test on the right side. Which of the following is MOST likely expected to be present in this patient? A. Pendulum nystagmus B. Abnormal saccades C. Ageotropic nystagmus D. Upbeating torsional nystagmus

D. Upbeating torsional nystagmus Pendulum nystagmus: central vestibular disorder Abnormal saccades: central vestibular disorder Ageotrophic nystagmus: horizontal BPPV which would be positive with the supine roll test

A patient who has right upper extremity lymphedema post radical mastectomy discusses the cause of lymphedema with her physician. The physician explains that both the venous system and the lymphatic system are responsible for collecting and transporting interstitial fluid. What percentage of interstitial fluid is collected by a normally functioning lymphatic system? A. 15% B. 35% C. 55% D. 75%

A. 15% Lymphatic system: 10-20% of interstitial fluid Venous system: 80-90% of interstitial fluid

A physical therapist treats a patient diagnosed with osteonecrosis of the femoral condyle. Which patient profile is the MOST typical with this medical condition? A. A 42-year-old female with osteonecrosis of the lateral femoral condyle B. A 64-year-old female with osteonecrosis of the medial femoral condyle C. A 46-year-old male with osteonecrosis of the medial femoral condyle D. A 68-year-old male with osteonecrosis of the lateral femoral condyle

B. A 64-year-old female with osteonecrosis of the medial femoral condyle Osteonecrosis of the femur occurs when a segment of the bone loses its blood supply. Females are more likely than males to experience osteonecrosis of the femoral condyle with peak incidence occurring in women who are over 60 years of age. The medial femoral condyle is more likely to be affected than the lateral femoral condyle due to increased weight bearing forces caused by the center of gravity being medial to the knee.

A patient presents with an acute sprain of the right ankle. According to the patient, this has occurred frequently over the past five years. The clinical test that the physical therapist should use to examine the laxity of the anterior talofibular ligament is: A. Anterior drawer test B. Talar tilt C. Thompson's test D. Morton's test

A. Anterior drawer test Talar test: CFL Thompson's test: Achilles tendon

A physical therapist observes thenar atrophy when examining a patient's hand. In the absence of other relevant findings, this could BEST be explained by which of the following conditions? A. C8 nerve root lesion B. Paralysis of the interossei C. Radial nerve lesion D. Ulnar nerve lesion

A. C8 nerve root lesion A C8 or T1 nerve root lesion often results in atrophy of the thenar eminence. The muscles associated with the thenar eminence are all innervated by nerves that arise from the C8-T1 nerve roots.

After evaluating a baseball player, the PT concludes that there is insufficient deceleration of the shoulder during throwing. Which of the following strengthening techniques will be MOST effective in improving control of deceleration of the shoulder?

A. Eccentric exercises of the teres minor and infraspinatus Teres minor concentrically does ER so you would eccentrically contract it to slow down IR.

A group of PT students are compiling an injury prevention program for adolescent soccer players. The independent t-test was used as the statistical tool. How would statistically significant results be presented if alpha was set at 0.05? A. p < 0.05 B. p > 0.05 C. r > 0.05 D. R^2 < 0.05

A. p < 0.05

The patient in the photograph sustained a twisting injury to the knee 1 week ago. The physical therapist notes laxity when performing the test demonstrated in the photograph. The patient wants to resume athletic activity as soon as possible. Which of the following criteria MUST be met before the patient returns to athletic activity? A. The patient can demonstrate ability to jump and land with stability B. The patient's gait is nonantalgic C. Knee functional range of motion is restored, and pain is rated at less than 5/10 D. Knee effusion is abolished, quadriceps strength is Fair (4/5), and pain is rated 0/10

A. The patient can demonstrate ability to jump and land with stability The test shown in the photograph is used to assess the integrity of the anterior cruciate ligament. Jumping and landing with stability are required for the patient's functional goals associated with treatment of an anterior cruciate ligament injury.

The individual in the photograph is demonstrating how he lifts his laundry basket. What impact does the position of the basket have on the forces at the lumbar spine and disc? A. The position increases the compressive forces and flexion moment of the disc B. The position decreases the compressive forces and flexion moment of the disc C. The position decreases the compressive forces and increases the flexion moment of the disc D. The position increases the compressive forces and extension moment of the disc

A. The position increases the compressive forces and flexion moment of the disc The patient in the photograph demonstrates a lordotic posture of the lumbar spine, which increases compressive forces on the disc (Kisner). The position of the arms outstretched and away from the body increases the flexion moment around the body's center of mass (Samuels).

Palsy of which of the following cranial nerves would MOST likely lead to the presentation of torticollis in a child? A. Trochlear nerve (CN IV) B. Facial nerve (CN VII) C. Vestibulocochlear nerve (CN VIII) D. Hypoglossal nerve (CN XII)

A. Trochlear nerve (CN IV) Ocular torticollis may result from a lesion to the trochlear nerve. Damage to the trochlear nerve results in diplopia. Patients will frequently compensate for the diplopia by tilting the head anteriorly and laterally toward the side of the normal eye. Facial nerve: Bell's palsy Vestibulocochlear nerve: hearing loss and vestibular symptoms Hypoglossal nerve: deviation of the tongue to one side during protrusion

A male patient has signs and symptoms of intermittent claudication, unilateral pain in the buttock and thigh (sometimes exacerbating bilaterally), and a diminished femoral pulse. His pain is relieved by rest. Based on the clinical presentation, where is the site of the occlusion? A. Aortic birfucation B. Iliac artery C. Femoral and popliteal artery D. Tibial and common peroneal artery

B. Iliac artery The location of the pain is determined by the site of the major arterial occlusion The occlusion of iliac artery produces pain or discomfort in the buttock, hip, thigh of the affected leg, diminished or absent femoral or distal pulses. Femoral and popliteal artery occlusion produces pain in calf and foot. Tibial and common peroneal artery occlusion produces pain in calf and feet (occasionally).

Which of the following combinations of activities would be MOST beneficial for maintaining bone density in a patient with osteoporosis? A. Treadmill walking and balance training B. Treadmill walking and resistance training C. Swimming and balance training D. Swimming and resistance training

B. Treadmill walking and resistance training Walking is a beneficial weight-bearing aerobic activity. Resistance training is important to maintain bone mineral density or prevent loss of bone mineral density. SWIMMING HAS LIMITED VALUE FOR A PATIENT WITH OSTEOPOROSIS BECAUSE OF THE LACK OF WEIGHT-BEARING COMPONENT

During an examination, a physical therapist notes a yellow discoloration of the patient's skin and sclera of the eye. This discoloration would be LEAST likely observed with which of the following conditions? A. Cholecystitis B. Ulcerative colitis C. Pancreatitis D. Hepatitis

B. Ulcerative colitis Ulcerative colitis is an inflammation and ulceration of the inner lining of the colon and rectum. S/s include rectal bleeding, diarrhea, nausea, vomiting, weight loss, and fever. Jaundice is not a symptom typically seen with this condition.

A patient presents to clinic with a posterior herniated nucleus pulposus at L2-3. During the traction treatment in supine, the patient reports increased localized back pain. Which of the following actions is MOST appropriate for the PT to take? A. Change the position of the patient to sidelying B. Refer the patient back to the physician C. Change the position of the patient to prone D. Reduce the traction force to 15% of body weight

C. Change the position of the patient to prone For lumbar traction you need 25-50% of body weight so D would be too low to have an effect. For a posterior herniated nucleus pulposus you want to encourage spinal extension to help push the disc anteriorly back to neutral.

To determine if paratenonitis is present in the thumb (1st digit), which of the following tests is MOST appropriate to perform? A. Tinel B. Allen C. Finkelstein D. Phalen

C. Finkelstein The Finkelstein test is used to determine the presence of de Quervain or Hoffmann disease, a paratenonitis of the thumb (1st digit). Tinel- carpal tunnel syndrome Allen- checking blood flow in hands Phalen- carpal tunnel syndrome

A physical therapist is designing exercises for a patient who is on Beta Blockers. Which of the following statements is NOT correct about the effects of Beta Blockers? A. Decrease HR with exercise B. Decrease preload C. Increase stroke volume D. Increase afterload

C. Increase stroke volume Beta blockers blunt HR and decrease stroke volume

A physical therapist treats a patient post femur fracture with external fixation. While monitoring the patient during an exercise session, the therapist observes clear drainage from a distal pin site. Which of the following actions is the MOST appropriate for the therapist to take? A. Discontinue the exercise session and contact the referring physician B. Use a gauze pad to absorb the drainage and notify nursing C. Use a gauze pad to absorb the drainage and continue with the exercise session D. Document the finding and discontinue the exercise session

C. Use a gauze pad to absorb the drainage and continue with the exercise session Clear drainage from a pin site is not uncommon and should not be viewed as a sign of infection or any other serious medical complication. The exercise session can continue after the drainage has been aborbed.

A 65-year-old male had a right parietal lobe infarct 6 weeks ago. Since the CVA, he requires moderate assistance for ADLs, minimal/moderate assistance to ambulate in his home environment with a walker, and moderate assistance to maintain static standing balance. Based on the scenario, which of the following physical therapy interventions would be LEAST appropriate at this time in his recovery? A. Visual feedback for midline orientation B. Seated activities lifting both hands in midline C. Use of cane in unaffected side for walking D. ADL training using both hands to drink from a cup

C. Use of a cane in unaffected side for walking Midline activities are main focus of treatment. Holding cane on unaffected side will further promote imbalance causing leaning to affected side, hence it should be avoided.

An 85-year-old patient with chronic congestive heart failure was recently admitted to the hospital. Which of the following plans for prophylactic respiratory care is LEAST appropriate? A. Frequent turning and positioning every two waking hours B. Gentle percussion two times per day C. Vigorous chest vibrations, with the foot of the bed elevated, four times per day D. Gentle coughing and deep breathing exercises every four hours

C. Vigorous chest vibrations, with the foot of the bed elevated, four times per day The patient is already struggling enough to get blood to his extremities and now you are raising the foot of the bed and making it even more difficult.

A 58-year-old patient recently suffered an above knee amputation and underwent surgery. A physical therapist is examining the wound site and sees an exudate. Which of the following findings indicates an infected wound and should be reported to be physician? A. Dark red blood B. Bright red blood C. Viscous yellow exudate D. Serosanguineous exudate

C. Viscous yellow exudate Yellow = infected Serosanguineous- watery and pink/red

A PT is analyzing a patient's posture and observes the plumb line falls through the greater trochanter, anterior to the patella, and just anterior to the lateral malleolus. What can the PT document about the patient's posture? A. Hip retroversion is present B. Posture is normal C. Forefoot pronation is present D. Genu recurvatum is present

D. Genu recurvatum is present The line is going anterior to the knee which pushes the knee into hyperextension.

What conditions are the abnormal heart sounds S3 and S4 associated with, respectively? A. S3= MI or HTN; S4= CHF B. S3= tracheal stenosis; S4= asthma C. S3= closure of mitral and tricuspid valves; S4= closure of pulmonary valves D. S3= CHF; S4= MI or HTN

D. S3= CHF; S4= MI or HTN S3 is heard in early diastole (after S2) and is associated with CHF. S4 is heard in late diastole (before S1) and is associated with an MI or hypertension.

Joe presents to the clinic with a large burn on his forearm. When the therapist tries to touch it, he does not complain of pain on pressure or light touch. Which of the following is the MOST LIKELY presentation of this wound? A. Mixed red, waxy white appearance with blanching and slow capillary refill B. Erythematous pink with no blisters C. Erythematous with blanching and quick capillary refill D. White or tan with no blanching

D. White or tan with no blanching Full thickness burns are white or tan and are usually anesthetic with poor circulation.

A physical therapist administers iontophoresis to a patient with a lower extremity ulceration in an attempt to promote tissue healing. Which of the following ions would BEST meet the stated goal? A. Acetate B. Magnesium C. Lidocaine D. Zinc

D. Zinc Zinc, from zinc oxide, is a positively charged ion used to promote healing, most often with open lesions and ulcerations.

55-year-old, female patient five weeks post-MI, clinically stable, walks in the outpatient cardiac rehab department after a period of aerobic conditioning. The goal of treatment is to strengthen the lower extremity musculature. At present she is at 9 METS. What will be the best measure to monitor the responses for the resistive exercise? A. BP, HR, VO2max B. Maximum oxygen consumption C. Heart rate reserve D. Rate pressure product

D. Rate pressure product RPP is used to estimate the increased metabolic demand that exercise places on the heart. It incorporates BP which is important to monitor. RPP = HR x SBP

Nancy suffered a shoulder injury and had been immobilized for 4 weeks. She came for her first physical therapy appointment after removal of the cast and the physical therapist needs to check the functioning of GH ligaments. In order to stretch the inferior ligament of GH joint: A. Arm should be elevated to 45 degrees B. Arm should be extended to 30 degrees C. Arm should be elevated to 90 degrees D. Arm should be externally rotated to 45 degrees

C. Arm should be elevated to 90 degrees Inferior glenohumeral ligament is taut/ stretched with 90 degrees of shoulder abduction. Middle glenohumeral ligament is taut between 45-60 degrees of abduction.

A PT student is studying the effects and characteristics of ultrasound. Which of the following statements regarding therapeutic ultrasound is true? A. A 40% duty cycle would have an off time of 4 ms and an on time of 6 ms B. A 40% duty cycle would have an off time of 4 ms and on time of 10 ms C. A 40% duty cycle would have an off time of 6 ms and on time of 4 ms D. A 40% duty cycle would have an off time of 10 ms and on time of 4 ms

C. A 40% duty cycle would have an off time of 6 ms and an on time of 4 ms Duty cycle = duration of pulse/pulse period x 100 40= x * 100 x = 0.4 OR duration of pulse/pulse period = 0.4 A- pulse duration of 10 B- pulse duration of 14 C- pulse duration of 10 D- pulse duration of 14 You now divide the on time by the pulse duration and C is the only one that gets 0.4.

A PT student suffered low back pain due to excessive studying during NPTE week and is using hot packs in the PT clinic. When using hot packs, which of the following scenarios is MOST likely to result in burns? A. Using 6 layers of toweling between the hot pack and the patient B. A treatment time of 30 minutes C. A hot pack heated by immersion in water heated to 205 degrees Fahrenheit D. The patient's skin having too much hair

C. A hot pack heated by immersion in water heated to 205 degrees Fahrenheit Hot packs can be used for 20-30 minutes so you can rule out B. The recommendations are 6-8 layers of toweling so you can rule out A. Too much hair will not have an impact so you can rule out D. A hot pack should be heated to 165-170 degrees Fahrenheit.

A study is looking at the effects of different exercise options for a patient with knee OA. The study compared pool exercise, treadmill walking, and ankle weights on knee pain in 30 adults. An appropriate statistical test to assess the effectiveness of treatment in three groups is: A. Spearman's rho B. Two paired t-tests C. ANOVA D. Chi square

C. ANOVA You are looking at three groups and ANOVA is the only option that allows for that.

A patient complains of sharp pain at the ankle. According to the patient, the ankle was injured while playing basketball last week. When performing the pictured test on the patient, the PT finds that the patient's ankle remains in a neutral position. This would indicate: A. Medial meniscus tear B. Anterior talofibular ligament injury C. Achilles tendon rupture D. Calcaneofibular ligament tear

C. Achilles tendon rupture This is an image of a Thompson test which tests for Achilles tendon rupture. You would expect the ankle to go into PF if the Achilles tendon is intact.

A 72-year-old patient admitted to the hospital with a diagnosis of pneumonia presents with shallow breathing and drowsiness. The physical therapist is reviewing the lab reports and sees the following values: pH- 7.30, HCO3- 22 mEq/L, PaCO2- 52 mmHg, O2- 85% Which of the following is the MOST APPROPRIATE interpretation of the ABG? A. Partially compensated respiratory acidosis B. Partially compensated metabolic alkalosis C. Uncompensated respiratory alkalosis D. Uncompensated respiratory acidosis

D. Uncompensated respiratory acidosis pH- low; acidic (7.35-7.45) HCO3- normal (22-26) PaCO2- high (35-45) You know it is not partially compensated because HCO3 is normal and with partially compensated all three values need to be abnormal so you can rule out A and B. Since PaCO2 is the abnormal value it is respiratory and since the pH is low it is acidosis.

A physical therapist has determined that a patient has unilateral vestibular hypofunction. What would be the MOST appropriate INITIAL home exercise to provide to the patient? A. Brandt-Daroff exercsises B. VOR x2 walking C. Walking towards a busy wallpaper D. VOR x1 sitting

D. VOR x1 sitting Brandt-Daroff exercises are used for BPPV as a HEP. You always start with VOR x1 and THEN progress to VOR x2. VOR x1 involves looking at the patient's thumb while rotating the head. VOR x2 involves looking at the patient's thumb while rotating the head and moving the thumb in the opposite direction.

A 53-year-old female patient presented to the clinic complaining of a large, painful, non healing wound on the medial aspect of the right ankle. Upon examination, the PT noted lower extremity hyper pigmentation and hemosiderin deposition, hypergranulation tissue present at the wound base, and irregularity in shape. What kind of ulcer/wound is it? A. Pressure injury B. Diabetic foot ulcer C. Arterial ulcer D. Venous ulcer

D. Venous ulcer

You are testing a patient's deep tendon reflexes during an evaluation for low back pain in outpatient physical therapy. Which of the following is MOST likely to be consistent with a false negative patellar reflex? A. 2+ reflex and the patient has a confirmed L4 nerve root syndrome B. 1+ reflex and the patient has a confirmed S1 nerve root syndrome C. 3+ reflex and the patient has a confirmed L4 nerve root syndrome D. 2+ reflex and the patient does not have L4 nerve root syndrome

A. 2+ reflex and the patient has a confirmed L4 nerve root syndrome 2+ reflexes are considered normal according to the reflex grading system. A false negative indicates that the test is normal but the patient has pathology. The correct answer states that the reflexes are normal but the patient has a pathology that affects the L4 nerve root.

A 58-year-old female patient with uncomplicated MI has been discharged from the acute care. Which of the following activities would be MOST appropriate during early phase two rehabilitation? A. 30 minutes of walking at 4 mph with no incline B. 15 minutes of elastic band training with yellow elastic bands C. 15 minute upper limb workout with 2-pound hand weights D. 30 minutes walking at 4 mph with 5% incline

A. 30 minutes walking at 4 mph with no incline Phase II: 2-3 sessions/week, 70-85% of HRmax, 30-60 minutes with 5-10 minutes for warm-up and cool-down NO RESISTANCE TRAINING YET

A physical therapist reads in the medical record that the foot progression angle of a four-year-old child was recorded as -10 degrees. Which range of motion measurement at the hip would MOST likely be associated with the obtained foot progression angle? A. 75 degrees of hip medial rotation and 25 degrees of hip lateral rotation B. 35 degrees of hip medial rotation and 70 degrees of hip lateral rotation C. 30 degrees of hip medial rotation and 20 degrees of hip lateral rotation D. 45 degrees of hip medial rotation and 45 degrees of hip lateral rotation

A. 75 degrees of hip medial rotation and 25 degrees of hip lateral rotation A negative foot progression angle represents in-toeing and a positive number is for out-toeing. Exaggerated hip medial rotation (i.e., 75 degrees) and diminished hip lateral rotation (i.e., 25 degrees) are commonly observed with femoral anteversion which is the most common cause of in-toeing in children. A foot progression angle of -10 degrees indicates 10 degrees of in-toeing.

A physical therapist reads in the medical record that a patient has an ejection fraction of 40%. Which class of medication is the patient MOST likely to be taking? A. Angiotensin-converting enzyme inhibitor agents B. Nitrate agents C. Anticholinergic agents D. Thrombolytic agents

A. Angiotensin-converting enzyme inhibitor agents The ejection fraction is a measure of left ventricular contractility. A normal ejection fraction is 55-70%. Anything less than 55% of the blood pumped out of the ventricles with each heartbeat is abnormal and indicates impairment in left ventricular function. Ejection fraction is decreased in patients with left-sided congestive heart failure since the left ventricle is unable to maintain a normal cardiac output. Angiotensin-converting enzyme inhibitor (ACE inhibitor) agents decrease BP and afterload by suppressing the enzyme that converts angiotensin I to angiotensin II. This medication is indicated for medical conditions presenting with a low ejection fraction such as congestive heart failure.

A physical therapist is working with a patient suffering from pain and limitation in range of motion at the right shoulder. The patient has limitation mainly with extension and external rotation? As a physical therapist, the MOST appropriate treatment would be: A. Anterior glide at the shoulder B. Posterior glide at the shoulder C. Inferior glide at the shoulder D. Joint mobilizations are contraindicated

A. Anterior glide at the shoulder Anterior glide of shoulder joint is used to increase external rotation, extension and horizontal abduction.Posterior glide helps to improve internal rotation, flexion and horizontal adduction. Inferior glide is used to increase abduction.

A patient who has low back pain and a medical history of chronic obstructive pulmonary disease and heart failure has been referred for physical therapy. Which of the following interventions requires MOST patient monitoring? A. Aquatic therapy B. Supine lumbar traction C. Core strengthening D. Lumbar soft-tissue mobilization

A. Aquatic therapy In patients who have heart failure, the concern is that the increase in cardiac volume that occurs during immersion may overwhelm the pumping ability of the heart (Cameron, pp. 345-346). Patients who have chronic obstructive pulmonary disease must be carefully monitored while immersed due to the additional respiratory challenge (Cameron, p. 346).

The manual therapy technique appropriate to correct a bilateral closing restriction of T2 on T3 is: A. Central PA glide on the spinous process of T3 while stabilizing T2 B. Central PA glide on the spinous process of T2 while stabilizing T3 C. Unilateral PA glide on the right transverse process of T3 while stabilizing T2 D. Unilateral PA glide on the left transverse process of T3 while stabilizing T2

A. Central PA glide on the spinous process of T3 and T2 This is a bilateral closing restriction so you don't mobilize unilaterally. You can rule out C and D. If there is a closing restriction that means it is having difficulty closing. "Bottoms up". By mobilizing T3 anteriorly it will also move superiorly and close the gap.

A patient has nontraumatic neck and shoulder pain, decreased hand dexterity, paresthesia in the right upper extremity, hyperreflexia, and urinary retention with overflow incontinence. The patient MOST likely has which of the following conditions? A. Central cord syndrome B. Cervical transverse ligament tear C. Cervical disc herniation D. Cervical myelopathy

A. Central cord syndrome Central cord syndrome: loss of motor in the UE's, loss of pain and temperature; result of hyperextension injury

A patient who has gastroesophageal reflux disease is MOST likely to benefit from education to reduce consumption of which of the following types of food? A. Coffee, fatty foods B. Coffee, dairy foods C. High-sugar foods, fatty foods D. High-sugar foods, dairy foods

A. Coffee, fatty foods Modifications to help manage symptoms of gastroesophageal reflux disease includes avoiding eating large meals that can distend the stomach and avoiding items such as chocolate, peppermint, alcohol, caffeinated coffee, and fried and/or fatty foods.

In the gait lab, the PT notices that the patient deviates the trunk to the left during the swing phase of the right leg. Which of the following is the MOST LIKELY cause for this deviation? A. Contracture of the right ankle plantarflexors B. Contracture of the left hip flexors C. Shorter limb length of the right leg D. Weakness of the right knee extensors

A. Contracture of the right ankle plantarflexors If there is a contracture of the right PF's then that will make the right leg longer and the patient will compensate by leaning to the left.

Which of the following activities should be the PRIMARY emphasis of a PT treatment program for a child who has athetoid cerebral palsy with abnormal, involuntary movement? A. Facilitate co-contraction patterns and encouraging control in voluntary movement B. Preservation of strength and muscle tone C. Facilitating use of primitive reflexes to perform fine motor skills D. Prevention of contractures and determine the best method of mobility

A. Facilitate co-contraction patterns and encouraging control in voluntary movement Athetoid CP is a disorder of the basal ganglia. It is characterized by involuntary movements that are slow and writhing.

A PT is assessing the lung function of a patient with a diagnosis of COPD. The PT would like to assess the amount of air in the lungs after a NORMAL exhalation. Which of the following parameters is the PT referring to? A. Functional residual capacity B. Tidal volume C. Expiratory reserve volume D. Vital capacity

A. Functional residual capacity Tidal volume is a normal exhalation. Expiratory reserve volume everything you can exhale with maximal exhalation. Vital capacity is IRV + TV + ERV.

A 30-year-old male client visits a town which is 9,000 feet above sea level. What are the INITIAL cardiovascular responses during his first few days in town? A. Increased BP, increased cardiac output with tachycardia, and no significant changes in SV B. Decreased BP, decreased cardiac output with bradycardia, and increased SV C. Increased BP, decreased cardiac output with bradycardia, and increased SV D. Decreased BP, increased cardiac output with tachycardia, and increased SV

A. Increased BP, increased cardiac output with tachycardia, and no significant changes in SV When you think of going up in altitude imagine that it is your first time going to Colorado and seeing the mountains. You are super excited and nervous and you feel your heart beating faster. This could be your excitement, but it is also an effect of altitude. So, you can rule out B and C. Your blood pressure also "increases" with an "increase" in altitude so you can rule out D.

A 69-year-old patient has symptoms of lightheadedness, along with numbness and tingling of arms and legs. His blood gas report is: pH- 7.48, PaCO2- 30 mm Hg, and HCO3- 26. Which of the following is the most INAPPROPRIATE treatment? A. Intravenous injection of sodium bicarbonate B. CO2 administration C. Pain control D. Using rebreathing mask/paper bag

A. Intravenous injection of sodium bicarbonate The pH is high (7.35-7.45) you immediately know he is in an alkalosis state. The PaCO2 is also abnormally low (35-45) so it is respiratory alkalosis. HCO3 is normal but both pH and PaCO2 are abnormal so it seems the body is not doing a good job of balancing PaCO2 and HCO3 to return the pH to a normal state so this is uncompensated respiratory alkalosis.

A physical therapist treats a patient who sustained deep partial-thickness burns to the anterior surface of both lower extremities. After identifying an irregularity in the patient's laboratory results, the referring physician discusses with the therapist the possibility of discontinuing the use of the topical medication silver sulfadiazine. Which of the following findings is MOST likely related to the use of silver sulfadiazine? A. Leukopenia B. Peripheral edema C. Hypokalemia D. Altered pH balance

A. Leukopenia Silver sulfadiazine is a sulfa drug that can produce a decrease in the number of WBC's circulating (leukopenia)

A patient who has right shoulder pain exhibits bruising, palmar erythema, and signs of confusion. Which of the following organs is MOST likely involved? A. Liver B. Pancreas C. Kidney D. Spleen

A. Liver Right shoulder pain, bruising, palmar erythema, and confusion are all among the signs and symptoms of liver disease

A patient who has a T3 spinal cord injury (ASIA Impairment Scale A) becomes flushed and exhibits diaphoresis and bradycardia. The patient also reports having a headache. Which of the following actions is MOST appropriate for a physical therapist? A. Loosen the patient's pants B. Lie the patient flat on a mat table C. Clamp the patient's urinary catheter D. Apply an abdominal binder to the patient

A. Loosen the patient's pants The patient's presentation is consistent with autonomic dysreflexia. One of the common irritants or causes is tight or restrictive clothing below the level of the lesion. It is important to loosen any restrictive clothing if a patient presents with autonomic dysreflexia.

A physical therapist completes a developmental assessment on an infant. Assuming normal development, which of the following positions would typically be the LAST to occur? A. Modified plantigrade B. Quadruped C. Ring sitting D. Bridging

A. Modified plantigrade Bridging --> ring sitting --> quadruped --> modified plantigrade Modified plantigrade typically occurs at 10 months. Modified plantigrade is characterized by lower extremity weight bearing in supported standing while leaning the UE support on a table or weight bearing surface.

When observing a patient ambulating, a PT notices that the patient's gait has the following characteristics: impaired trunk control and ataxic gait. There is also circumduction to assist with foot clearance due to weakness of dorsiflexors of the right foot. This gait pattern is often observed in patients with a diagnosis of: A. Multiple sclerosis B. Parkinson's disease C. Duchenne muscular dystrophy D. Guillain barre syndrome

A. Multiple sclerosis Parkinson's is characterized by a shuffling (festinating) gait so they can hardly lift their foot off of the ground and will not utilize circumduction. Duchenne muscular dystrophy is characterized by the Gower sign. Guillain barre is presented like a LMN lesion and MS is like an UMN lesion. An UMN lesion would be characterized by an ataxic gait so MS is the correct answer. MS is also characteristically unilateral.

A PT is treating a patient who was originally diagnosed with a pressure injury stage 3. The patient presents to clinic with the ulcer shown in the picture. Which of the following findings is MOST accurate? A. No change has occurred in the pressure injury B. The pressure injury has improved to Stage 1 C. The pressure injury has improved to Stage 2 D. The pressure injury has become unstageable

A. No change has occurred in the pressure injury Yes, this still presents as a stage 3 pressure injury BUT you NEVER change the stage of a pressure injury regardless of how it progresses!!! If it starts as a stage 3 pressure injury it will remain that throughout the healing process.

A physical therapist performs segmental breathing exercises with a patient post atelectasis. Which of the following hand placements would be the MOST appropriate to emphasize of the lingula? A. On the left side of the chest below the axilla B. Below the clavicle on the anterior chest wall C. Over the posterior aspect of the lower ribs D. On the right side of the chest below the axilla

A. On the left side of the chest below the axilla The lingula is a segment of the left upper lobe. Placing the hands on the left side of the chest below the axilla would provide tactile stimulation to facilitate expansion of the chest wall to improve ventilation of the left upper lobe.

Which of the following responses is a positive finding for the test shown in the photograph? A. Palpation of pulsating mass B. Presence of back or flank pain C. Distention on the costal margin D. Costovertebral tenderness

A. Palpation of pulsating mass The test shown in the photograph would be necessary if a patient reports a pulsing or pounding sensation in the abdomen during increased activity or when lying in supine position. A positive finding would be a palpable pulsating mass and would indicate an abdominal aneurysm.

An acute care physical therapist is reading the reports of a patient with chronic kidney disease. The patient appears to be confused and lethargic. His body is warm to touch and he is in bradycardia. His lab findings are as follows: pH: 7.20; HCO3: 21mEq/L; PaCO2: 34 mm Hg. Which of the following is the MOST APPROPRIATE diagnosis for this patient? A. Partially compensated metabolic acidosis B. Uncompensated metabolic acidosis C. Compensated metabolic acidosis D. Uncompensated metabolic alkalosis

A. Partially compensated metabolic acidosis Chronic kidney disease with renal failure results in acidosis because the failing kidney not only is unable to rid the body of excess acids but also cannot produce necessary bicarbonate. Normal values for pH = 7.35-7.45, PaCO2 = 35-45 mm Hg; and HCO3- = 22-26 mEq/L. First step: Check the pH value. If it is less than 7.4 it is in an acidotic state. This rules out D. Second step: Is it respiratory or metabolic? If it is metabolic HCO3 will be abnormal. If it is respiratory PaCO2 will be abnormal. For this instance, it appears both HCO3 and PaCO2 are not within normal values so the body is trying to compensate for the pH. However, the pH is still abnormal so it is partially compensated for.

A physical therapist works with a patient diagnosed with Down Syndrome. The therapist determines that the patient has abnormalities in muscular tone consistent with the diagnosed condition. Which of the following techniques would be the MOST beneficial when treating the patient's tone abnormalities? A. Quick stretch B. Deep pressure C. Prolonged icing D. Neutral warmth

A. Quick stretch Providing a quick stretch to the muscle helps activate muscle spindles and results in enhanced muscular contraction. For patients with low tone, this technique may help facilitate increased activity in affected muscles. The other options all decrease tone!!!

A college student presents with complaints of right-sided neck pain. During the AROM examination, the physical therapist observes the following osteokinematic neck motions: full side-bending left, full rotation to the left, full forward flexion, limited and painful extension, limited and painful right side-bending, and limited and painful right rotation. Based on this pattern, what is the MOST likely arthrokinematic restriction? A. Restriction with downglide of a facet on the right B. Restriction with upglide of a facet on the right C. Restriction with downglide of a facet on the left D. Restriction with upglide of a facet on the left

A. Restriction with downglide of a facet on the right In the cervical spine, rotation and side bending occur to the same side (except C1-2). With extension limited, the most likely restriction is a downglide. With right rotation, and right side bending, the right cervical facets would be going into a downglide. If this motion is limited, then restriction of downglide with facet on the right is the most likely problem.

A right pleural irritation results in sharp localized pain aggravated by respiratory movement. Which of the following positions is MOST likely to reduce the pain? A. Right side lying B. Left side lying C. Prone lying D. Supine lying

A. Right side lying In pleural pain, pain is alleviated by lying on the affected side (in this case- right side), which diminishes the movement of that side of the chest. This is called auto-splinting.

A physical therapist prepares to examine a patient's triceps using a reflex hammer. Which of the following positions for the patient's upper extremity is the MOST appropriate to test the triceps reflex? A. Shoulder extension and elbow flexion B. Shoulder flexion and elbow extension C. Shoulder extension and elbow extension D. Shoulder flexion and elbow flexion

A. Shoulder extension and elbow flexion An acceptable alternate position to test the triceps reflex would be shoulder abduction and elbow flexion.

A patient with weakness of the muscle group being tested in the photograph would have the most difficulty with which of the following activities? A. Stepping down from a 7" step B. Walking on a level surface C. Getting up from a chair D. Leaning forward in standing position

A. Stepping down from a 7" step The photograph demonstrates terminal knee extension which is strengthening the quadriceps muscle. Stepping down from a 7" step would require the quadriceps to contract eccentrically to slow the lowering.

A 30-year-old male goes to Colorado for the ski season. He's never been to that state before and learns that the city where he will be staying is over 8,000 feet above sea level. What are the INITIAL cardiovascular responses during his first few days in Colorado? A. Temporarily slightly increased BP, increased cardiac output with tachycardia, and no significant changes in SV B. Decreased BP, decreased cardiac output with bradycardia, and increased SV C. Increased BP, decreased cardiac output with bradycardia, and increased SV D. Decreased BP, increased cardiac output with tachycardia, and increased SV

A. Temporarily slighted increase BP, increased cardiac output with tachycardia and no significant changes in SV The initial cardiovascular response to altitude is characterized by an increase in cardiac output with tachycardia, no change in stroke volume, whereas blood pressure may be slightly increased

A physical therapist needs to check the walking gait of a chronic female stroke patient (10 years ago) and decides to record a video in slow motion. In regards to normal walking gait, dorsiflexors terminate their action by the end of which gait phase? A. The end of loading response B. The end of terminal swing C. The end of terminal stance D. The end of midswing

A. The end of loading response Dorsiflexors (tibialis anterior, extensor digitorum longus, extensor hallucis longus) act during the initial stance phase to decelerate forefoot lowering and draw tibia forward following initial contact. In other words, they contract eccentrically to slow plantar flexion. The action of dorsiflexors terminates at the end of loading response and the plantarflexors begin their action.

A physical therapist works with a patient post CVA on a therapeutic exercise program. The therapist assists the patient in lateral weight shifting activities while positioned in prone on elbows on a mat table. Which of the following facilitation techniques would BEST allow the patient to improve dynamic stability with this activity? A. Alternating isometrics B. Approximation C. Rhythmic initiation D. Timing for emphasis

B. Approximation Approximation is a therapeutic exercise technique designed to facilitate contraction and stability through joint compression. The compression force is most often applied to joints through gravity acting on body weight, manual contacts, or weight belts.

A 40-year-old male patient has been diagnosed with Spondylolisthesis, a condition where the body of the involved lumbar vertebra slips anteriorly upon the body of the vertebra below. What is the most important recommendation for him? A. Avoid knee flexion and adduction B. Avoid hyperextension at the spine C. Avoid posterior pelvic tilt D. Avoid tying shoes or picking things up from the floor

B. Avoid hyperextension at the spine In spondylolisthesis, repetitive extension and torsion activities exacerbate the pain of the patient. So, it will be advisable to avoid hyperextension activities.Flexion exercise and stabilization activities are helpful for the patient.

A physical therapist is assessing a patient with a sudden onset of dizziness two days ago. Upon performing the Roll Test, the patient had geotropic nystagmus, stronger on the right side as compared to the left. Which of the following is the MOST APPROPRIATE intervention for this patient? A. Bar-B-que roll for cupulolithiasis maneuver starting on the R B. Bar-B-que roll maneuver for canalithiasis starting on the R C. Semont maneuver for canalithiasis with side lying on the L D. Semont maneuver for cupulolithiasis with side lying on the R

B. Bar-B-que roll maneuver for canalithiasis starting on the R Horizontal canal canalithiasis presents as geotropic nystagmus which the question states the patient has. The most appropriate intervention for canalithiasis is the bar-b-que roll with the head turned to the involved side first. Note: Canalithiasis is much more common than cupulolithiasis

A physical therapist is treating a patient in an acute care setting with a hematologic disorder. Which of the following test results would be the MOST appropriate to monitor on a daily basis in order to ensure patient safety during physical therapy? A. Hemoglobin B. Complete blood count C. Arterial blood gas D. Blood glucose

B. Complete blood count While hemoglobin is important to monitor, it is not the only thing you must monitor. A CBC allows the therapist to determine trends and to recognize abnormal lab values that may require modification or deferment of services.

During lung auscultation, a physical therapist asks a patient to continuously say "E." Transmission of an "A" sound is heard over the right lower lobe when the patient says "E." Which of the following conditions is MOST likely present in the right lower lobe? A. Atelectasis B. Consolidation C. Pleural effusion D. Pneumothorax

B. Consolidation When a patient has consolidated lung tissue or increased secretions, egophony will be present. Egophony is a voice sounds test and is the transmission of an "A" sound when the patient says "E." Voice sounds will increase in patients who have consolidation .

Which of the following burn wound classifications is MOST likely to produce the keloid scar shown in the picture? A. Superficial burn B. Deep partial-thickness burn C. Superficial partial-thickness burn D. Subdermal burn

B. Deep partial-thickness burn Superficial burn: partial epidermis Superficial partial-thickness burn: partial epidermis and dermis Deep partial-thickness burn: complete epidermis destruction and partial dermis; all that is left is the dermis so the body tries to compensate with scarring but overcompensates and forms a keloid scar Full thickness: complete epidermis and dermis destruction Subdermal: epidermis, dermis, and fat destruction

A pt arrived at an outpatient clinic with chief c/o LBP. Upon further evaluation, the pt mentioned taking calcium channel blockers. Which of the following is MOST LIKELY accurate regarding the medication effect on the cardiovascular system? A. Constrict coronary blood vessels B. Dilates coronary blood vessels C. Activates coronary vasospasms D. No effect towards coronary vasospasms

B. Dilates coronary blood vessels C and D- calcium channel blockers INHIBIT coronary vasospasms

A 45-year-old male patient underwent below knee amputation of the right lower extremity and is using a prosthesis. The PT notices that the patient demonstrates a certain gait deviation and concludes that it is caused by the keel being too short. Which of the following prosthetic causes would LEAST likely cause the same deviation? A. Dorsiflexion stop too soft B. Dorsiflexion stop too stiff C. Socket too far anterior D. Socket excessively flexed

B. Dorsiflexion stop too stiff If someone has too much PF they will have hyperextension at the knee. If the DF stop is too soft it will allow for DF and thus will allow for knee FLEXION. Whereas if the dorsiflexion stop is too stiff it does not allow for DF, so the foot will be in a plantarflexed position and the knee will be extended. A, B, and C all would cause knee flexion.

A patient complains of persistent wrist pain after painting a house a few weeks ago. The patient demonstrates signs and symptoms consistent with de Quervain's tenosynovitis. An appropriate special test to confirm the diagnosis is: A. Phalen's test B. Finkelstein's test C. Froment's sign D. Cozen's test

B. Finkelstein's test Phalen's test- detects carpel tunnel syndrome Froment's test- examines weakness in the abductor pollicis and short head of the flexor pollicis brevis muscles, indicates ulnar nerve entrapment at the elbow or wrist Cozen's test- examines lateral epicondylitis at the elbow

A patient's medical record indicates the presence of a significant electrolyte imbalance. Which of the following conditions would MOST commonly be associated with an increased risk of cardiac arrest if left untreated? A. Hypercalcemia B. Hyperkalemia C. Hypermagnesemia D. Hypernatremia

B. Hyperkalemia Hyperkalemia refers to an excessive level of potassium in the blood. If levels exceed 8.5 mEq/L it can cause respiratory paralysis or cardiac arrest.

During the Clinical Test for Sensory Integration and Balance, a patient demonstrates increased sway when standing on a foam surface with eyes closed and when standing on foam with vision obscured by a dome. The sway is normal during all other conditions. Which of the following patient problems is the MOST likely reason for the findings? A. Inability to effectively adapt sensory information B. Inability to use vestibular input for postural control C. Dependence on the visual system for postural control D. Dependence on the somatosensory system for postural control

B. Inability to use vestibular input for postural control Patients who exhibit increased sway when standing on foam with eyes closed and foam with obscured vision demonstrate a vestibular loss pattern. Patients who are visually dependent will also exhibit increased sway when standing on a firm surface with eyes closed or with vision obscured.

A patient complains of gradual onset of dyspnea with exertion. He is a teacher by occupation and noticed that he needs to take frequent breaks while teaching/talking because he gets fatigued easily. He has been a smoker for over 30 years and reports an increase in cough with expectoration. He is suspected to have obstructive pulmonary disease. Which of the following will most likely NOT be associated with this condition? A. Hypertrophy of accessory muscles of respiration B. Increase in the amount of exhalation in 1 sec C. Obliteration of Lovibond angle D. Bluish discoloration of the mucous membrane

B. Increase in the amount of exhalation in 1 sec The symptoms of COPD include: Hypertrophy of accessory muscles of ventilation, pursed-lip breathing, cyanosis (bluish discoloration of skin or mucous membrane), and digital clubbing (obliteration of Lovibond angle is grade 2 of stages of clubbing). Spirometry is the GOLD standard test for diagnosis of COPD which indicates a decrease in the expiratory flow rates (especially FEV1 which is the amount of air exhaled in 1 sec).

A physician orders an electrocardiogram (ECG) for a patient diagnosed with congestive heart failure. The medical record indicates the patient is currently taking digitalis. What effect would digitalis MOST likely have on the patient's ECG? A. Sinus tachycardia B. Lengthened PR interval C. Lengthened QT interval D. Elevated ST segment

B. Lengthened PR interval

A physical therapist treats a patient diagnosed with cystic fibrosis. As part of the treatment session the therapist attempts to improve the efficiency of the patient's breathing. Which of the following techniques is the MOST appropriate to encourage full expansion at the base of the lungs? A. Manual percussion over the posterior portion of the ribs with the patient in prone B. Manual contacts with pressure over the lateral borders of the ribs with the patient in supine C. Manual vibration over the lateral portion of the ribs with the patient in sidelying D. Manual cues over the epigastric area with the patient in supine

B. Manual contacts with pressure over the lateral borders of the ribs with the patient in supine Applying direct pressure with the hands on the lateral borders of the ribs with the patient in supine can promote a more efficient breathing pattern

A 48-year-old patient suffering from diabetes mellitus was referred to an outpatient clinic with a diagnosis of adhesive capsulitis. The physical therapist examined the patient and decided to utilize mobilizations as part of the initial treatment. During a grade 2 inferior glide on the glenohumeral joint, the patient complains of severe pain and irritability. Which of the following interventions is the BEST course of action for the PT to make? A. Do active assisted exercises and wait for pain to subside B. Moist heat, glides in opposite direction to the direction of resistance C. Interferential therapy, moist heat and active assisted exercises D. Ultrasound, moist heat and active exercises

B. Moist heat, glides applied in opposite direction to the direction of restriction Painful joint, reflex muscle guarding, and muscle spasm can be treated with gentle joint-play techniques to stimulate neuro physiological and mechanical effects. Grades I and II are primarily used for treating joints limited by pain or muscle guarding. The moist heat is given as an adjunct to reduce pain. Assistive exercises will not reduce the irritability.

A 48-year-old male patient reports SOB and complains of fatigue. During the examination, the PT examines the patient's heart sounds before starting an exercise program. Which valve is being auscultated in the picture? A. Tricuspid valve B. Pulmonary valve C. Mitral valve D. Aortic valve

B. Pulmonary valve

A patient is being seen in a PT clinic one week following PCL reconstruction. When planning for the patient's upcoming session, what exercise focus should the therapist consider implementing? A. Hamstring stretching B. Quadriceps strengthening in closed kinematic chain C. Hamstring strengthening D. Quadriceps stretching

B. Quadriceps strengthening in closed kinematic chain Strengthening the quadriceps is emphasized for knee control after PCL reconstruction because it acts as a dynamic restraint to posterior tibial translation. Hamstrings strengthening will increase posterior translation of the tibia.

A physical therapist has prescribed an exercise program for a patient with bilateral knee pain and morbid obesity. Which of the following is the LEAST appropriate exercise for this patient? A. Weight-lifting program B. Step aerobics C. Aquatic exercises D. Stationary bicycling

B. Step aerobics Individuals who are obese are at higher risk for the development of knee osteoarthritis (Goodman, p. 37). Aerobic programs should be designed to minimize joint stress and impact in this population (Brody, p. 324). Step aerobics is the least appropriate intervention of those listed.

All of the following are guidelines to prevent pressure ulcers EXCEPT: A. Position the patient at a 30-45 degree oblique angle when side lying B. Use a donut-type cushion when seated in wheelchair C. Elevate the head of the bed to no greater than 30 degrees when the patient is supine D. Encourage frequent position changes at least every 2 hours in bed, every hour while seated, and every 15 minutes if the patient can move himself

B. Use a donut-type cushion when seated in wheelchair Donut type of cushion should not be used for pressure reduction as the rim of the cushion creates pressure, which occludes capillaries and deprives the local tissue of a proper blood supply and flow.

A patient has an ankle-brachial index (ABI) of 1.5. Which of the following conditions affecting the lower extremity should a physical therapist suspect? A. Arterial aneurysm B. Arterial thrombosis C. Arterial calcification D. Arterial occlusive disease

C. Arterial calcification Ankle-brachial index is a ratio of the systolic blood pressure at the ankle and the brachial systolic pressure. The normal value of the ankle-brachial index is 1.0, indicating similar blood flow in the ankle and brachial arteries. An ankle-brachial index greater than 1.1 relates to arterial calcification in the leg. With arterial calcification, the artery cannot be fully compressed for valid measurement of arterial pressure at the ankle. An ankle-brachial index greater than 1.1 is mostly found in patients who have diabetes. Aneurysm, thrombosis, and occlusive disease: ABI will be LESS than 1

A 79-year-old patient is receiving physical therapy for gait instability. She is moderately ambulatory and has difficulty with community distances. Which of the following outcomes measure is MOST appropriate to determine her fall risk? A. Fullerton assessment of balance B. Function in sitting test C. Berg balance scale D. 6-minute walk test

C. Berg balance scale The Berg Balance Scale is the most appropriate given that the patient is only capable of ambulating minimal to moderate distances. BBS is useful in predicting falls in the elderly. FAB and 6MWT are both too high level for this patient, while the FIST is too low level as it is a measure of early sitting in persons with limited functional ability

A patient with complete C5 tetraplegia works on a forward raise for pressure relief. The patient utilizes loops that are attached to the back of the wheelchair to assist with the forward raise. Which muscles should be particularly strong in order for the patient to be successful with the forward raise? A. Brachioradialis, brachialis B. Rhomboids, levator scapulae C. Biceps, deltoids D. Triceps, flexor digitorum profundus

C. Biceps, deltoids Muscles below the C5 level will not be innervated. The triceps are innervated by C7-C8 and the flexor digitorum profundus is C8-T1 so you can rule out D. The biceps (C5-C6) and deltoids (C5-C6) would both be innervated. The deltoids assist with all shoulder motions except adduction. The biceps act to flex the shoulder, flex the elbow, and supinate the forearm.

A 42-year-old female patient presents with chief complaints of asymmetrical weakness of lower and upper extremities. There is the presence of muscle fasciculations and impaired speech, but her sensations seem relatively intact. The patient's husband mentions that she randomly starts laughing or crying for "no reason at all". Which of the following signs and symptoms should the therapist LEAST likely expect to be positive in this case? A. Presence of memory impairments B. Presence of foot slap while walking C. Cervical flexor muscle weakness D. Reduced vital capacity

C. Cervical flexor muscle weakness These symptoms describe ALS. "ALl Symptoms": UMN + LMN. Cervical EXTENSOR muscle weakness is characteristic of ALS. S/s of ALS: Bulbar muscle weakness, dysphagia, dysarthria, pseudobulbar affect Respiratory muscle weakness, dyspnea, exertional dyspnea Loss of insight, emotional blunting Attention deficits, deficits in cognitive flexibility Irritability, social disinhibition

The patient presents with acute inflammatory demyelinating polyradiculoneuropathy (AIDP), four weeks following a relatively benign respiratory illness with complaints of finger dysesthesias and muscle weakness of the lower extremities. The weakness may progress over hours to days to involve the arms, truncal muscles, cranial nerves, and muscles of respiration. These characteristics are for what condition? A. Amyotrophic lateral sclerosis B. Multiple sclerosis C. Guillain barre syndrome D. CVA

C. Guillain barre syndrome Guillain barre syndrome is caused by a viral infection or respiratory infection. It is characterized by a symmetrical progression of paralysis from LE's to UE's. It progresses over 3-7 days and most people recover within a few weeks to months.

A 28-year-old female patient with AKA presents with a classic marked increase in lumbar lordosis posture. Prior to performing a hands-on objective exam, the PT should anticipate which of the following based on her posture? A. Prosthetic limb is too long B. High posterior prosthetic wall C. High anterior prosthetic wall D. The socket is in excessive abduction

C. High anterior prosthetic wall A high anterior prosthetic wall will imitate tight hip flexors so you would see an anterior pelvic tilt and increased lumbar lordosis.

A 58-year-old patient has chronic obstructive pulmonary disease. During an examination, a PT finds that the patient has a weak, wet cough. Which of the following is MOST appropriate to help this patient clear secretions? A. Assisted coughing in supine position B. Postural drainage in side lying position C. Huffing D. Manual or mechanical percussion

C. Huffing Imagine trying to fog up your mirror. That is the same thing as huffing. You usually huff in three's (HA HA HA) A- they already have a weak cough so they don't need assistance with coughing they just need help with excretion B- we don't know which lobe the secretions are in so we don't know how to position them for this D- this is used to loosen the secretions BEFORE airway clearance

Which of the following treatment guidelines below is LEAST important for diabetic sensory neuropathy? A. Foot care B. Orthotics C. Improving circulation D. Maintain HbA1c less than 7%

C. Improving circulation Improving circulation is least important for Diabetic neuropathy. Foot care is needed to prevent injury. Orthotics is needed for decreasing weight-bearing stresses or offloading weight while walking and HbA1c less than 7 % is very essential to facilitate wound healing.

A physical therapist is treating a patient diagnosed with leukemia who is currently receiving chemotherapy. The patient has been referred to physical therapy to improve their functional mobility and endurance. Which activity would be the least desirable to accomplish the stated goal? A. Yoga B. Bike riding C. Jumping rope D. Swimming

C. Jumping rope Leukemia is a cancer of the blood that occurs when leukocytes change into malignant cells. Patients with leukemia are at risk for osteonecrosis, which can occur at the hips, knees, and ankles. High-impact activities such as jumping rope should be avoided as an intervention to improve endurance since these types of activities place the patient at risk for further injury.

A PT is examining a patient in an outpatient clinic and notices swelling on the patient's right foot. Which of the following conditions can the PT suspect? A. Heart failure B. Lipedema C. Lymphedema D. Chronic venous insufficiency

C. Lymphedema Swelling is bilateral for heart failure so you can rule out A. Lipedema results in PROXIMAL swelling (feet do not swell) AND it's bilateral so you can rule out B. If it was chronic venous insufficiency they would describe a discoloration of the leg on top of the edema.

A 13-year-old patient reports moderate knee pain persisting more than 3 weeks, with no trauma noted. The patient exhibits an out-toeing gait pattern, leg length discrepancy, and restriction in medial (internal) rotation of the involved leg. Which of the following test findings would MOST likely be present? A. Pain with palpation of the trochanteric region B. Pain and instability during the application of valgus stress to the knee in full extension C. Pain in the groin region with hips flexed 80-90 degrees and then medially (internally) rotated with adduction D. Pain in the gluteal region with combined movements of hip flexion to 45-60 degrees, abduction, and lateral (external) rotation

C. Pain in the groin region with hips flexed 80-90 degrees and then medially (internally) rotated with adduction The stem describes a case of suspected slipped capital femoral epiphysis. Signs and symptoms are typically found in adolescent patients (10-16 years old) and include leg shortness, knee pain, and pain when the hip is medially (internally) rotated. Groin pain will be triggered with the anterior impingement test (hips flexed to 80° to 90° and medially [internally] rotated with adduction) if slipped capital femoral epiphysis exists.

A 58-year-old-male patient comes to the outpatient clinic for Phase II cardiac rehabilitation. On the initial evaluation, the patient mentions that he was taking propanolol, metformin, and vitamin supplements. The therapist wants the patient to use the stationary bike and make sure he maintains 60-70% of the maximum heart rate. What is the BEST way to measure the patient's exertion level? A. Heart rate monitor B. Radial pulse C. Rate of perceived exertion scale D. Systolic blood pressure

C. Rate of perceived exertion scale Propanolol is a beta blocker that would decrease the heart rate of the patient, so the heart rate monitor/pulse would not give an accurate measurement.

A 45-year-old man with lymphedema of the left leg is being examined by a PT. The PT determines that it is a stage 2 lymphedema and pitting scale grade is 3+. Which of the following statements is the MOST likely clinical presentation? A. Elevation of the foot reduces the swelling B. The indentation on finger pressure last for less than 15 seconds C. Skin is hardening due to fibrosis D. The indentation produced is less than 1/2 inch

C. Skin is hardening due to fibrosis Stage 2 (spontaneously irreversible): hard swelling present, nonpitting "brawny" edema; positive Stemmer sign; tissue is fibrosclerotic, proliferation of adipose tissue Grade 3+: severe, depression takes 15-30 seconds to rebound; 1/2-1 inch pitting A- stage I (reversible) B- 2+ D- 2+

A physical therapist is assessing a patient in a skilled nursing facility with RLE Lymphedema. The patient has a past medical history of cardiac arrhythmia. Objective assessment reveals fibrosis of the RLE, frequent infections, and a positive Stemmer's Sign. Which of the following is MOST likely true? A. The patient is in stage 4 lymphedema B. The patient has congestive heart failure C. The patient is in stage 2 lymphedema D. The patient is in stage 0 lymphedema

C. The patient is in stage 2 lymphedema The patient is in stage 2 lymphedema. Stage 2 lymphedema is spontaneously irreversible lymphedema and is characterized by a + Stemmers Sign and brawny edema. Patients in this stage have fibrosis.

You are treating an 80 year old patient s/p total hip arthroplasty posterior approach. The patient complains of 8/10 pain in the gluteal region. The patient's goal is to be able to get up from a low toilet at home. Which of the following is the MOST appropriate intervention? A. Squats to a low toilet to improve toilet transfers at home B. Gait training to promote safe ambulation between rooms at home C. Toilet transfers with a raised toilet seat to improve functional tolerance D. Resisted squats to improve glute strength and toilet transfers

C. Toilet transfers with a raised toilet seat to improve functional tolerance This patient had a posterior approach and has a goal of getting up from a low toilet. However, hip flexion greater than 90 deg, adduction and internal rotation is contraindicated with a posterior approach THA. Patients are advised to use a raised toilet seat so as to not break posterior hip precautions but promote functional independence.

During a postural screen, the therapist observes medial winging of the scapulae during a push up motion. The winging of the scapulae is MOSTLY due to: A. Weakness of upper trapezius B. Shortening of middle and lower trapezius C. Weakness of serratus anterior D. Shortening of rhomboids

C. Weakness of serratus anterior If the serratus anterior is weak or paralyzed, the winging of the scapula occurs on abduction and forward flexion (especially with a "punch out" forward against resistance). It causes the scapula to elevate and move medially with the inferior angle rotating medially.If the trapezius is weak or paralyzed, the winging of the scapula occurs before 90° abduction, and there is little winging on forward flexion. It causes the scapula to depress and move laterally with the inferior angle rotated laterally.

A 35-year-old male presents with a skin condition as shown in the picture below. There are irregular areas of localized skin edema. Which of the following is the MOST likely diagnosis for this patient? A. Blisters B. Vesicles C. Wheals D. Pustules

C. Wheals Blisters are filled with serous fluid. Vesicles are also filled with serous fluid but vesicles are smaller than blisters. Pustules are filled with pus.

In a cardiac rehab clinic a patient is exercising and the monitoring physical therapist notes that the patient has ventricular quadrigeminy. What should the initial response be? A. Its an emergency, call 911 B. Call the physician immediately C. Ask the patient to keep exercising D. Check the HR of the patient

D. Check the HR of the patient As a PT we should check the HR first. If the HR is below normal we will stop the exercise and if the HR is normal we should continue exercise at a low intensity. Ventricular quadrigeminy- every fourth beat is a premature ventricular contraction

A physical therapist treats a patient diagnosed with myasthenia gravis. Which of the following tests should the therapist MOST likely expect to be abnormal with this condition? A. Coordination testing B. Sensory testing C. Deep tendon reflex testing D. Endurance testing

D. Endurance testing Myasthenia gravis is an autoimmune disorder that disrupts neuromuscular transmission and results in muscle weakness and fatigability. A patient with myasthenia gravis could demonstrate normal strength with manual muscle testing since it only requires a single muscle contraction. However, the patient would demonstrate significant weakness if required to perform repeated contractions. Endurance testing would likely be abnormal for this patient.

A PT is assessing a patient with a chief complaint of low back pain and a diagnosis of L2-L3 nerve compression. Which of the following gait deviations at the hip and pelvis are NOT to be expected in this patient? A. Trunk lurches backward and toward the unaffected stance leg from heel off to mid swing B. Posterior tilt of the pelvis during initial swing C. Semicircle movement of the hip during swing- combining hip flexion, hip abduction, and forward rotation of the pelvis D. Excessive hip flexion at initial and mid swing

D. Excessive hip flexion at initial and mid swing L2-L3 nerve compression leads to weak hip flexors. Hence excessive hip flexion will not be seen at the initial and mid-swing phase of the gait cycle. Options A: Hip flexion is passively generated by a backward movement of the trunk Option B: Abdominals are used during initial swing to advance the swing leg Option C: Semicircular movement combining hip flexion, hip abduction, and forward rotation of the pelvis to compensate for hip flexor weakness.

All of the following are cardiovascular and respiratory effects of full immersion hydrotherapy (up to neck) EXCEPT: A. Increased cardiac volume B. Reduced heart rate and reduced systolic blood pressure C. Increased cardiac output D. Increased vital capacity

D. Increased vital capacity In full immersion hydrotherapy, vital capacity decreases as the lung expansion is inhibited due to hydrostatic pressure against the chest wall. SV and CO increase whereas HR remains same or slightly decreases.

A PT instructed a patient in home postural drainage positioning. The PT finds the patient sitting in a recliner, and leaning forward trying to perform drainage for the apical segments of upper lobes. What is the best way for the PT to correct the position? A. Change the position to prone lying on a bed with two pillows under the pelvis B. Keep sitting in the recliner, leaning forward over a folded pillow C. Change the position to supine on a bed with pillows under the knees D. Keep sitting in a recliner, leaning slightly backward

D. Keep sitting in a recliner, leaning slightly backward Leaning forward is for the posterior segment of the upper lobes, you would want to recline for the apical segment of the upper lobes

A 52-year-old male patient with chronic left sided low back pain due to limited left side bend and right side rotation restriction at L2-L3 level will MOST likely benefit from which of the following mobilization techniques? A. PA glide to the left L2 transverse process B. PA glide to the right L3 transverse process C. AP glide to the L3 spinous process D. PA glide to the right L2 transverse process

D. PA glide to the right L2 transverse process You never do AP glides so you can automatically rule out C. If the patient is having difficulty with side bending to the left and rotating the right he is having a closing restriction on the L side because he is unable to full close the facet joints. In order to help with closing you need to mobilize the bottom vertebrae L3 on the left side . However, the only option with L3 is on the right side. So, you have to rethink this. Maybe it is not an issue with closing on the left side but it is an issue with opening on the right side? Which leaves you with D.

A PT is assessing a patient with chief concerns of dizziness and loss of balance. Presence of which of the following findings during assessment will MOST likely lead the therapist to refer the patient to a physician? A. Mild ataxia B. Positional testing reproducing the nystagmus C. Vertigo suppressed by visual fixation D. Pendular nystagmus

D. Pendular nystagmus A-C are all symptoms of a peripheral vestibular pathology but pendular nystagmus is a symptom of CENTRAL vestibular pathology!

During a balance assessment of a patient with left hemiplegia, it is noted that in sitting the patient requires minimal assistance to maintain the position and cannot accept any additional challenge. What grade should the physical therapist use to document this patient's sitting balance? A. Normal B. Good C. Fair D. Poor

D. Poor A grade of poor is indicative of a person that is unable to maintain their balance in sitting without external support or assistance

A 68-year-old male patient comes to the clinic with a history of cognitive deficits and Alzheimer disease. The patient's chief complaint is urinary incontinence. Which of the following is the MOST appropriate intervention for this patient? A. Use of absorbent pads B. Pelvic floor exercises targeting levator ani muscles C. Breathing exercises to develop sphincter control D. Scheduled toileting and prompted voiding

D. Scheduled toileting and prompted voiding If they have a cognitive deficit they will not be able to follow exercises well so you can rule out B and C. It does not state the patient is PHYSICALLY impaired so you can assume they are able to get out of bed. If they are able to move you wouldn't want an absorbent pad for them. This is an example of functional incontinence (typically associated with cognitive deficits and Alzheimer's disease).

A 38-year-old male patient has been diagnosed with atherosclerosis by his cardiologist and wants to avoid primary risk factors. Which of the following factors is NOT considered to be a primary risk factor for atherosclerosis? A. High blood pressure B. Increased lipids C. Cigarette smoking D. Sedentary lifestyle

D. Sedentary lifestyle Sedentary lifestyle is a secondary risk factor. Secondary risk factors are more easily modified. You can fix your sedentary lifestyle. Primary: HBP, diabetes, high cholesterol, family history, gender, smoking

A patient is pregnant in her third-trimester and she would like advice on resting positioning from her PT. What is the most effective position for her to rest in at night? A. Supine B. Side-lying right C. Supine at a 20 degree angle D. Side-lying left

D. Side-lying left Lying on her right side will cause compression of internal organs, as well as the aorta, thereby reducing CO.

A patient walks on the lateral edge of the foot and demonstrates diminished heel off (terminal stance) during gait. Which of the following nerves is MOST likely compromised? A. Plantar nerve B. Posterior tibial nerve C. Deep fibular (peroneal) nerve D. Superficial fibular (peroneal) nerve

D. Superficial fibular (peroneal) nerve The patient's impaired ability to evert and plantar flex the foot at heel off (terminal stance) implicates the fibularis (peroneus) longus muscle, which is innervated by the superficial fibular (peroneal) nerve.

An otherwise healthy young adult who has a C5 spinal cord injury (ASIA Impairment Scale A) is being examined by a physical therapist. Which of the following functional levels is the MAXIMUM that the patient can potentially achieve? A. Dependent transfer to a wheelchair by using an overhead lift B. Independent rolling side to side in bed C. Independent community-level mobility with a manual wheelchair D. Toilet transfer with a sliding board and assistance

D. Toilet transfer with a sliding board and assistance A patient who has a C5 spinal cord injury (ASIA Impairment Scale A) can potentially assist with a sliding board transfer with the use of deltoids, biceps, and rhomboids, especially if normal strength is present in all innervated muscles.

Maximum lateral displacement occurs during which phase of walking GAIT? A. Mid stance B. Terminal swing C. Initial contact D. Pre swing

A. Mid stance Lateral displacement is the side-to-side movement of the pelvis during walking. It is necessary to center the weight of the body over the stance leg for balance. Maximum displacement is seen during the midstance.

Following a total knee arthroplasty, a patient has been receiving moist heat to the knee prior to exercise and gait training. During the current visit, the physical therapist notes new redness, swelling, and increasing warmth surrounding the knee. Which of the following actions should the therapist take? A. Notify the orthopedic surgeon of the changes B. Continue with the use of hot packs for 5-10 minutes prior to physical therapy C. Substitute ice instead of heat prior to physical therapy D. Discontinue physical therapy until the problems are resolved

A. Notify the orthopedic surgeon of the changes New symptoms of redness, swelling, and increasing warmth following a surgical procedure are indicators of a possible infection. The surgeon should be notified.

A patient presents with complaints of pain and numbness in the right arm extending to the thumb. The PT performs the special test as shown in the picture below. If the test is positive, which is the MOST appropriate intervention? A. Strengthen the right levator scapulae and sternocleidomastoid muscles B. Perform (posterior-anterior) PA glide of the C6 vertebrae with right side bending and rotation C. Stretch the right pectoralis minor and sternocleidomastoid muscles D. Perform grade 3 manipulation of the first and second ribs

C. Stretch the right pectoralis minor and sternocleidomastoid muscles This image is a picture of the military brace test or Eden's test for TOS. TOS calls for stretching of the SCM and pectoralis minor muscles! You are trying to RELEASE for TOS NOT STRENGTHEN. Other TOS tests: Roos, Adson

What is the best way to monitor the intensity of exercise for a patient limited mostly by claudication? A. Assess ankle-brachial index (ABI) during exercise B. Maintaining heart rate (HR) between 60-70% of age-predicted HR max during exercise C. Sustained pain levels of at least a 2 out of 4 on the claudication scale during exercise D. Upholding rate of perceived exertion (RPE) levels of 11-13 out of 20 during exercise

C. Sustaining pain levels of at least 2 out of 4 on the claudication scale during exercise

A patient presents with foot pain during running. The PT noted the patient has excessive foot pronation and would like to provide an orthotic that could help relieve pain. Which of the following orthotic would MOST LIKELY benefit this patient? A. Forefoot medial post under first metatarsal head B. Cushion heel C. Forefoot lateral post under fifth metatarsal head D. Posterior leaf spring

A. Forefoot medial post under first metatarsal head Stick to your plane so you can automatically rule out B and C. A would help to correct for the deviation by lifting the medial side of the foot up to neutral.

A patient who has decreased tactile sensation due to diabetes mellitus is being treated for a mild Achilles tendon strain. Which of the following interventions is MOST appropriate to restore normal alignment of the healing collagen? A. Gastrocnemius stretching B. Ultrasound to the gastrocnemius C. Cross-friction massage at the muscle belly D. Hot pack to the gastrocnemius

A. Gastrocnemius stretching Stretching is a stimulus in the early healing stages for the proper alignment of healing collagen. Gentle stretching is performed to provide a stimulus for fiber orientation without disruption of the immature collagen. ULTRASOUND AND HOT PACKS ARE NOT USED WITH PATIENTS WHO HAVE DIMINISHED SENSATION!!!

A patient with lower back pain presents to the clinic. The patient is experiencing radicular symptoms down the left leg, crossing the front of the knee, and their pain becomes worse with repeated extension. Based on these symptoms, what is the MOST likely cause? A. L4 nerve root irritation B. L5 nerve root irritation C. Spinal stenosis D. Piriformis syndrome

A. L4 nerve root irritation The location of the pain correlates with irritation of the L4 nerve root. Repeated extension would provoke irritation of a nerve root.

Lesions of the skin are the FIRST clinical sign of underlying disease for which of the following diagnoses? A. Scleroderma B. Thromboangiitis obliterans C. Anemia D. Hypothyroidism

A. Scleroderma Lesions in the skin are often the first sign of an underlying rheumatic disease. Scleroderma is accompanied by many skin changes.

You are assessing a patient with Parkinson's disease in the hospital. The patient complains of difficulty turning around and reports he has had 3 falls, with the most recent one occurring while getting up off the toilet at home. The patient is classified as a Hoehn and Yahn III. Which of the following is the MOST appropriate intervention for the initial treatment session? A. Sit to stand with emphasis on anterior weight shifting B. Standing on an even surface with eyes closed and feet together C. Dynamic balance on foam to improve balance reactions D. Gait training emphasizing starting and stopping

A. Sit to stand with emphasis on anterior weight shifting A Hoehn and Yahn stage III would include impaired righting reflexes, unsteadiness when turning or rising from a chair and the patient can live independently. The patient's most recent fall was rising from the toilet. An appropriate intervention would be to improve the patient's ability to sit to stand.

During the examination, the patient reports increased pain while standing. Pain is alleviated with lumbar flexion and sitting. Which special test will be positive to confirm the diagnosis shown in the picture? A. Stork standing test B. Gillet test C. Van Gelderen bicycle test D. Quadrant test

A. Stork standing test This image is spondylolisthesis. The stork standing test used for this condition. Gillet test- SI joint Van Gelderen bicycle test- stenosis Quadrant test- facet joint

A 39 year old comes to an outpatient clinic complaining of neck problems. Which of the following exercise combinations is MOST appropriate for a patient who has a forward head posture? A. Strengthen the deep cervical flexors and stretch the SCM's and upper cervical extensors B. Strengthen the deep cervical flexors and SCM's and stretch the upper cervical extensors C. Strengthen the cervical extensors and stretch the SCM's and deep cervical flexors D. Strengthen the cervical extensors and SCM's and stretch the deep cervical flexors

A. Strengthen the deep cervical flexors and stretch the SCM's and upper cervical extensors When your head is forward C3-C7 will flex and C1-C2 will be extended. Think of the upper cross syndrome for a forward head. The deep cervical flexors and lower trap/serratus anterior are weak. The SCM/pecs and upper traps/levator scapulae are tight.

A physical therapist evaluates the fall risk of a 73-year-old patient after being placed on a new medication. Which of the following medications would MOST likely contribute to an increased risk of falling? A. Nonopioid analgesics B. Benzodiazepines C. Thrombolytics D. Antidiarrheals

B. Benzodiazepines Benzodiazepines promote sleep through increasing inhibitory effects on the CNS synapses where GABA is found. Side effects include drowsiness, decreased motor performance, anterograde amnesia, and diminished alertness. Trade names include Xanax, Valium, and Klonopin.

The bed sore of a 72-year-old male patient has to be debrided. The wound has blunt edges with 40% slough and no maceration. Tunneling is absent. Which of the following will be the best to use before debridement? A. Low TENS B. Brief TENS C. High TENS D. Russian currents

B. Brief TENS Brief intense TENS is used for wound debridement. It is for a short amount of time (15 minutes) but it is intense so that they don't feel as much pain during debridement. It incorporates a high pulse frequency with long pulse duration. Low TENS= low frequency (accupuncture-like/motor) High TENS= high frequency (conventional/sensory)

A 76-year-old patient has been transferred to an acute care unit with symptoms of CHF. During auscultation, the PT finds S3 heart sound. Which of the following breath sounds is MOST likely associated with this finding? A. Stridor B. Crackles C. Wheeze D. Pleural rub

B. Crackles S3 is a ventricular gallop and is an abnormal heart sound. People with CHF often have pulmonary edema that comes with it. Pulmonary edema causes crackles in the lungs.

A patient is asked to complete a pain questionnaire. the patient selects words such as cramping, dull, and aching to describe the pain. What structure is MOST often associated with this type of pain description? A. Nerve root B. Muscle C. Bone D. Vascular

B. Muscle

The 54-year-old male patient's chart states that they have been taking beta-blockers for the past 5 years. Prior to starting an exercise training program, the patient should receive an explanation of the: A. Greater benefits from cardiovascular exercise to be achieved at lower SBP rather than at higher SBP levels B. Need to use measures other than heart rate to determine intensity of exercise C. Greater benefits from cardiovascular exercise to be achieved at lower HR than at higher HR levels D. Need for longer warm-up periods and cool-down periods during exercise sessions

B. Need to use measures other than heart rate to determine intensity of exercise Beta-blockers lower the HR during submaximal and maximal exercise so it is not representative of how hard they are working. You would use the RPE scale instead.

A physical therapist treats a patient with several injuries impacting the upper extremity including mallet finger. The therapist notes that the affected finger is immobilized using a static splint. Which position of the finger would be the MOST essential when splinting? A. 5 degrees of flexion at the distal interphalangeal joint B. 5 degrees of flexion at the proximal interphalangeal joint C. 5 degrees of hyperextension at the distal interphalangeal joint D. 5 degrees of hyperextension at the proximal interphalangeal joint

C. 5 degrees of hyperextension at the distal interphalangeal joint Splinting in 5 degrees of hyperextension allows the extensor tendon to heal in an optima position without becoming excessively lengthened. The amount of hyperextension should be very slight since too much can cause ischemia to the skin. Splinting may be required for as long as 6-8 weeks.

A new born is examined at birth using the APGAR screening test. The following scores are reported at 5 min: heart rate (2), respiration (2), reflex irritability (1), muscle tone (2), and color (2). The PT should: A. Monitor the APGAR score again at 10 minutes B. Expect respiratory complications due to low score on respiration C. Expect good heart rate, good respiration, and normal reflex responses D. Expect neurological complications due to low score on reflex irritability

C. Expect good heart rate, good respiration, and normal reflex responses A 9/10 APGAR is considered normal and would not require any further assessment. If there is a 7/10 or more there is no need for a follow-up!!! If there is a score of "0" THEN you would suspect complications. The APGAR score is recorded at 1 and 5 minutes after birth. You only test at 10 minutes if the score is less than 7 or there is a "0". 2- perfect 1- normal 0- concerning

A patient post CVA ambulates with a large base quad cane. The patient presents with left neglect and diminished proprioception. Which of the following actions is the MOST appropriate to ensure patient safety? A. Provide continuous verbal cues B. Utilize visual cues and demonstration C. Offer manual assistance on the left side D. Offer manual assistance on the right side

C. Offer manual assistance on the left side While verbal cues will be beneficial, they alone will not help the patient ambulate and reduce the risk of falling. The PT should offer manual assistance on the patient's left side during ambulation activities. The manual assistance can facilitate motor activity and weight bearing, as well as proprioception on the affected side. Manual contact significantly reduces the risk for fall or injury.

When examining a patient's pressure injury, a physical therapist notes that in the area of the wound, the patient has complete loss of skin and intact underlying fascia. The therapist should recognize this as a: A. Stage 1 wound B. Stage 2 wound C. Stage 3 wound D. Stage 4 wound

C. Stage 3 wound Stage 1: nonblanchable erethyma Stage 2: partial thickness; epidermis and part of dermis Stage 3: full-thickness; dermis and epidermis. May extend down to but not through underlying fascia. Stage 4: full-thickness down to the bone

A child who has athetoid cerebral palsy is MOST likely to exhibit which of the following characteristics? A. Sustained limb posturing B. Low frequency tremor C. Rapid, jerky motions D. Mixed muscle tone

D. Mixed muscle tone Athetoid cerebral palsy is characterized by slow, involuntary, writhing, twisting, "wormlike" movements. Some muscles demonstrate tone that is too high, and others demonstrate tone that is too low. Sustained limb posturing- dystonia Low frequency tremor- cerebellar involvement or PD Rapid, jerky motions- chorea

In the acute care setting, a patient who has which of the following conditions is MOST likely to receive physical therapy intervention? A. Dissecting aortic aneurysm B. Decompensated chronic heart failure C. Third-degree heart block with a rate of 56 bpm D. Chronic atrial fibrillation with a rate of 96 bpm

D. Chronic atrial fibrillation with a rate of 96 bpm Atrial fibrillation alone is not an absolute indication that the patient is unstable and that treatment should be withheld. New, onset atrial fibrillation with a rapid ventricular rate (greater than 100 bpm) is an indication that physical therapy services should be withheld.

A patient who sustained a superficial abrasion and a fracture to the left thumb (1st digit) 2 months ago reports constant pain over the thumb and medial hand. The thumb is red, swollen, and hypersensitive. There is excessive sweating over the medial hand. Which of the following conditions is MOST likely present? A. Infection B. Arthrofibrosis C. Osteoarthritis D. Complex regional pain syndrome

D. Complex regional pain syndrome Complex regional pain syndrome is characterized by an exaggerated response to injury in a limb, with intense prolonged pain, vasomotor disturbance, delayed functional recovery, and trophic changes. Symptoms described in the stem should alert the clinician to the possibility of complex regional pain syndrome.

A physical therapist is planning to start their private PT practice and is working on making the clinic wheelchair accessible. Which of the following is NOT appropriate for a wheelchair accessible plan? A. Ramp slope: 1:12 B. Ramp width: 36 inches C. Tile or hardwood floors D. Door width should be 22 inches

D. Door width should be 22 inches The door width needs to be a MINIMUM of 32 inches with 36 inches being preferred Other important measurements: Need at least 36 inches to make a 90 degree turn Need at least 60 inches to turn 180 degrees

A therapist is assessing cranial nerve integrity in a patient. On shining light in the patient's right eye none of the pupils constrict, however on shining the light in the patient's left eye both pupils constrict. Which of the following is MOST appropriate about this finding? A. Left oculomotor nerve is affected B. Right oculomotor nerve is affected C. Left optic nerve is affected D. Right optic nerve is affected

D. Right optic nerve is affected There is a problem with the sensory aspect (both eyes are impacted) so you can automatically rule out A and B because the oculomotor nerve is involved with MOTOR. The pupils should constrict on both sides when light is shined in one eye. So, if the light is shined in the right eye and both eyes do not constrict but they constrict normally when you shine light in the left eye it is safe to assume the RIGHT optic nerve is affected. A- left side would stay dilated and right side would constrict B- right side would stay dilated and left would constrict C- when the light is shone in the left eye and BOTH eyes do not respond D- when light is shone in the right eye and BOTH eyes do not respond

A PT is considering implementing electrical stimulation in the patient's POC to promote healing on an infected wound. Which of the following is the BEST technique with regards to pad placement and polarity? A. Pad should be placed first on wound bed followed by placing saline-soaked gauze with negative polarity B. Saline-soaked gauze placed first on wound bed followed by placing pad with positive polarity C. Pad should be placed first on wound bed followed by placing saline-soaked gauze with positive polarity D. Saline-soaked gauze placed first on wound bed followed by placing pad with negative polarity

D. Saline-soaked gauze placed first on wound bed followed by placing pad with negative polarity If there is an infection that is NEGATIVE (you obviously don't want an infection) so you would want a negative polarity. Always place the saline-soaked gauze FIRST and then the pad.

A 52-year-old female has a history of multiple sclerosis and described that the symptoms initially would appear for a few days and disappear but have no become continuous and do not disappear. This patient MOST likely has which of the following types of MS: A. Secondary-progressive B. Primary-progressive C. Progressive-relapsing D. Relapsing-remitting

A. Secondary-Progressive Secondary-progressive begins as relapsing-remitting but the body essentially gets tired and it turns into a steady decline with periods of remission.

A patient who has a transfemoral amputation should perform which of the following movement patterns when being taught to pick up an object from the floor while wearing a prosthesis? A. Place both lower extremities at the same level and weight-bear through both extremities while reaching for the object B. Place the affected lower extremity forward and weight-bear through the affected lower extremity while bending forward to reach for the object C. Place the affected lower extremity forward and weight-bear through the unaffected lower extremity while bending forward to reach for the object D. Place the unaffected lower extremity forward and weight-bear through the unaffected lower extremity while bending forward to reach for the object

D. Place the unaffected lower extremity forward and weight-bear through the unaffected lower extremity while bending forward to reach for the object The correct way of picking up an object from the floor is to put the unaffected extremity forward and put body weight on the unaffected extremity while bending and reaching for the object on the floor.

A PT is testing the active shoulder ROM of a 45-year-old female patient. The PT asks the patient to move the shoulder to full medial/internal rotation. During medial rotation at the shoulder joint (GH joint), the humerus will slide: A. Anteriorly B. Superiorly C. Inferiorly D. Posteriorly

D. Posteriorly For medial rotation you will roll anteriorly. This is a convex on concave movement so slide is opposite and in the posterior direction.

Which of the following factors BEST predicts improvements in upper extremity functional outcomes for a patient following a cerebrovascular accident? A. Family involvement in the patient's care B. Use of blocked practice during intervention C. Patient's cognitive understanding of the impairments and intervention program D. Presence of active wrist and finger extension

D. Presence of active wrist and finger extension Research evidence supports that patients who possess active wrist and finger extension have improved upper extremity functional outcomes after a cerebrovascular accident.

A 55-year-old patient presents with a history of gradual onset of diffuse back pain that is not altered by change of position. The pain is partially relieved by nonprescription analgesics. Assessment reveals no weakness or lack of sensation in the lower extremities and no significant limitation in range of motion. The physical therapist's MOST appropriate action would be to: A. Evaluate for chronic pain syndromes B. Initiate a trial of pool therapy to reduce stress on the spine C. Initiate a trial of transcutaneous electrical nerve stimulation D. Refer the patient for a medical evaluation

D. Refer the patient for a medical evaluation Pain that is unchanged by position along with a lack of significant objective musculoskeletal findings may indicate systemic disease; therefore, the patient may require medical evaluation

Self selected speed is measured on a continuous scale in 2 groups of professional and amateur individuals. The data shows markedly skewed distribution. Which of the tests is MOST appropriate to find significant difference in gait speed between two groups? A. Mann-Whitney U test B. Analysis of variance C. T-test D. Kruskal-Wallis test

A. Mann-Whitney U test There is a skewed distribution so you can automatically rule out the parametric tests: t-test and ANOVA. Kruskal-Wallis is used with 3 or more groups so that can also be ruled out.

A PT in treating a patient with a diagnosis of lower cervical level spinal cord injury. While the PT was performing passive hip stretching, the PT noticed that the patient had started sweating profusely and had goosebumps. On checking the vitals, the patient's HR had dropped and BP was higher than normal. Which of the following is the MOST APPROPRIATE intervention for this patient? A. Bring the patient to a sitting position and remove the abdominal binder B. Keep the patient in the supine position and start CPR C. Bring the patient to sitting position and clamp the catheter D. Stop the treatment and let the patient rest for at least 2 hours before starting the treatment again

A. Bring the patient to sitting position and remove the abdominal binder Patients with a T6 or higher level of injury are at a risk of Autonomic Dysreflexia (AD) in the presence of a noxious stimuli. The symptoms of AD include profuse sweating, increased spasticity, hypertension, bradycardia, headache, piloerection, blurred vision, etc. The onset of these symptoms should be treated as a medical emergency and in order to lower the BP, the patient must be brought to a sitting position, noxious stimuli, tight clothing and abdominal binder must be removed, and the catheter must be unclamped.

A patient complains of left hip pain for the past two months. During gait analysis, the therapist notices that the patient's left LE is more internally rotated as compared to the right LE. Which of the following is the MOST LIKELY cause of this gait deviation? A. Excessive internal rotation of left pelvis B. Excessive internal rotation of right pelvis C. Spasticity of external rotators of left lower extremity D. Spasticity of the adductor muscles of right lower extremity

A. Excessive internal rotation of left pelvis Spasticity or contractures always affect the opposite side of the body whereas weakness affects the same side. So, with this since the deficit is noted on the left spasticity must be on the right so you can rule out C. D can also be ruled out because we are focusing on the hip rotators NOT adductors. Spasticity in the hip adductors would cause a left hip drop not rotation. If there is an internal rotation of the leg you can assume that the pelvis is probably rotating with the femur.

A PT is evaluating a 26-year-old female basketball player with a vague diagnosis of right knee pain. The patient is performing a jump landing as shown. The PT treatment should MOST likely focus on: A. Strengthening of the right gluteus medius B. Strengthening of the right adductor magnus C. Strengthening of the right vastus medialis D. Strengthening of the right tibialis anterior

A. Strengthening of the right gluteus medius Since the patient is in genu valgum their hips are currently adducted so you need to strengthen an abductor to help position the hip in a neutral position when landing and control the pelvis. Note: VMO used to be considered correct but more recent research shows strengthening the more proximal muscles is correct

A physical therapist is treating a patient with presence of geotrophic nystagmus on head turns in the supine position. The nystagmus was weaker on the right side. Which of the following is the MOST APPROPRIATE sequence of administering the intervention for these symptoms? A. Canalith repositioning maneuver with head in 20 deg of extension, head turned to 90 deg to right, then moved to the left, then get the patient in prone position with neck in flexion, and finally sit the patient up B. Canalith repositioning maneuver with head in 20 degrees of flexion, head turned to 90 deg to left, then moved to the right, then get the patient in prone position with neck in flexion, and finally sit the patient up C. Canalith repositioning maneuver with head in 20 deg ext, head turned to 90 deg to left, then moved to the right, then get the patient in prone position with neck in flexion, and finally sit the patient up D. Canalith repositioning maneuver with head in 20 deg flexion, head turned to 90 deg to right, then moved to the left, then get the patient in prone position with neck in flexion and finally sit the patient up

B. Canalith repositioning maneuver with head in 20 degrees of flexion, head turned to 90 deg to left, then moved to the right, then get the patient in prone position with neck in flexion, and finally sit the patient up If a patient has geotrophic nystagmus that means they have horizontal BPPV that is going towards the ground. Geotrophic nystagmus is canalisthiasis and is named for the STRONGER side. So, this would be left horizontal BPPV with canalisthiasis. Horizontal BPPV is treated with the BBQ roll. To treat the left side the head is initially turned to the left in 20 degrees of FLEXION.

A therapist is observing a patient with a transtibial prosthesis and notices "drop off" during the late stance phase of gait. Which of the following prosthetic/anatomical causes would be the LEAST LIKELY cause of this gait deviation? A. Keel too short B. Insufficient plantarflexion C. Socket too far posterior D. Knee flexion contracture

C. Socket too far posterior An early knee flexion during late stance phase of gait is also known as "drop off". Prosthetic causes: high shoe heel, insufficient plantarflexion, keel too short, DF stop too soft, socket too far anterior, socket excessively flexed, cuff tabs too posterior Anatomical causes: knee flexion contracture A socket placed too far posterior would cause delayed knee flexion

A physical therapist treats a patient who has a stage 3 pressure injury over the left ischial tuberosity. The patient's pressure injury has healed over time and currently shows the characteristics of a stage 1 pressure injury. What pressure injury stage should the therapist assign to the wound? A. Stage 1 pressure injury B. Modified stage 1 pressure injury C. Stage 3 pressure injury D. Modified stage 3 pressure injury

C. Stage 3 pressure injury A pressure injury's stage is NEVER changed. It is always classified as the initial classification.

All of the following are absolute contraindications for exercise during pregnancy EXCEPT: A. Restrictive lung disease B. Preeclampsia C. Severe anemia D. Chronic bronchitis

D. Chronic bronchitis The woman with chronic bronchitis may participate in an exercise program under close observation by a physician and a therapist as long as no further complications arise. Exercises often require modification which should be discussed with the referring practitioner. Restrictive lung disease, preeclampsia and severe anemia are absolute contraindications to exercise during pregnancy.

A PT instructed a patient in home postural drainage positioning. The PT visits the home and finds the patient trying to perform drainage for the posterior segments of the upper lobe in supine. What is the best way for the PT to correct the position? A. Change the position to prone lying on a bed with two pillows under the pelvis B. Change the position to sitting on a chair, leaning forward over a folded pillow C. Keep the position to supine on a bed with pillows under the knees D. Change the position to sitting in a recliner, leaning slightly backward

B. Change the position to sitting on a chair, leaning forward over folded pillow Option C is the same as they are currently doing so you can rule that out. A- posterior lower lobes B- posterior upper lobes C- anterior upper lobes D- apical segment of upper lobes

A 45-year-old female patient with chronic low back pain is visiting an outpatient clinic for her first treatment visit. The PT begins training the stability muscles. Which is the FIRST exercise taught to the patient for stabilization training? A. Begin training with awareness of safe spinal motions and the neutral spine position B. Begin training aggressive pelvic tilt exercises C. Begin training lumbar stabilization exercises sitting on an exercise ball with extremity motion and flexed spine position D. Begin training supine single leg lifts training the core

A. Begin training with awareness of safe spinal motions and the neutral spine position

Which of the following findings is MOST commonly associated with patients who have chronic obstructive pulmonary disease? A. Below normal diaphragmatic excursion of 0.4 to 0.8 inches (1 to 2 cm) B. Above normal diaphragmatic excursion of 0.4 to 0.8 inches (1 to 2 cm) C. Below normal diaphragmatic excursion of 1.2 to 2 inches (3 to 5 cm) D. Above normal diaphragmatic excursion of 1.2 to 2 inches (3 to 5 cm)

A. Below normal diaphragmatic excursion of 0.4 to 0.8 inches (1 to 2 cm) Excursion is decreased in patients who have chronic obstructive pulmonary disease due to hyperinflation of the chest and a resultant flattened diaphragm. Normal excursion of the diaphragm is 1.2 to 2 inches (3 to 5 cm); therefore, 0.4 to 0.8 inch (1 to 2 cm) would be below the normal excursion value.

A 45 year old female is referred to physical therapy with rheumatoid arthritis. The physical therapist notes increased flexion at the proximal interphalangeal joints and hyperextension at the distal interphalangeal joints. The MOST likely diagnosis will be: A. Boutonniere deformity B. Mallet finger C. Swan neck deformity D. Ulnar drift

A. Boutonniere deformity Boutonniere Deformity: Rupture of the central band (central slip) of the extensor hood results in the lateral bands of the extensor apparatus (extensor hood) slipping in a volar direction to the PIP joint, causing PIP flexion and DIP extension.Swan Neck Deformity: Laxity of the PIP joint with an over- stretched palmar plate and bowstringing of the lateral bands of the extensor hood result in hyperextension of the PIP and flexion of the DIP joints.Ulnar drift: Ulnar deviation of the digits because of weakening of the capsuloligamentous structures of the metacarpophalangeal joints and the accompanying "bowstring" effect of the extensor communis tendons.Mallet finger: It is the result of a rupture or avulsion of the extensor tendon where it inserts into the distal phalanx of the finger. The distal phalanx rests in a flexed position.

A 76-year-old patient suffered a recent stroke. PT determined that the patient has apraxia, and she is unable to drink from the bottle on command. However, she can point out the bottle and verbalize the purpose of the bottle. From this information, what sort of apraxia does the patient have? How should the PT approach treatment? A. Ideomotor apraxia; The PT should speak in short concise sentences B. Ideational apraxia; The PT should always give the patient 3-step commands C. Ideomotor apraxia; The PT should always give the patient 3-step commands D. Ideational apraxia; The PT should speak in short concise sentences

A. Ideomotor apraxia; The PT should speak in short concise sentences The person knows what the bottle is but does not know how to drink from it. You don't want to provide the patient with several commands because that will make it more complicated for them. Ideomotor apraxia: inability to carry out, on command, learned motor acts despite adequate motor and sensory abilities

A physical therapist attempts to quantify the patient's endurance level by administering a maximal exercise test. What is the PRIMARY limitation of a maximal exercise test? A. It requires participants to exercise to the point of volitional fatigue B. It does not typically allow a steady state heart rate at each work rate C. It is not useful in diagnosing coronary artery disease D. It requires progressive stages of increasing work intensities without rest intervals

A. It requires participants to exercise to the point of volitional fatigue D is true but this concept applies to most standard exercise tests not just maximal exercise tests. The primary limitation of a maximal exercise test is that it requires the individual to exercise to the point of volitional fatigue. Some subjects terminate the exercise test due to fatigue or exercise intolerance before reaching their physiological maximum. This reduces the sensitivity of the estimate of maximum oxygen intake.

A patient complains of frequent episodes of tingling on the medial aspect of the calf and ankle especially on forward flexion. On evaluation, the knee jerk is found to be weak along with weakness of tibialis anterior and extensor hallucis muscles and a limited straight leg raise with negative prone knee bend. A disc herniation at which of the following levels would cause these symptoms? A. L3-L4 B. L2-L3 C. L4-L5 D. L5-S1

A. L3-L4 L4 lesions would be associated with weakness of tibialis anterior, extensor hallucis along with paresthesias in the medial aspect of calf and ankle, weak knee jerk and a limited straight leg raise. L4 nerve root is affected in this case. At the lumbar level between L3-L4: Nerve root exiting- L3 Nerve affected in stenosis- L3 But with herniation, it is the L4 nerve root that is involved. Because of the course of the nerve root as it exits, the L4 disc (between L4 and L5) only rarely compresses the L4 nerve root; it is more likely to compress the L5 nerve root. In this case the symptoms aggravated on forward bending- indicating it was a disc pathology. So for L4 to be affected, the herniation should have taken place at L3-L4 level.

A physical therapist assesses a patient who has significant biceps brachii weakness and suspects there may be underlying neurological damage that is causing the weakness. Damage to what cord of the brachial plexus would MOST likely explain this finding? A. Lateral B. Posterior C. Medial D. Anterior

A. Lateral The lateral cord branches off to become the musculocutaneous nerve, median nerve, and lateral pectoral nerve. The musculocutaneous nerve innervate the muscles of the anterior arm, including the biceps brachii.

A patient with right hemisphere damage would MOST likely have: A. Left unilateral neglect and difficulty reaching for objects presented on the left side of the body B. Left unilateral neglect and difficulty reaching for objects presented in the midline of the body C. Difficulty crossing midline to reach objects on the right side of the body D. Difficulty stabilizing gaze and locating objects on the right side of the body

A. Left unilateral neglect and difficulty reaching for objects presented on the left side of the body Damage to the right hemisphere will affect the LEFT side of the body.

A 14 year old girl presents with a chief complaint of pain in the medial three fingers of the dominant hand, especially during gripping activities. The pain lasts for a while and subsides on its own after resting. She likes to play tennis on weekends but has a sedentary lifestyle otherwise. During postural assessment, the therapist noticed significant winging of the scapula on the same side. Strength of shoulder flexors: 4/5 and abductors is 4+/5 on the affected side. Strength of intrinsic muscles is intact. Which of the following nerves is MOST LIKELY involved? A. Long thoracic nerve B. Axillary nerve C. Median nerve D. Suprascapular nerve

A. Long thoracic nerve Presence of scapular winging is indicative of serratus anterior weakness. The long thoracic nerve (C5-C7) is a motor nerve that supplies the serratus anterior muscle. The long thoracic nerve can be injured due to carrying heavy loads on the shoulder (e.g. backpacks) or when serving while playing tennis. Due to the weakness of scapular muscles, shoulder elevation may also be affected. Long thoracic nerve injury can be associated with inability/pain when fully flexing an extended arm.

A patient with subluxation of the shoulder presents to the PT clinic. During evaluation, the therapist notices atrophy of the flexor surface of the upper arm along with weakness of elbow flexion with a supinated forearm. Which of the following findings will MOST LIKELY be associated with this finding? A. Loss of 2-point discrimination on the lateral aspect of the forearm B. Loss of 2-point discrimination on the posterior aspect of the forearm C. Loss of 2-point discrimination on the lateral aspect of the upper arm D. Loss of 2-point discrimination on the posterior aspect of the arm

A. Loss of 2-point discrimination on the lateral aspect of the forearm If you think about a muscle that flexes the elbow while supinated you would think of the biceps. The musculocutaneous nerve innervates the biceps brachii. The musculocutaneous nerve's sensory supply is to the lateral aspect of the forearm

Which of the following motions will close the right lumbar facet joints from a neutral position? A. Lumbar extension, right side flexion, left rotation B. Lumbar extension, left side flexion, right rotation C. Lumbar flexion, right side flexion, left rotation D. Lumbar flexion, left side flexion, right rotation

A. Lumbar extension, right side flexion, left rotation To close the lumbar spine facet joints you always begin by extending the lumbar spine (closes the gap, "bottoms up"). You then flex to the same side but you can only go so far before you have to rotate the opposite direction to completely close the facet joint. (keep in mind rotation and side-bending are to the SAME side for C-spine and T-spine but OPPOSITE for L-spine)

A patient who has multiple sclerosis reports increasing problems related to urinary incontinence. The patient experiences constant leaking of small amounts of urine and has a sensation of the bladder not being fully emptied after voiding. The patient MOST likely has which of the following conditions? A. Overflow incontinence caused by an underactive detrusor muscle B. Stress incontinence caused by anxiety about the disease C. Urge incontinence caused by detrusor muscle spasms D. Functional incontinence due to mobility changes

A. Overflow incontinence caused by an underactive detrusor muscle The symptoms fit the description for overflow incontinence. In patients who have multiple sclerosis, overflow incontinence is usually the result of a hypotonic or underactive detrusor muscle. Overflow incontinence: occurs when you are unable to completely empty your bladder; this leads to overflow, which leaks out unexpectedly.

A patient is instructed to perform the exercise shown in the picture (standing quad stretch) at home, however the patient is unable to complete the movement. The patient's lower extremity limitation is MOST likely caused by which of the following conditions? A. Passive insufficiency of the quadriceps B. Active insufficiency of the quadriceps C. Active insufficiency of the hamstrings D. Passive insufficiency of the iliopsoas

A. Passive insufficiency of the quadriceps This position is both shortening the hamstrings and lengthening the quads so you can rule out B and D. Both A and C are plausible. The reason it is A is because you are not contracting the hamstrings to get to this point so it is not an active motion. It is a passive movement that is focusing on stretching the quadriceps so it is "passive" insufficiency.

A 72-year-old female, who spends most of her time indoors, visited the beach last weekend. She reports she kept losing her balance while walking on the beach irrespective of whether the beach was crowded or not. She does not report any falls or episodes of loss of balance at home during the day or night. Which of the following is the MOST appropriate conclusion based on this scenario? A. Patient is dependent on the somatosensory system B. Patient is dependent on the visual system C. Somatosensory system is affected D. All the systems are affected

A. Patient is dependent on the somatosensory system The patient has vision but her somatosensory is absent (sand) which is condition 4 of the CTSIB. If vision is Available and the patient falls the vision is Affected. BUT if somatosensory is NOT available and she falls she may be dependent on somatosensory. Dependency questions often have a comparison (she was fine at home but now that she's at the beach she is struggling). When the somatosensory system is available she doesn't fall but when it isn't available she falls. TO BE AFFECTED IT HAS TO BE AVAILABLE IF A SYSTEM IS NOT AVAILABLE AND A PATIENT FALLS THEN THEY MAY BE DEPENDENT ON THAT SYSTEM

During shoulder flexion, the physical therapist observes that the scapulae is moving abnormally on the right side. To check the proper functioning of the lower trapezius, a physical therapist resists which of the following sets of movements? A. Scapular depression, adduction, and upward rotation B. Scapular depression, abduction, and downward rotation C. Scapular elevation, abduction, and upward rotation D. Scapular elevation, adduction, and downward rotation

A. Scapular depression, adduction, and upward rotation The lower trapezius muscle acts as a scapular depressor, upward rotator (along with upper trapezius) and adductor (retractor) of the scapula. So, these movements can be resisted to check the proper functioning of this muscle. *Remember...rhomboiDs are related to downward rotation. The upper and lower traps and serratus anterior are upward rotation.

A patient admitted to an acute care hospital is not able to attend a scheduled physical therapy session due to suspected critical limb ischemia. Which of the following symptoms is MOST commonly associated with this condition? A. Severe pain in the legs and feet at rest B. Increased temperature in the lower leg and foot C. Bounding lower extremity peripheral pulses D. Flaking skin on the legs with brownish discoloration

A. Severe pain in the legs and feet at rest Critical limb ischemia is characterized by a DECREASE in temperature so you can rule out B. It is also characterized by a DIMINISHED peripheral pulse so you can also rule out C. Flaking skin on the legs with brownish discoloration is a symptom of venous insufficiency not ischemia. Critical limb ischemia is characterized by severe pain in the legs and feet at rest. A patient often experiences the pain when in bed and may be able to diminish the intensity of the pain by hanging the legs over the edge of the bed or getting up to walk around.

A 66 yr old patient was admitted to ICU following a cerebrovascular accident. The physician orders PT services to address position recommendations when lying on the hemiplegic side. For the protection of shoulder, the MOST appropriate position to place the patient's shoulder in is: A. Shoulder forward, slight abduction, and external rotation B. Shoulder neutral, slight abduction only C. Shoulder forward, slight abduction, and internal rotation D. Shoulder neutral, slight abduction, and external rotation

A. Shoulder forward, slight abduction, and external rotation Effective positioning of the hemiparetic extremities encourages proper joint alignment while positioning the limbs out of the abnormal postures typically assumed. Positioning strategy in side lying on the more affected side - Scapula protracted; shoulder forward; arm placed in slight abduction and external rotation; elbow extended, forearm supinated, wrist neutral, fingers extended, and thumb abducted.

A patient presents to the clinic with chief complaints of left sided neck pain that started about 2 weeks ago. After assessing, the PT concludes that the patient has a closing restriction at C5-C6 level on the left. Which of the following is MOST appropriate about this patient? A. Side bending and rotation to the left is restricted at C5-C6 level B. Side bending and rotation to the right is restricted at C5-C6 level C. Side bending is restricted to the right and rotation is restricted to the left at C5-C6 level D. Side bending is restricted to the left and rotation is restricted to the right at C5-C6 level

A. Side bending and rotation to the left is restricted at C5-C6 level Side bending and rotation are in the same direction for the C-spine so you can automatically rule out C and D. If you side bend and rotate to the left you are closing off that left facet joint. If you are having closing restrictions you will have difficulty closing and therefore you will be restricted going in that direction.

A PT was assessing a patient's balance response to perturbations. The perturbation given was a slow-paced, small amplitude posterior perturbation. The patient showed a normal balance response and was able to maintain stability. For a normal balance response, what would be the order of muscle activation? A. Tibialis anterior, quadriceps, abdominals B. Paraspinals, hamstrings, gastrocnemius C. Gastrocnemius, hamstrings, paraspinals D. Abdominals, quadriceps, tibialis anterior

A. Tibialis anterior, quadriceps, abdominals If it a slow-paced, small amplitude perturbation you can assume the patient used an ankle strategy. With an ankle strategy, the muscles fire from distal to proximal and on the opposite side of the body. So for a posterior perturbation you would have a posterior sway and the muscles on the anterior side of the body will fire.

A patient presents to the clinic with history of burns on the left hip and thigh region causing tight hip flexors. What gait abnormality is MOST likely demonstrated by the patient? A. A shorter step length with the left lower extremity B. A shorter step length with the right lower extremity C. Backward lean during stance phase on left lower extremity D. Lateral lean during swing phase on right lower extremity

B. A shorter step length with the right lower extremity Stick to your plane so you can immediately rule out D. If the flexors are tight it will not shorten your step length because you are going into hip flexion. Tight hip flexors causes decreased extension so when you take a step with the RIGHT leg you are extending your left hip which will be inhibited.

The exercise shown in the photograph is MOST likely to be prescribed for a patient who has which of the following findings? A. Decreased strength of the flexor carpi ulnaris B. Decreased strength of the extensor carpi ulnaris C. Diminished sensation in the lateral deltoid region D. Diminished sensation in the palm and thumb, index finger, and middle finger (1st, 2nd, and 3rd digits)

B. Decreased strength of the extensor carpi ulnaris The intervention in the photograph is a radial nerve stretch. A possible finding with a radial nerve mobility issue is decreased strength of the extensor carpi ulnaris muscle, which is innervated by this nerve.

An entry in the medical record indicates that electromyography revealed denervation of the flexor pollicis longus, flexor digitorum profundus, and pronator quadratus muscles. This finding would MOST likely be associated with which of the following conditions? A. Anterior compartment syndrome B. Anterior interosseous syndrome C. Cubital tunnel syndrome D. Erb's palsy

B. Anterior interosseous syndrome Anterior interosseus syndrome is characterized by an injury to the anterior interosseous nerve, a branch of the median nerve which is sometimes pinched or entrapped as it passes between the two heads of the pronator teres muscle. This leads to pain and functional impairment of the flexor pollicis longus, the lateral half of the flexor digitorum profundus, and the pronator quadratus muscles.

A physical therapist designs a research study that will examine the effect of high voltage galvanic electrical stimulation on edema following arthroscopic knee surgery. Which of the following methods is the MOST appropriate to collect the necessary data? A. Anthropometric measurements B. Circumferential measurements C. Goniometric measurements D. Volumetric measurements

B. Circumferential measurements Circumferential measurements allow the physical therapists to obtain a gross estimate of edema in the knee. Volumetric measurements are used to check for edema in the WRIST and HAND.

A physical therapist examines a patient diagnosed with cerebral palsy. The therapist has limited experience treating patients who have cerebral palsy and is concerned about their ability to provide appropriate treatment. Which of the following actions is the MOST appropriate for the therapist to utilize? A. Inform the patient of your area of expertise B. Co-treat the patient with another more experienced therapist C. Treat the patient D. Refuse to treat the patient

B. Co-treat the patient with another more experienced therapist By co-treating the patient, the therapist receives external assistance and at the same time improves their skills with a particular patient population.

A physical therapist examines a patient who has limited cervical range of motion. As part of the examination, the therapist attempts to screen the patient for possible vertebral artery involvement, but is unable to position the patient's head and neck in the recommended test position. Which of the following actions is the MOST appropriate for the therapist to take? A. Complete the vertebral artery test with the head and neck positioned in approximately 50% of the available cervical range of motion B. Complete the vertebral artery test as far into the available cervical range of motion as tolerated C. Avoid completing the vertebral artery test until the patient has full cervical range of motion D. Avoid all direct cervical treatment techniques until the vertebral artery test can be assessed at the limits of normal cervical range of motion

B. Complete the vertebral artery test as far into the available cervical range of motion as tolerated The PT should perform the test and clear the patient's vertebral artery for their available range of motion. As the patient gains additional range of motion the test can be readministered.

A patient who has chronic obstructive pulmonary disease is participating in a mild graded exercise program at a level of 2 metabolic equivalents (METs). The patient's heart rate at rest is 80 bpm. During an incremental increase in exercise up to 4 metabolic equivalents (METs), the patient experiences an elevation in heart rate to 120 bpm. Which of the following actions is MOST appropriate? A. Discontinue exercise while monitoring vital signs B. Continue the exercise session while monitoring vital signs C. Continue the exercise session and refer the patient to a physician to evaluate exercise response D. Discontinue the exercise session and refer the patient to an emergency department to evaluate exercise response

B. Continue the exercise session while monitoring vital signs During acute exercise, patients who have chronic obstructive pulmonary disease experience elevated heart rates and blood pressures with incremental exercise. The patient's response is an expected compensatory response, and exercise can continue as prescribed with continued monitoring of vital signs.

In establishing an exercise program for a patient with type I diabetes, the therapist should be aware that regular exercise should: A. Decrease blood glucose levels and decrease the insulin sensitivity B. Decrease blood glucose levels and increase the insulin sensitivity C. Increase blood glucose levels and decrease the insulin sensitivity D. Increase blood glucose levels and increase the insulin sensitivity

B. Decrease blood glucose levels and increase the insulin sensitivity Exercise leads to hypoglycemia (low blood glucose levels) so you can rule out C and D. The reason your blood glucose decreases when exercising is because exercise increases insulin sensitivity (or effectiveness) and begins taking too much glucose out of the blood.

A PT is working on rehabilitating a 29-year-old ACL repair patient using aquatic therapy. The patient is immersed to the level of sternoclavicular notch. Which of the following is the MOST expected response at rest during aquatic therapy? A. Decreased cardiac output B. Decreased systolic blood pressure C. Increased heart rate D. Increased VO2 max

B. Decreased systolic blood pressure Imagine you took a vacation to the beach. You are super relaxed and chilling in the water. Your blood pressure will be decreasing. Cardiac output is increasing and heart rate is decreased because you are relaxed.

A 54-year-old patient presents to an outpatient clinic after a partial rotator cuff repair. Which of the following describes the optimal parameters for duty cycle and MVIC to address the injured rotator cuff muscles? A. Duty cycle of 30%; current intensity to produce 30% MVIC B. Duty cycle of 20%; current intensity to produce 40% MVIC C. Duty cycle of 60%; current intensity to produce 40% MVIC D. Duty cycle of 80%; current intensity to produce 20% MVIC

B. Duty cycle of 20%; current intensity to produce 40% MVIC Duty cycle is time on/total time (on + off). Duty cycle is never above 50% if there is an on and off!!! So, you can rule out C and D. The key to these questions is that you want to have the lowest duty cycle with the greatest MVIC. This means that you are putting little energy forward but are achieving the greatest contraction. Think of it like an investment. You invest 20% of you money and you get back 40% worth of your money. You don't want to take a risk and invest too much.

A patient has weakness of the quadriceps secondary to a femoral nerve injury. When examining the patient's gait from heel strike to foot flat, the physical therapist would MOST likely observe the patient compensate for this injury with which of the following gait deviations? A. Excessive dorsiflexion B. Forward trunk lean C. Excessive knee flexion D. Vaulting on the contralateral limb

B. Forward trunk lean Yes, the patient will have excessive knee flexion with quadriceps weakness BUT that is not a compensation for the injury but the natural result. Instead a patient with quadriceps weakness may use a forward trunk lean to compensate for their impairment. A forward trunk lean reduces the flexion moment at the knee and helps compensate for the quadriceps weakness.

A physical therapist reads in the medical record that patient was recently prescribed a thrombolytic agent. Which condition would be considered a contraindication to this type of pharmacological agent? A. Myocardial infarction B. Hemorrhagic stroke C. Pulmonary embolism D. Venous thrombosis

B. Hemorrhagic stroke Thrombolytic agents facilitate clot dissolution through the conversion of plasminogen to plasmin. Plasmin breaks down clots and allows occluded vessels to reopen to restore blood flow. Side effects include hemorrhage and cardiac arrhythmia. Hemorrhagic stroke is characterized by abnormal bleeding in the brain due to a rupture in a blood vessel. Thrombolytic agents are contraindicated for individuals experiencing a hemorrhagic stroke as this can result in increased hemorrhaging. However, thrombolytic agents are indicated for the management of ISCHEMIC stroke.

On assessment, the therapist notices redness on the skin as pictured below. Which of the following is the MOST APPROPRIATE description of this finding? A. Xanthoma B. Herpes zoster C. Rosacea due to H. Pylori D. Maculopapular rashes

B. Herpes zoster Herpes zoster is also called shingles. Shingles is an outbreak of a rash or blisters on the skin that may be associated with severe pain. The pain is associated with the involved nerve root and associated dermatome and generally presents on one side of the body or face in a pattern characteristic for the involved site. Early signs of shingles include burning or shooting pain and tingling or itching. The rash or blisters are present anywhere from 1 to 14 days.

A physical therapist observes that a patient has difficulty controlling the affected lower extremity during the loading response. This phase of gait is characterized by which of the following muscle activity responses? A. Increased quadriceps activity and increased hamstrings activity B. Increased quadriceps activity and decreased hamstrings activity C. Decreased quadriceps activity and increased hamstrings activity D. Decreased quadriceps activity and decreased hamstrings activity

B. Increased quadriceps activity and decreased hamstrings activity The goal of the loading response phase is to accept body weight onto the stance limb in a manner that ensure limb stability and permits forward progression. The loading response phase requires increased quadriceps activity to limit the rate of knee flexion. Hamstrings activity is decreased since the muscles are no longer needed to prevent knee hyperextension.

In order to conduct an experimental study on hip strengthening and walking speeds in lower limb amputees, a physical therapist divides 30 subjects into two groups. One group is treated with a 6-week hip strengthening intervention program and gait training and the other group is only given gait training. In this experimental design, the hip strengthening intervention is the: A. Continuous variable B. Independent variable C. Dependent variable D. Reliable variable

B. Independent variable The intervention/ activity (hip strengthening) is an independent variable that brings about a change in the dependent variable (speed of lower limb amputees).

A physical therapist initiates gait training on a patient who has T10 spina bifida. Initially, the BEST method to teach the patient how to maintain standing is with the use of which of the following devices? A. Bilateral hip-knee-ankle-foot orthoses (HKAFO) and forearm crutches B. Parapodium and the parallel bars C. Bilateral knee-ankle-foot orthoses (KAFO) and the parallel bars D. Bilateral ankle-foot orthoses (AFO) and the parallel bars

B. Parapodium and the parallel bars The parapodium is a HKAFO with a thoracolumbar orthosis that supports the trunk and lower extremities. It has a large base of support and is used with or without an assistive device. This would be ideal for a patient with T10 spina bifida to initiate standing within the parallel bars.

A PT is examining a 10-year-old patient diagnosed as level 3 on the gross motor classification of CP. According to the gross motor classification of CP what is the MOST likely ambulation status of this patient? A. Patient will walk without restrictions but will have limitations in more advanced gross motor skills B. Patient will walk with assistive device with limitations in walking outdoors and in the community C. Patient self mobility will be severely limited, even with the use of assistive technology D. Patient will walk without assistive device with limitations in walking outdoors and in the community

B. Patient will walk with assistive device with limitations in walking outdoors and in the community 1- walk without restrictions 2- walk without an assistive device but some limitations 3- walk with assistive device and limitations 4- self mobility is severely limited; transported or use power mobility outdoors and in community 5- self mobility is severely limited even with assistive technology

A 36-year-old female is experiencing chronic pain and tenderness in her cervical spine at the level of C5-C6 vertebrae. The physician determines there is reduced space between the vertebrae. What will be the MOST appropriate PT intervention? A. Provide an ice pack to relieve pain B. Perform cervical mobilization by moving the C5 vertebrae anteriorly C. Perform cervical mobilization by moving the C6 vertebrae anteriorly D. Provide ultrasound therapy to relieve pain

B. Perform cervical mobilization by moving the C5 vertebrae anteriorly Ice pack is considered "lame" in the NPTE world so you can typically rule that out. Never choose ultrasound because there is no evidence to support it so rule out D. You are left with B and C. To create space between C5 and C6 you will want to mobilize anteriorly at C5. This is because when you mobilize anteriorly at C5 it will also move superiorly. If you performed this mobilization at C6 you would be pushing C6 superiorly and closer to C5, thus decreasing the space even more.

The PT is observing the gait of a patient with right transfemoral amputation. The therapist notices excessive right plantar flexion at heel strike. What could be the possible cause for this foot slap? A. Plantar flexion bumper too rigid B. Plantar flexion bumper too soft C. Heel cushion is too rigid D. Excessive inset of the foot

B. Plantar flexion bumper too soft Stick to the plane! You can automatically rule out D because it is not sticking to the plane and would cause a lateral thrust. Both A and C would cause more dorsiflexion and therefore excessive knee flexion would be seen. If the plantarflexion bumper is too soft it will not stop the individual from going into plantar flexion so that will cause excessive plantar flexion. Rigid heel cushion- excessive knee flexion Soft heel cushion- knee hyperextension

A patient who has left hemiplegia post CVA is referred for orthotic examination. Significant results of manual muscle testing include: hip flexion 3+/5, hip extension 3/5, knee flexion 3+/5, knee extension 3+/5, ankle dorsiflexion 2/5, and ankle inversion and eversion 1/5. Sensation is intact and no abnormal tone is noted. Which of the following orthoses is the MOST appropriate for this patient? A. Knee-ankle-foot orthosis with a locked knee B. Plastic articulating ankle-foot orthosis C. Metal upright ankle-foot orthosis locked in neutral D. Prefabricated posterior leaf spring orthosis

B. Plastic articulating ankle-foot orthosis Since the patient has voluntary control of their knee they do not need a knee-ankle-foot orthosis so you can rule out A. A metal upright would be used for a patient with abnormal tone and/or sensory deficits which this patient does not have so you can rule out C. The posterior leaf spring would not offer enough stability for this patient so rule out D. This leaves you with a plastic articulating ankle-foot orthosis. The patient's strength at the hip and knee allows for an AFO to be used. A plastic AFO is appropriate since there is intact sensation and an articulating ankle joint is recommended to improve biomechanics during gait if there is no abnormal tone.

A patient who fell while running 3 days ago reports diffuse lateral ankle pain with active movement. The patient exhibits localized swelling distal and anterior to the lateral malleolus. Minimal laxity is noted with an anterior drawer test. Which of the following interventions would be MOST appropriate for the patient at this time? A. Gastrocnemius-soleus stretching B. Posterior talocrural joint mobilizations C. Stationary cycling for up to 30 minutes D. Active range of motion to end-range

B. Posterior talocrural joint mobilizations The patient has signs and symptoms consistent with a diagnosis of a lateral ankle (inversion) sprain. A grade 2 lateral ankle sprain is characterized by localized swelling and more diffuse lateral tenderness (Dutton, p. 1151). Gentle posterior talocrural joint mobilizations should be performed in the acute stage of healing to maintain mobility and inhibit pain.

When analyzing the patient's gait, the PT observes a collapse of the medial arch and over-pronation during stance phase. Which muscle would you strengthen to support the medial arch, and also assist in decelerating pronation during stance? A. Anterior tibialis B. Posterior tibialis C. Soleus D. Abductor hallucis

B. Posterior tibialis Think about the patient that you treated for posterior tibialis tendonitis who had a collapsed medial arch! Posterior tibialis is a dorsiflexor.

Nearly 2 months ago, a patient noticed left shoulder pain after walking the dog. This pain has progressively worsened. The patient now is unable to move the left upper extremity overhead while performing ADLs. An orthopedic surgeon diagnosed the problem as adhesive capsulitis. The MOST effective direction for glenohumeral mobilization for this patient would be: A. Posterosuperior translatory glides B. Posteroinferior translatory glides C. Anterosuperior translatory glides D. Anteroinferior translatory glides

B. Posteroinferior translatory glides Capsular pattern of restriction is present in adhesive capsulitis i.e. lateral rotation> abduction> medial rotation. Clinical studies have demonstrated that posterior glide is more effective than an anterior glide to increase glenohumeral external rotation range of motion (exception to the convex- concave rule) in adhesive capsulitis. Patient's external rotation can be treated with posterior glide and inferior glide are used to resolve abduction. So, posterior inferior glides should be given in this patient.

A physical therapist completes a family training session with a patient rehabilitating from a spinal cord injury. During the training, the patient asks a question regarding their functional ability following rehabilitation. Which of the following responses is the MOST appropriate option for the therapist to utilize? A. Explain that it is difficult to predict since all patients progress differently B. Provide information on the expected prognosis based on the nature and severity of the injury C. Refer the patient to the director of rehabilitation D. Refer the patient to the physiatrist

B. Provide information on the expected prognosis based on the nature and severity of the injury Physical therapists can share information with patients related to projected functional outcomes following rehabilitation. Physical therapists should be careful to make sure that the topics addressed fall within their scope of practice.

A physical therapist who was working in an outpatient clinic is leaving the practice for better career opportunities. He is the only lymphedema certified specialist at that location and single-handedly manages all the patients with lymphedema. Which of the following will be the BEST course of action for the physical therapist toward patients requiring continued care? A. Discharge the patients before leaving B. Provide information to patients about alternative options to obtain care C. Find a replacement for the position before leaving D. Therapist is not allowed to terminate provider relationship if the patient requires continued care

B. Provide information to patients about alternative options to obtain care Per the Code of Ethics for Physical Therapists: Principle #5: Physical therapists shall fulfill their legal and professional obligations. (Core Values: Professional Duty, Accountability). Physical therapists shall provide notice and information about alternatives for obtaining care in the event the physical therapist terminates the provider relationship while the patient or client continues to need physical therapist services.

A physical therapist attempts to strengthen the lumbricals on a patient who has a low metatarsal arch. Which of the following exercises would be the MOST appropriate? A. Resisted extension of the metatarsophalangeal joint B. Resisted flexion of the metatarsophalangeal joint C. Resisted abduction of the metatarsophalangeal joint D. Resisted adduction of the metatarsophalangeal joint

B. Resisted flexion of the metatarsophalangeal joint The lumbricals act to flex the metatarsophalangeal joints so they can be strengthened by resisting that action. This can be performed with manual resistance or by gathering a towel or another similar object placed on the floor.

Three DPT students are looking at effects of drugs on falls in older individuals. The students are using data from previous patient medical records to compare drug usage in fallers and non-fallers. Which of the following will be the MOST appropriate study design? A. Clinical case report B. Retrospective cohort study C. Randomized controlled trials D. Prospective cohort study

B. Retrospective cohort study

A patient with a right transfemoral amputation is referred to a clinic. During gait analysis, the PT observes that the patient's trunk is bending laterally on the right side. Which of the following would be the MOST likely cause of this gait abnormality? A. Right socket is too small B. Right medial wall too high C. Right lateral wall too high D. Right prosthetic limb is too long

B. Right medial wall too high A right medial wall that is too high is equivalent to tight hip adductors on the R. If the hip adductors are tight you would have weak hip abductors. Weak hip abductors on the R would result in a left hip drop and the patient would lean to the right to compensate.

A physical therapist administers a series of cranial nerve tests to a patient with a confirmed lower motor neuron disease. Assuming the patient has a lesion impacting the right hypoglossal nerve, which clinical presentation would be most likely? A. Right-sided tongue atrophy and deviation toward the left with tongue protrusion B. Right-sided tongue atrophy and deviation toward the right with tongue protrusion C. Left-sided tongue atrophy and deviation toward the left with tongue protrusion D. Left-sided tongue atrophy and deviation toward the right with tongue protrusion

B. Right-sided tongue atrophy and deviation toward the right with tongue protrusion "Lick your lesions". Your tongue always goes toward the lesion!!

A 21 year old male works as a cashier at a gas station. He is having trouble giving cash back to a customer. According to roll and slide mechanism, during right forearm pronation at the proximal radioulnar joint, the PT should do the following to the radial head: A. Roll and slide posteriorly B. Roll anteriorly and slide posteriorly C. Roll and slide anteriorly D. Roll posteriorly and slide anteriorly

B. Roll anteriorly and slide posteriorly Roll and slide are always in the opposite direction at the proximal radioulnar joint so you can rule out A and C.

A patient who has Stage 0 lymphedema should be educated to AVOID which of the following activities? A. Prolonged air travel B. Running barefoot C. Aquatic activities D. Stationary cycling

B. Running barefoot Individuals who have lymphedema in any stage (Stage 0 or risk for other stages) should protect the skin and avoid any activities that could pierce the skin tissue.

A patient is unable to correctly perform the push up due to weakness in the push-up sling. Based on the principle of muscle slings, which muscle group should be strengthened to help in correcting the push-up technique? A. Cervical flexors B. Serratus anterior C. Hamstrings D. Ankle dorsiflexors

B. Serratus anterior Push-up sling: cervical extensors, serratus anterior, abdominals, quads Wall push-ups are ideal for strengthening the serratus anterior!!! Remember that the serratus anterior is innervated by the long thoracic nerve.

A patient presents to the clinic four weeks after a reverse arthroplasty of the right shoulder. Which of the following movements would you perform while treating the patient at this stage? A. Shoulder ER beyond 20 degrees B. Shoulder elevation in scapular plane up to 90 degrees C. Shoulder extension beyond 0 degrees D. Shoulder IR up to 30 degrees

B. Shoulder elevation in scapular plane up to 90 degrees Phase 1 of rehabilitation post reverse shoulder arthroplasty lasts for up to 6 weeks. The motions that are restricted during this period include: - No GH extension or internal rotation past neutral - No combined GH extension, adduction, and internal rotation - 0°-20° external rotation - Up to 90°-120° arm elevation in scapular plane

An adult patient who reports a new onset of back pain had a radiograph that identified wedging of the L1 vertebral body. Which of the following muscle groups would be MOST appropriate to stretch? A. Should horizontal adductors, shoulder lateral (external) rotators, hip flexors, and hip lateral (external) rotators B. Shoulder horizontal adductors, shoulder medial (internal) rotators, hip flexors, and hip medial (internal) rotators C. Shoulder horizontal abductors, shoulder lateral (external) rotators, hip extensors, and hip medial (internal) rotators D. Shoulder horizontal abductors, shoulder medial (internal) rotators, hip extensors, and hip lateral (external) rotators

B. Shoulder horizontal adductors, shoulder medial (internal) rotators, hip flexors, and hip medial (internal) rotators The symptoms and radiographic bony changes suggest osteoporosis. Compression fractures are commonly associated with trunk flexion, and symptoms are provoked with flexion activities. Stretching of the antagonist muscles, such as the shoulder horizontal adductors and medial (internal) rotators, hip flexors and medial (internal) rotators is recommended for patients who have compression fractures of the vertebral bodies secondary to osteoporosis.

If the forced expiratory volume in one second (FEV1) test is negative for airway obstruction in 99 percent of individuals without lung disease, then the measurement of FEV1 BEST represents which of the following research terms? A. Sensitive B. Specific C. Reliable D. Valid

B. Specific Specificity is the probability of obtaining a negative test among individuals WITHOUT the disease

A patient with arterial insufficiency is starting a walking program using a treadmill. The patient complains of moderate pain and reaches a 2 on the claudication scale. What is the NEXT step the therapist should take? A. Continue walking, pain is an expected and normal response B. Stop exercise, sit patient down, continue walking when pain subsides C. Continue exercising but slow the treadmill speed to 1.0 mph D. Stop walking since patient isn't tolerating it well, use arm cycle ergometry instead

B. Stop exercise, sit patient down, continue walking when pain subsides Walking programs are used for patients with claudication, which is common in patients with arterial insufficiency. The patient should report a 1 on claudication scale (mild, first feeling of any pain) within 3-5 minutes, stop the exercise if they reach a 2 (moderate pain). Rest until the pain subsides if they reach a 2. The session should last 20-60 minutes with intervals if necessary, 3-5 days a week. Therapist should record the time of pain onset and duration.

A physical therapist treats a patient diagnosed with spinal stenosis. As part of the treatment plan, the patient lies in the prone position on a treatment plinth with a hot pack draped over the low back. Which of the following methods is the MOST effective for the therapist to use to monitor the patient during application of the hot pack? A. Check on the patient at least every ten minutes B. Supply the patient with a bell to ring if the hot pack becomes too hot C. Instruct the patient to remove the hot pack if it becomes too hot D. Select an alternate superficial heating modality

B. Supply the patient with a bell to ring if the hot pack becomes too hot It is important to check on the patient but it needs to be done more frequently than every ten minutes. Supplying a patient with a bell to ring if the hot pack becomes too intense provides a form of instant communication with the therapist. The use of a bell DOES NOT negate the need for the PT to formally check on the patient frequently.

A PT needs to check the walking gait of a patient who suffered from a CVA and decides to record a video in slow motion. The 3 hamstring muscles contract eccentrically during which phase of the gait cycle? A. Initial swing B. Terminal swing C. Midstance D. Terminal stance

B. Terminal Swing

The three hamstring muscles (semimembranosus, biceps femoris long head, and semitendinosus) have their MOST intense action in which phase of gait? A. Initial swing B. Terminal swing C. Midstance D. Terminal stance

B. Terminal swing Eccentric action is more intense than concentric action of a muscle. In terminal swing, the hamstrings act eccentrically to decelerate knee extension in preparation for the placement of the foot on the ground.

During exercise, a patient has the sudden onset of dyspnea, crackles (rales) on auscultation, and an S3 heart sound. Which of the following courses of action is MOST appropriate for the physical therapist? A. Terminate exercise due to myocardial ischemia B. Terminate exercise due to cardiovascular pump dysfunction C. Continue exercise with close monitoring due to bronchiectasis D. Continue exercise with close monitoring due to ventilatory pump dysfunction

B. Terminate exercise due to cardiovascular pump dysfunction Presence of an S3 heart sound is the hallmark of cardiovascular pump failure. In patients who have pump failure, crackles (rales) are heard on inspiration and do not disappear with coughing. Crackles (rales) may be absent at rest and appear during exercise, indicating that the exercise intensity is too strenuous and is likely causing a transient pump failure. Exercise should be terminated, and dose must be adjusted prior to resuming exercise.

A PT is treating a patient with complaints of chest pain. The PT attempts to assess heart sounds with a stethoscope. Which of the following is true about the first sound during auscultation of the heart? A. The first sound is of the closure of the aortic and pulmonic valves B. The first sound is of the closure of the mitral and tricuspid valves C. The first sound is of the opening of the aortic and pulmonic valves D. The first sound is of the opening of the mitral and tricuspid valves

B. The first sound is of the closure of the mitral and tricuspid valves The first sound heard is the "lub" (S1) sound which comes from the mitral and tricuspid valves so you can rule out A and C. Like doors in your house, when you close the door it makes a sound but when you open it it is quiet. The heart works the same way. When it closes it makes the sound so you can rule out D.

A patient who had a cerebrovascular accident exhibits a flexion synergy of the left upper extremity. To promote good upper extremity movement, a physical therapist should mobilize the patient's scapula toward which of the following directions? A. Upward rotation and retraction B. Upward rotation and protraction C. Downward rotation and retraction D. Downward rotation and protraction

B. Upward rotation and protraction Flexion synergy of the upper extremity includes scapular retraction/elevation or hyperextension. In the upper extremity, correct passive range of motion techniques require careful attention to lateral (external) rotation and distraction of the humerus, especially as ranges approach 90° of flexion or more. The scapula should be mobilized on the thoracic wall with an emphasis on upward rotation and protraction to prevent soft tissue impingement in the subacromial space during overhead movements of the arm.

A patient complains of waking up several times at night from severe "pins and needles" in the right hand. On awakening, the hand feels numb for half an hour and fine hand movements are impaired. The therapist's examination reveals sensory loss and paresthesias in the thumb, index, middle, and lateral half of the ring finger and reduced grip and pinch strength. Some thenar atrophy is present. Based on these examination findings, the MOST appropriate diagnosis is: A. Thoracic outlet syndrome B. Ulnar nerve entrapment C. Carpal tunnel syndrome D. Pronator teres syndrome

C. Carpal tunnel syndrome The characteristic features of carpal tunnel syndrome include intermittent pain and paresthesias in the median nerve distribution of the hand and, muscle weakness/ paralysis is also present. The symptoms of CTS are typically worse at night due to the position of wrist flexion typically adopted during sleep and can be associated with morning stiffness.The chief complaint of TOS is diffuse arm and shoulder pain, especially when the arm is elevated beyond 90 degrees.Ulnar nerve entrapment has different levels of affection and it will cause weakness in the muscles supplied by the ulnar nerve.Pronator teres syndrome -The patient typically complains of an insidious onset of pain that is usually felt on the anterior aspect of the elbow, radial side of the palm, and the palmar side of the first, second, third, and half of the fourth digits. Unlike CTS, there is no Tinel's sign at the wrist, and there are no nocturnal symptoms.

A patient attending outpatient physical therapy for right shoulder pain has a positive sulcus sign. This finding would MOST likely be present in which of the following medical conditions? A. Early-onset Alzheimer's disease B. Rupture of the biceps tendon C. Cerebrovascular accident D. Duchenne muscular dystrophy

C. Cerebrovascular accident A common finding with a CVA is weakness or paralysis contralateral to the side of the lesion, often leading to a flaccid shoulder and potentially a positive sulcus sign even after the patient has completed a rehabilitative program.

A computer programmer presents to a PT clinic with increased neck discomfort due to faulty posture. The physical therapist designs rehabilitation protocol to decrease spasm of the upper trapezius. If the PT wants to apply real time biofeedback, which protocol would be the BEST to implement? A. Closely placed electrodes with high sensitivity B. Widely spaced electrodes with low sensitivity C. Closely placed electrodes with low sensitivity D. Widely spaced electrodes with high sensitivity

C. Closely placed electrodes with low sensitivity Trapezius is in spasm, the electrodes should be closely placed and with low sensitivity to minimize cross-talk. For spastic muscles we start with low sensitivity and gradually increase it while with flaccid or weak muscles we start with high sensitivity. Wide space electrodes recruits more muscle activity while closely spaced is for specific muscle.

A patient ambulates with an absent heel strike (initial contact) on the left. Range of motion assessment reveals a loss of left ankle dorsiflexion. Which of the following modality and treatment interventions would be MOST appropriate? A. Moist heat application followed by stretching into dorsiflexion B. Cryotherapy application while stretching into dorsiflexion C. Continuous ultrasound while stretching into dorsiflexion D. Continuous ultrasound followed by stretching into dorsiflexion

C. Continuous ultrasound while stretching into dorsiflexion Continuous ultrasound along with stretching is the best way to increase soft tissue extensibility, thereby reducing soft tissue shortening and increasing joint range of motion. Ultrasound is best used because it can penetrate into the muscle and has been shown to be better than when stretching is used alone.

A patient diagnosed with Guillain-Barre syndrome works on weight shifting activities while standing in the parallel bars. The PRIMARY objective of this activity is to improve which stage of motor control? A. Mobility B. Stability C. Controlled mobility D. Skill

C. Controlled mobility Controlled mobility refers to the ability to move within a weight bearing position or rotate around a long axis Examples of controlled mobility include weight shifting or activities on prone on elbows

A golfer complains of sharp localized pain at the right side of his low back when swinging towards the right. Sitting, stooping, and bending to the left relieves the pain. Based on the presentation, the MOST likely cause is: A. Sacroiliac dysfunction B. Spondylolisthesis C. Facet arthropathy D. Spinal lateral stenosis

C. Facet arthropathy In Facet arthropathy, pain is sharp, localized, increases with extension & ipsilateral rotation. Pain occurs while swinging to right and is relieved with flexion and contralateral rotation. Also, no sensory/motor changes occur as seen with stenosis. In spondylolisthesis, pain increases on extension, ipsilateral side bending and opposite side rotation.

A patient has Fair (3/5) quadriceps strength, intact sensation in the lower extremity, and 0/10 pain. Which of the following electrical stimulation parameters would be MOST appropriate for the patient? A. Frequency of 35 pps, duration of 50 microseconds B. Frequency of 35 pps, duration of 150 microseconds C. Frequency of 50 pps, duration of 250 microseconds D. Frequency of 150 pps, duration of 50 microseconds

C. Frequency of 50 pps, duration of 250 microseconds For a large muscle group with intact innervation, the most effective parameters for promoting increased muscle strength are a frequency of 35-80 pps and a pulse duration of 200-350 microseconds

A physical therapist performs a chart review of a new patient and finds the patient is positive for the Helicobacter pylori bacterium. The therapist should anticipate that the patient presents with which of the following medical conditions? A. Meningitis B. Pneumonia C. Gastric ulcer disease D. Tetanus

C. Gastric ulcer disease Gastric ulcer disease is often caused by the gram-negative bacterium Helicobacter pylori that is found in the upper GI tract. This infection is believed to be a potential cause of gastroduodenal ulcers and must be treated with antibiotics. It is treated primarily with amoxicillin or clarithromycin and may be combined with other medications to enhance the healing process of any ulcers that are present.

A physical therapist observes the gait of a 34-year-old male patient. The PT suspects a leg length discrepancy. Which of the following gait deviations is MOST likely seen by the therapist? A. Increased dorsiflexion of the short limb during swing and increased plantar flexion of the long limb during stance B. Decreased knee flexion and increased dorsiflexion of the long limb during stance and increased dorsiflexion of the short limb during swing C. Increased dorsiflexion with early heel rise of the long limb at heel off and increased plantar flexion of the short limb during stance D. Increased plantar flexion of the long limb at heel strike and decreased knee flexion of the short limb during heel off

C. Increased dorsiflexion with early heel rise of the long limb at heel off and increased plantar flexion of the short limb during stance Imagine two legs and one leg is shorter. In order to reach the ground with the short leg you will need to point your toes down (PF). So you will always have PF of the short limb with a leg length discrepancy. OR imagine you are wearing heels and one of them breaks. For the rest of the day you have to walk on your toes on the leg with a broken heel.

A patient with Stage 4 (H&Y Scale) Parkinson's disease presents with tonal changes in the trunk and extremities. What should be NEXT performed by the PT to complete the examination of the rigidity? A. Measure the limbs muscles strength using MMT B. Timed Test for rapid alternative movements C. Inspection of voluntary repetitive movements D. Check the location, persistence, and severity of tremor

C. Inspection of voluntary repetitive movements In order to test for rigidity you NEED to inspect voluntary repetitive movements. A- looking at strength B- determining the effects of bradykinesia D- looking at tremors 1: ONE side of body 2: TWO sides of body and no difficulty walking 3: both sides of body and MINIMAL difficulty walking 4: both sides of body and MODERATE difficulty walking 5: both sides of body and UNABLE to walk

A 44-year-old male who consumes excessive amounts of alcohol is referred to the PT clinic for knee pain. During treatment, the patient reports right-sided shoulder pain. Which of the following structures is the MOST likely source of the referred pain? A. Prostate B. Appendix C. Liver D. Kidney

C. Liver Excessive amounts of alcohol damage your liver. Liver pathologies will refer pain to the right shoulder.

A 34-year-old male has been diagnosed with a complete C5 spinal cord injury. Patient used a power chair to commute from the parking lot to the PT clinic. While performing a transfer, the MOST important consideration is to: A. Make sure he is using his shoulder extensors to support his upper body B. Make sure his legs are supported by PT aide C. Make sure his power chair is turned off D. Make sure to use the sliding board

C. Make sure his power chair is turned off The patient will have no innervation of the UE's so you can rule out A and D. A PT aide should not be assisting with transfers so you can rule out B.

A 60-year-old male is undergoing cardiac rehabilitation post complicated MI in the hospital. On day two, the PT wants to progress the patient to sitting. Which of the following is an appropriate INITIAL task for this patient? A. Make the patient stand and do weight bearing activities B. Make the patient sit on the upright chair during the visitors time C. Make the patient sit on reclining chair and check vitals D. Patient is not ready for upright posture yet

C. Make the patient sit on reclining chair and check vitals Before you allow the patient to do sitting activities you need to make sure their vitals are okay in that position.

A patient is admitted to a hospital for a left femur fracture after a recent fall at home. A review of the patient's medical chart reveals blood pressure (BP) of 165/90, triglyceride level of 160 mg/dL, and a fasting blood glucose level of 115 mg/dL. The patient's BMI is 40 kg/m^2 with a 54-inch waistline. These findings are MOST consistent with the medical diagnosis of: A. Chronic heart disease B. Type 2 diabetes C. Metabolic syndrome D. Cushing's syndrome

C. Metabolic syndrome The metabolic syndrome is characterized by a group of metabolic risk factors in one person. The risk of serious illness increases in anyone with three or more of the following factors: waist size of more than 40 inches in male, more than 35 inches in female, Triglycerides >150mg/dl, elevated blood pressure (130/85 mm Hg or more), fasting blood sugar >100 mg/dl, HDL cholesterol less than 50 mg/dL for woman and less than 40 mg/dL for man.

A patient who has a C6 spinal cord injury (ASIA Impairment Scale A) is MOST likely to exhibit which of the following movement patterns during inhalation? A. Inward motion of the abdomen and inward motion of the upper chest B. Inward motion of the abdomen and outward motion of the upper chest C. Outward motion of the abdomen and inward motion of the upper chest D. Outward motion of the abdomen and outward motion of the upper chest

C. Outward motion of the abdomen and inward motion of the upper chest A patient who has a C6 spinal cord injury retains full use of the diaphragm but lacks innervation to abdominal and intercostal musculature. The patient will display outward motion of the abdomen and inward motion of the upper chest. The outward motion of the abdominal area is caused by the diaphragm contracting and pushing abdominal contents forward and outward, and the inward motion of the upper chest is due to the lack of structural support from paralyzed thoracic musculature.

A physical therapist is working with an 18-year-old patient who has a complete T5 spinal cord injury. The patient has a stage 2 sacral pressure ulcer and is incontinent of bowel and bladder. Which is the MOST appropriate intervention for the patient? A. Apply low-volt monophasic pulsed current to manage the ulcer B. Perform independent curl-ups and bridging exercises C. Perform independent rolling from side to side D. Administer diet and medications to manage a flaccid bowel

C. Perform independent rolling from side to side Electric current to the sacral area is not advised for a SCI patient due to loss of sensation below T5. Curl ups and bridging is not achievable independently as the abdominals are not innervated. Managing a flaccid bowl will not heal the ulcer. Independent rolling is achievable with level of injury T5 and can be done to reduce pressure on the sacral area.

All of the following management guidelines are followed in the recovery phase of nerve injury EXCEPT: A. Use electrical stimulation to reinforce active movement B. Protect weak muscles with a splint C. Place the weak muscle in its lengthened position D. Desensitize the extremity involved with contact particles such as beans or macaroni

C. Place the weak muscle in its lengthened position The muscle should be positioned in its shortened position as it will put less pressure on the nerve. Positioning the muscle in lengthened position will increase the stress on the nerve. A, B, &D are management guidelines used in recovery phase.

You are evaluating a 37 y.o. secretary in an outpatient clinic who presents with right upper extremity pain shooting into the hand. On examination, the therapist performed the test as shown in the image and the patient complained of pain, weakness, and tingling in the hand. To confirm the diagnosis, the therapist can perform which of the following additional tests? A. Positive Neer's test B. Positive Roos test C. Positive Adson's test D. Positive Anterior Apprehension test

C. Positive Adson's test The test shown in the image is Roos test which is used for the diagnosis of thoracic outlet syndrome. To confirm the diagnosis, the therapist can perform Adson test which is one of most common methods of testing for thoracic outlet syndrome. Anterior apprehension test is used for anterior shoulder instability; Neer's test is used for impingement.

A 28-year-old male patient with a left above knee amputation is referred to an outpatient amputee clinic. During gait analysis, the physical therapist observes the gait deviation shown in the picture below. Which of the following is the MOST likely cause of this deviation? A. The foot is too far medially inset B. Inadequate medial rotation of the knee joint C. Prosthetic knee bolt is externally rotated D. The socket is in excessive abduction

C. Prosthetic knee bolt is externally rotated The picture is showing a medial heel whip which is a swing phase deviation. With a heel whip, the heel will move either internally or externally when the knee is flexed. If the knee bolt was rotated too far externally then the heel will compensate. So the knee is moving outward and the heel moves inward to avoid falling. If it was a lateral heel whip the knee bolt would be internally rotated!!!

The patella acts to increase ____________, and patellofemoral compression forces are increased by _______________. A. Quadriceps moment arm; a more extended knee position B. Hamstrings moment arm; a more extended knee position C. Quadriceps moment arm; a more flexed knee position D. Quadriceps tension; a more extended knee position

C. Quadriceps moment arm; a more flexed knee position The primary function of the patella is to increase the moment arm of the quadriceps muscle in its function to extend the knee. In full extension, because there is minimal to no contact of the patella with the trochlear groove, there is no compression of the articular surfaces. There is increase in compressive forces as the knee flexes.

A physician analyzes the results of a magnetic resonance imaging (MRI) study to determine the extent of a patient's lung cancer. When staging the patient's cancer using the TNM system, which of the following factors would NOT be considered? A. Size of the tumor B. Involvement of the lymphatic system C. Rate of growth of the cancer cells D. Presence of metastasis

C. Rate of growth of the cancer cells The TNM system is one of the most commonly used methods for staging cancer. T- tumor size N- nodes (has it spread to the lymph nodes?) M- metastasis

A PT is treating a patient with a diagnosis of ALS. The patient has weakness of all extremities and gets fatigued very easily while doing ADLs. Patient's main goal is to maintain mobility and function as much as possible. Which of the following will be LEAST appropriate for this patient? A. Recommending soft foam collar for neck stability B. Taking frequent breaks during activities C. Recommend KAFO and walker for ambulation D. Slow, prolonged stretches and ROM exercises for UE and LE

C. Recommend KAFO and walker for ambulation A KAFO will add weight to their leg and if they are already getting fatigued this will not be a good idea.

A patient with a tracheostomy tube has been hospitalized for two days. During physical therapy the patient suddenly exhibits dyspnea, cyanosis of lips, and cramping of the right calf muscle. What should the therapist suspect based on the symptoms? A. Heart failure B. Deep vein thrombosis C. Respiratory distress D. Pulmonary embolism

C. Respiratory distress Dyspnea, shortness of breath, or cramping in the calf muscles are common signs of respiratory distress. Mucus plugging, tube displacement, disruption or disconnection of oxygen can cause respiratory distress in a tracheostomy patient.

A PT is having a 45-year-old male with a BMI of 23 exercise on a treadmill with a non-complex medical history. Vitals are monitored throughout the session. It is determined that resting HR was 65 bpm, during the exercise it was 155 bpm, and five minutes after the exercise had ended the HR returned to 65 bpm. The patient then asks his therapist how his twin brother might respond to the same exercise program he just completed. The patient states that they are the same age and height, but the brother weights 50 more pounds and has a BMI of 32. What is the most accurate response to exercise the unfit twin brother would have for HR five minutes after the exercise had ended? A. Resting HR was 90 bpm, during the exercise it was 170 bpm, and five minutes after the exercise had ended the HR returned to 90 bpm B. Resting HR was 45 bpm, during the exercise it was 155 bpm, and five minutes after the exercise had ended the HR returned to 65 bpm C. Resting HR was 75 bpm, during the exercise it was 165 bpm, and five minutes after the exercise had ended the HR returned to 100 bpm D. Resting HR was 65 bpm, during the exercise it was 100 bpm, and five minutes after the exercise had ended the HR returned to 90 bpm

C. Resting HR was 75 bpm, during the exercise it was 165 bpm, and five minutes after the exercise had ended the HR returned to 100 bpm First, the unfit person begins to exercise with a higher resting heart rate. Once the unfit person begins exercising, the heart rate will rise quicker to a total heart rate higher than that of the fit person. The unhealthy person's heart rate will then take longer to slow down than the average person, balancing out to a higher resting heart rate once again.

A physical therapist is performing cranial nerve testing on a 55-year-old male patient. When observing the patient's right papillary response to light, the physical therapist observes that there is no constriction of the right pupil but the left pupil does constrict. Which cranial nerve is the MOST likely cause of this impairment? A. Left oculomotor nerve (CN III) B. Right optic nerve (CN II) C. Right oculomotor nerve (CN III) D. Left trochlear nerve (CN IV)

C. Right oculomotor nerve (CN III) CN II is afferent and CN III is efferent component of pupillary reflex. Pupillary reaction (constriction) is tested by shining light in eye. When light is shown in the right eye, intact right CN II (optic) carries sensory information, which stimulates efferent CN III (occulomotor) to cause constriction of the pupil. As there is constriction of left pupil and no constriction of right pupil, it indicates that right CN III is affected.

A child who has a myelomeningocele at the T7 level has a new onset of vomiting, lethargy, irritability, headache, and increased seizure frequency. What is the MOST likely cause of these signs and symptoms? A. Latex allergy B. Tethered cord C. Shunt dysfunction D. Chiari II malformation

C. Shunt dysfunction At least 85% of children who have myelomeningocele have hydrocephalus, and 80% to 90% will require a shunt, especially those with high-level lesions. Shunt dysfunction is common, and therapists should be familiar with signs and symptoms for early detection. Early warning signs include the clinical manifestations described in the stem.

A physical therapist discusses the importance of proper posture with a patient post back surgery at the L3-L4 spinal level. Which body position would place the MOST pressure on the lumbar spine? A. Standing in the anatomical position B. Standing with 45 degrees of hip flexion C. Sitting in a chair slouching forward D. Sitting in a chair with reduced lumbar lordosis

C. Sitting in a chair slouching forward (lowest to greatest load) Supine, sidelying, standing in anatomical position, standing with 45 degrees of hip flexion, sitting in a chair with reduced lumbar lordosis, sitting in a chair slouching forward

A 13-year-old girl discusses the possibility of ACL reconstruction with an orthopedic surgeon. The patient injured her knee while playing soccer and is concerned about the future impact of the injury on her athletic career. Which of the following factors would have the GREATEST influence on her candidacy for surgery? A. Anthropometric measurements B. Hamstrings/quadriceps strength ratio C. Skeletal maturity D. Somatotype

C. Skeletal maturity Due to the potential impact on future bone growth, lack of skeletal maturity can be a contraindication to ACL reconstruction surgery.

A 25-year-old female with supraventricular tachycardia has an exacerbation of her symptoms due to anxiety related to exams. She asked her cardiologist for an alternate to the beta-blockers due to unfavorable side effects. The most appropriate intervention by her cardiologist would be: A. Calcium channel blockers B. Anti anxiety medication to relieve her exam induced stress C. Teach her Valsalva maneuver D. Increase the dose of beta blockers

C. Teach her Valsalva maneuver Carotid massage and Valsalva maneuver are effective methods of reducing the heart rate if done correctly. These two methods are easy to do and do not have any undesirable side effects. A- beta blockers are the best drugs for SVT so changing to another category of drugs is not desirable B- anti-anxiety drugs are mostly sedative and are not appropriate for a patient that has to study and be alert D- if she wants off of beta blockers increasing the dosage would not be appropriate

A 2-year-old patient is being treated for gait training in outpatient physical therapy. The patient has calluses on the dorsal aspect of the toes and is unable to perform MTP extension. Which of the following is MOST likely true? A. The patient has a persistent ATNR B. The patient has an integrated STNR C. The patient has a persistent plantar grasp reflex D. The patient has an integrated plantar grasp reflex

C. The patient has a persistent plantar grasp reflex Plantar grasp reflex should be integrated by 9 months. Stimulus: pressure to ball of foot under toes Response: maintained flexion of toes

A patient has limited movements throughout the arc of forearm supination and pronation due to moderate to severe pain. Which of the following manual therapy techniques to the proximal radioulnar joint is MOST appropriate to perform initially? A. With the patient positioned in 5° of supination and 90° of elbow flexion, perform small-amplitude mobilizations at the beginning of the available range of motion. B. With the patient positioned in 10° of supination and 70° of elbow flexion, perform large-amplitude mobilizations at the end of the range of motion. C. With the patient positioned in 35° of supination and 70° of elbow flexion, perform large-amplitude mobilizations in the middle of the range of motion. D. With the patient positioned in full supination and full elbow extension, perform small-amplitude mobilizations at the end of the range of motion.

C. With the patient positioned in 35° of supination and 70° of elbow flexion, perform large-amplitude mobilizations in the middle of the range of motion. Small-amplitude mobilizations performed at the beginning of the available range of motion and large-amplitude movement in the middle of the range of motion are used for pain relief and to induce an analgesic effect. This patient has a pain-dominant joint condition. It is best to mobilize a joint with a pain-dominant condition in an open-packed position, which is 35° of supination and 70° of elbow flexion for the proximal or superior radioulnar joint.

A physical therapist employed in a rehabilitation hospital creates a professional development plan as part of their annual performance appraisal. Which of the following methods for attending continuing education courses would be the MOST appropriate plan to facilitate the therapist's development in their present practice setting? A. Attend courses approved by the Federation of State Boards of Physical Therapy B. Attend courses featuring nationally recognized experts C. Attend a minimum of two courses annually D. Attend courses related to primary patient care responsibilities

D. Attend courses related to primary patient care responsibilities The Federation of State Boards of Physical Therapy DOES NOT approve continuing education courses so you can rule A out. The physical therapist should attend continuing education courses that directly relate to their primary patient care responsibilities. Developing this particular skill set will allow the physical therapist to improve the quality of patient care within their current practice setting.

A physical therapist suspects that a patient's upper extremity range of motion limitation may be the result of a fear-based psychological response. Which of the following examination findings would be MOST consistent with the therapist's hypothesis? A. Decreased passive range of motion; weak but pain-free resistive testing B. Decreased active range of motion; strong but painful resistive testing C. Decreased active and passive range of motion; strong and pain-free resistive testing D. Decreased active range of motion; decreased effort with resistive testing

D. Decreased active range of motion; decreased effort with resistive testing Fear-based psychological responses during movement testing typically result in a voluntary limitation of active and resisted motion. The voluntary limitation of active motion and the lack of effort with resistive testing are often associated with the fear of pain.

A PT is performing cranial nerve tests on a patient. Which of the following is LEAST appropriate to assess the integrity of the facial nerve? A. Ask the patient to show their teeth B. Ask the patient to puff out both cheeks C. Apply sugar or saline solution to the anterior part of the tongue and ask the patient to identify D. Have patient clench teeth and hold against resistance

D. Have patient clench teeth and hold against resistance The facial nerve is both motor and sensory so all of these can be valid initially. Facial expressions are controlled by the facial nerve. To test the motor function of facial muscles you will raise eyebrows, frown, show teeth, smile, close eyes tightly, and puff out both cheeks. D is testing the integrity of the trigeminal nerve (CN 5). Some Say Marry Money But My Brother Says Big Brains Matter Most

During intervention for a patient with weak tibialis anterior muscles, the PT decides to use FES to improve ambulation. Stimulation should be initiated for the weak muscles during which phase of gait cycle? A. Mid stance to terminal stance B. Initial contact to mid stance C. Loading response to mid stance D. Initial swing to mid swing

D. Initial swing to mid swing Yes, it is important for the contraction of the tibialis anterior during initial contact to midstance to slow the foot when lowering from heel strike and not make it slap on the floor. However, it is more important to ensure that the patient's toes clear during swing phase to avoid tripping.

Which of the following bladder management techniques is MOST likely to be used for a patient with bladder dysfunction due to a cauda equina lesion? A. Sacral nerve modulation B. Pelvic floor biofeedback C. Pelvic floor strengthening exercises D. Intermittent catheterization

D. Intermittent catheterization Spinal cord lesions at the level of S2 and below lead to bladder areflexia and dysfunction of the external sphincter. Bladder tone is preserved, but bladder compliance decreases with time. Catheterization is a commonly employed intervention to avoid excessive bladder distention.

A 34-year-old male presents to an outpatient clinic with jaw pain. What is the resting position of the temporomandibular joint (TMJ) and its functional ROM available? A. Mouth closed, 55 mm B. Mouth open, 25 mm C. Jaw slightly closed, 25 mm D. Jaw slightly opened, 40 mm

D. Jaw slightly open, 40 mm

A 13-year-old boy has a shorter lower extremity with ipsilateral thigh and hip pain. When walking, the patient is MOST likely to maintain the involved hip in which of the following positions? A. Flexion B. Abduction C. Medial (internal) rotation D. Lateral (external) rotation

D. Lateral (external) rotation Slipped capital femoral epiphysis is the most common hip disorder of adolescents. Presentation is more common in males than females and occurs between the ages of 10-17 years. Pain is commonly reported in the thigh and hip with limited hip medial (internal) rotation, abduction, and flexion, and adductor spasm. Antalgic gait and lateral (external) rotation of the involved hip are likely to be observed.

A patient has a positive finding on the test shown in the photograph. The patient will MOST likely make which of the following subjective reports? A. Anterior shoulder pain and a recent fall while bicycling B. Shoulder instability and clicking with throwing activities C. Diffuse upper extremity pain and numbness into the lateral upper extremity D. Lateral shoulder pain and pain with overhead activities

D. Lateral shoulder pain and pain with overhead activities The photograph shows the Hawkins-Kennedy test for subacromial impingement. Patients who have subacromial impingement often describe a painful arc and pain with overhead activities. A- AC joint dysfunction B- labral tear C- thoracic outlet syndrome

An adult patient diagnosed with cystic fibrosis is receiving postural drainage and percussion. The patient is positioned in right side-lying with the foot of the bed elevated to 18 inches and a pillow placed between and under the knees. Which lobe is being drained? A. Left lower lobe lateral basal B. Right lower lobe lateral basal C. Left lower lobe posterior basal D. Left lower lobe anterior basal

D. Left lower lobe anterior basal To drain left lower anterior basal segment, the patient is right side lying with foot elevated to 18 inches. For posterior basal (option C), patient lies on abdomen with pillow under hips. For Lateral basal (option A&B) the patient lies on abdomen, head down, then rotates 1/4 turn upward.

A 4-year-old child has been referred to physical therapy for weakness and "failure to thrive". The child's family reports a gradual onset of weakness and lack of desire to participate in play. After a brief examination, the child appears significantly fatigued. What condition would MOST likely be consistent with these symptoms? A. Cerebral palsy B. Spina bifida C. Patent ductus arteriosus D. Leukemia

D. Leukemia Symptoms for children with leukemia: - General malaise, fatigue and lethargy, Prolonged or recurrent episodes of fever, Irritability, Growth restriction and/or failure to thrive, Shortness of breath and/or reduced exercise tolerance, Dizziness and palpitations, Bleeding diathesis, particularly causing epistaxis, bleeding gums and/or easy bruising, Bone or joint pain, particularly in the legs, Troublesome constipation, Prolonged cough, Headache, Nausea and vomiting, particularly if central nervous system (CNS) infiltration is present.

A physical therapist performs a muscle length test for the long head of the triceps. Which of the following findings it the MOST consistent with shortening of this muscle? A. Limitation of elbow flexion with the shoulder maintained at the end range of extension B. Limitation of elbow extension with the shoulder maintained at the end range of extension C. Limitation of elbow extension with the shoulder maintained at the end range of flexion D. Limitation of elbow flexion with the shoulder maintained at the end range of flexion

D. Limitation of elbow flexion with the shoulder maintained at the end range of flexion For a muscle length test you elongate the muscle in the direction OPPOSITE of its actions. So, the long head of the triceps extends the elbow and shoulder. Therefore, for the muscle length test you will FLEX the elbow and shoulder. Following full shoulder flexion the therapist will try to passively flex the elbow but will notice a limitation because the triceps have shortened and are unable to lengthen further.

A 50-year-old male is performing a Bruce protocol in your clinic with ECG leads attached. During the protocol, the PT sees the picture above. What does it mean and what should the PT do? A. CVA causing a lateral medullary syndrome and the PT should stop the protocol and call 911 B. First degree AV block and the PT should continue the protocol because this type is benign C. Myocardial ischemia that has an elevation of the ST segment less than 1 mm and the PT should stop the protocol and call 911 D. Myocardial infarction that has an elevation of the ST segment greater than 1 mm and the PT should stop the protocol and call 911

D. Myocardial infarction that has an elevation of the ST segment greater than 1 mm and the PT should stop the protocol and call 911 A myocardial infarction is an emergency situation that precipitates calling 911. A- a CVA is in the brain and will not be seen on an ECG B- First degree AV block portrays a PR interval > 0.2 seconds and each P wave has a corresponding QRS. There is no elevation in any AV block. C- this condition has a ST segment depression greater than 1 mm

A 58-year-old is performing a Bruce protocol in your clinic with ECG leads attached. During the protocol, the PT sees the picture shown below (elevation of almost 4 mm). What is the MOST likely diagnosis and intervention? A. Myocardial ischemia that has an elevation of the ST segment greater than 1 mm and the PT should stop the protocol B. Myocardial infarction that has an elevation of the ST segment less than 1 mm and the PT should stop the protocol and call 911 C. Myocardial ischemia that has an elevation of the ST segment less than 1 mm and the PT should stop the protocol D. Myocardial infarction that has an elevation of the ST segment greater than 1 mm and the PT should stop the protocol and call 911

D. Myocardial infarction that has an elevation of the ST segment greater than 1 mm and the PT should stop the protocol and call 911 There is an elevation of the ST segment which tells you they are having a myocardial infarction. You can rule out A and C. If it was less than 1 mm they would have been able to continue.

An otherwise healthy patient sustained a compression injury to the obturator nerve. When testing hip abduction, the patient is MOST likely to achieve which of the following strength grades? A. Zero (0/5) B. Trace (1/5) C. Good (4/5) D. Normal (5/5)

D. Normal (5/5) The obturator nerve innervates the adductor longus, adductor brevis, adductor magnus, gracilis, and obturator externus, whose action is to adduct the hip (p. 287). A lesion of the obturator nerve would not affect hip abduction (p. 286).

A 44-year-old male with complaints of left shoulder pain is being evaluated by a physical therapist. After a thorough examination, the only abnormal finding is a positive Kehr's sign. Which of the following is NOT a potential cause of a positive Kehr's sign? A. Recent laparoscopy B. Stomach ulcer C. Rupture of the spleen D. Trauma to head of pancreas

D. Trauma to head of pancreas Pain in the left shoulder with pressure placed on the upper abdomen, caused by free air or blood in the abdominal cavity is called Kehr's sign. A positive Kehr's sign can be caused by perforation of viscus (e.g., stomach ulcer (Option B), after laparoscopy (Option A), or after rupture of the spleen (Option C). Trauma to head of pancreas causes referred pain to the right (NOT left) shoulder, which makes Option D correct.


Ensembles d'études connexes

INFECTION CONTROL/CHAIN OF INFECTION

View Set

Mental Health Pharmacology practice questions

View Set

Chapter 14 - Site Survey Fundamentals

View Set